Add Maths Form5 Reference Book Answer

Download as pdf or txt
Download as pdf or txt
You are on page 1of 170

Learning Area: Geometry

1 Circular Measure
Sukatan Membulat
1.1 Radian/ Radian

Smart Tip
For a sector with centre O, when the arc length is equal to its radius,
Conversion of radian and degree
the angle at the centre is 1 radian or 1 rad.
Penukaran radian dan darjah
Bagi suatu sektor berpusat O, apabila panjang lengkok adalah sama dengan jejarinya,
sudut pusat ialah 1 radian atau 1 rad. 180°
1 rad =
2π rad = 360° π
π rad = 180° π
P 1° = rad
180°
2 cm PQ = Arc length/Panjang lengkok
O 1 rad 2 cm OP = Radius/Jejari 180° π
a rad = a × a° = a × rad
π 180°
Q
1 rad ≈ 57.29°

Exercise 1 Convert each of the following angles into degrees. [Use π = 3.142]
Tukarkan setiap sudut yang berikut kepada darjah. [Guna π = 3.142]
PL 2 Demonstrate the understanding of circular measure.

Example 1
1 0.46 rad 2 0.738 rad
(a) 1.267 rad
7 0.46 rad 0.738 rad
(b) π rad 180° 180°
8 = 0.46 × = 0.738 ×
π π
Solution = 26.35° = 42.28°
180°
(b) 1.267 rad = 1.267 ×
π
= 72.58° 3 1.5 rad 4 2.47 rad

7 7 180° 1.5 rad 2.47 rad


(a) π rad = π ×
8 8 π 180° 180°
= 1.5 × = 2.47 ×
= 157.5° π π
= 85.93° = 141.50°

5 4.726 rad π 3π
6 rad 7 rad
3 4
4.726 rad π 3π
180° rad rad
= 4.726 × 3 4
π π 180° 3π 180°
= × = ×
= 270.74° 3 π 4 π
= 60° = 135°

4π 5π 1
8 rad 9 rad 10 2 π rad
5 6 6
4π 5π 1
rad rad 2 π rad
5 6 6
4π 180° 5π 180° 13π 180°
= × = × = ×
5 π 6 π 6 π
= 144° = 150° = 390°

F5 Mod A+ ADDM 1(1-17).indd 1 5/1/2021 4:03:34 PM


(Guna π = 3.142)

Exercise 2 Convert each of the following angles into radian. [Use π = 3.142]
Tukarkan setiap sudut yang berikut kepada radian. [Guna π = 3.142]
PL 2 Demonstrate the understanding of circular measure.

Example 2
1 26° 2 37.5°
67.3° π π
26° = 26° × 37.5° = 37.5° ×
180° 180°
Solution = 0.4538 rad = 0.6546 rad
π
67.3° = 67.3° ×
180°
= 1.175 rad

3 54.65° 4 146°15 5 168°


π π π
54.65° = 54.65° × 146°15 = 146°15 × 168° = 168° ×
180° 180° 180°
= 0.9539 rad = 2.553 rad = 2.933 rad

6 195.67° 7 208°20 8 312°


π π π
195.67° = 195.67° × 208°20 = 208°20 × 312° = 312° ×
180° 180° 180°
= 3.416 rad = 3.637 rad = 5.446 rad

1.2 Arc Length of a Circle / Panjang Lengkok Suatu Bulatan

Exercise 3 By using the formula s = rθ, find the arc length, s for each of the following sectors.
[Use π = 3.142]
Dengan menggunakan s = rθ, cari panjang lengkok, s bagi setiap sektor yang berikut. [Guna π = 3.142]
PL 2 Demonstrate the understanding of circular measure.

Example 3

(a) (b) Smart Tip


s s

78° P
m O
O 9c 7 cm r
121° θ rad s
O

Solution Solution Q
π
78° = 78° × 360° – 121° s = rθ
180°
= 239°
= 1.362 rad π where,
239° × rad dengan keadaan,
180°
s = rθ s = arc length
= 4.172 rad panjang lengkok
= 9 × 1.362
r = radius/ jejari
= 12.258 cm θ = angle in radian
s = rθ
sudut dalam radian
= 7 × 4.172
= 29.204 cm

F5 Mod A+ ADDM 1(1-17).indd 2 5/1/2021 4:03:34 PM


1 2 3

s s

m
6 cm 4.968 rad

4c
O 0.83 rad O 1.36 rad O
s 8 cm

s = rθ s = rθ
s = rθ s = 8 × 4.968
s = 4 × 0.83 s = 6 × 1.36
= 8.16 cm = 39.744 cm
= 3.32 cm

4 5 6

12 m s
16 cm
48° 20 m 147°
152° s
O O O

π
48° = 48° × rad
180° π
= 0.838 rad 152° = 152° × rad 360° – 147° = 213°
180°
= 2.653 rad π
s = rθ 213° = 213° × rad
180°
= 12 × 0.838 s = rθ = 3.718 rad
= 10.056 m = 16 × 2.653
= 42.448 cm s = rθ
= 20 × 3.718
= 74.36 m

Exercise 4 By using the formula s = rθ, find the radius, r. [Use π = 3.142]
Dengan menggunakan rumus s = rθ, cari jejari, r. [Guna π = 3.142]
PL 2 Demonstrate the understanding of circular measure.

Example 4
1 2
P P 7 cm

P
Q
12 cm
15 cm r cm
0.986 rad 0.845 rad
O 0.8 rad
r cm O
Q O
r cm
Q

Solution rθ = s rθ = s
rθ = s r(0.8) = 12 r(0.845) = 7
r(0.986) = 15 12 7
r = 0.8 r = 0.845
15
r = 0.986 = 15 cm = 8.284 cm
= 15.213 cm

F5 Mod A+ ADDM 1(1-17).indd 3 5/1/2021 4:03:35 PM


3 4 5
16 cm 60 m 75 cm

P 3.96 rad 351.75°

r cm 122.6° rm Q r cm Q
O
O Q
P P
π
122.6° = 122.6° × π
180° 351.75° = 351.75° ×
rθ = s 180°
= 2.14 rad
r(3.96) = 60 = 6.14 rad
rθ = s 60
r(2.14) = 16 r = 3.96 rθ = s
16 = 15.152 m r(6.14) = 75
r = 2.14 75
r = 6.14
= 7.477 cm
= 12.215 cm

Exercise 5 Solve each of the following. [Use π = 3.142]


Selesaikan setiap yang berikut. [Guna π = 3.142]
PL 2 Demonstrate the understanding of circular measure.

Example 5
1 2
P P
O
6 cm
u 4 cm
Q 7.32 cm
u u
O Q O
P 5 cm
8.5 cm 6 cm
Q
Find the value of θ, in radian. Find the value of θ, in radian.
Cari nilai θ, dalam radian. Find the value of θ, in radian.
Cari nilai θ, dalam radian.
Cari nilai θ, dalam radian.
Solution
rθ = s
rθ = s rθ = s
(5) θ = 4
(6) θ = 8.5 4 (6) θ = 7.32
8.5 θ=5 7.32
θ= 6 θ= 6
= 0.8 rad
= 1.417 rad = 1.22 rad

3 4 5
P O Q
Q
7 cm 19 cm
x° P
8 cm 12 cm
Q x° O
12.8 cm O
P x°

Find the value of x. 68 cm


Cari nilai x. Find the value of x.
19 cm
Cari nilai x. Find the value of x.
rθ = s Cari nilai x.

(7) θ = 12.8 rθ = s
12.8 O 8 cm
(8) θ = 19
Q rθ = s
θ= 7 (12) θ = 68
19
= 1.829 rad θ= 8 68
P θ = 12
180° = 2.375 rad
x = 1.829 × = 5.667 rad
π 180°
= 104.78° x = 2.375 × 180°
π x = 5.667 ×
= 136.06° π
= 324.65°

F5 Mod A+ ADDM 1(1-17).indd 4 5/1/2021 4:03:36 PM


Smart Tip
From Cosine Rule/Daripada Petua Kosinus,
PR2 = r 2 + r 2 – 2(r)(r) cos θ/ kos θ
P PR2 = 2r 2 – 2r 2 cos θ/ kos θ
r PR = 2r 2 – 2r 2 cos θ/kos θ
O  Q
Perimeter of the shaded segment/Perimeter tembereng berlorek
= Length of chord PR + Arc length PQR
R Panjang perentas PR + Panjang lengkok PQR
= 2r 2 – 2r 2 cos θ/kos θ + rθ

θ in degree/ θ dalam darjah θ in radian/ θ dalam radian

Exercise 6 Calculate the perimeter of the shaded segment PRQ. [Use π = 3.142]
Hitung perimeter tembereng berlorek PRQ. [Guna π = 3.142]
PL 3 Apply the understanding of circular measure to perform simple tasks.

Example 6 1
P
P
Q
1.5 rad O 0.8 rad Q
O
9 cm R 15 cm
R

180°
1.5 rad = 1.5 × = 85.93° 180°
π 0.8 rad = 0.8 × = 45.83°
Perimeter of the shaded segment π
Perimeter tembereng berlorek Perimeter of the shaded segment
= 2r 2 – 2r 2 cos θ/kos θ + rθ = 2(15)2 – 2(15)2 cos 45.83° + (15)(0.8)
= 2(9)2 – 2(9)2 cos 85.93°/kos 85.93° + (9)(1.5) = 11.681 + 12
= 12.268 + 13.5 = 23.681 cm
= 25.768 cm

2 3
P P
Q

Q 1.74 rad
O 1.66 rad O R
12 cm
7 cm
R

180° 180°
1.66 rad = 1.66 × = 95.10° 1.74 rad = 1.74 × = 99.68°
π π
O 1.66 rad Q
Perimeter of the shaded segment Perimeter of the shaded segment
= 2(7)2 – 2(7)2 cos
9 cm95.10° + (7)(1.66) = 2(12)2 – 2(12)2 cos 99.68° + (12)(1.74)
= 10.33 + 11.62 R = 18.342 + 20.88
= 21.95 cm = 39.222 cm

F5 Mod A+ ADDM 1(1-17).indd 5 5/1/2021 4:03:37 PM


4 Q 5
P
P
2.56 rad 1.86 rad
R O 15 cm R
O 16 cm

180° 180°
2.56 rad = 2.56 × = 146.66° 1.86 rad = 1.86 × = 106.56°
π π
Perimeter of the shaded segment Perimeter of the shaded segment
= 2(16)2 – 2(16)2 cos 146.66° + (16)(2.56) = 2(15)2 – 2(15)2 cos 106.56° + (15)(2π – 1.86)
= 30.655 + 40.96 = 24.047 + 66.36
= 71.615 cm = 90.407 cm

Exercise 7 Solve each of the following. [Use π = 3.142]


Selesaikan setiap yang berikut. [Guna π = 3.142]
PL 4 Apply appropriate knowledge and skills of circular measure in the context of simple routine problem solving.

Example 7 1 The diagram shows a garden in the shape of sector.


Rajah menunjukkan sebuah taman berbentuk sektor.
The diagram shows an exhibition area for “Flower
Festival” in the shape of a major segment with
N
centre O.
Rajah menunjukkan sebuah kawasan pameran untuk “Pesta
Bunga” yang berbentuk tembereng major berpusat O. M 16 cm
2.265 rad

F O

G
The garden has to be fenced. Calculate the length, in
1.35 rad O
m, of the fence needed.
16 m Taman itu akan dipasang pagar. Hitung panjang, dalam m,
H pagar yang diperlukan.

Calculate the perimeter, in m, of the exhibition Length of the fence


area. = Arc length MN + OM + ON
Hitung perimeter, dalam m, bagi kawasan pameran itu. = (16)(2.265) + 16 + 16
= 36.24 + 32
Solution = 68.24 m
/FOH = 1.35 rad
180°
= 1.35 rad ×
π
= 77.34°

Perimeter
= FH + Arc length FGH/ Panjang lengkok FGH
= 2(16)2 – 2(16)2 cos 77.34° + (16)(2π − 1.35)
= 19.995 + 78.944
= 98.939 m

F5 Mod A+ ADDM 1(1-17).indd 6 5/1/2021 4:03:37 PM


2 The diagram shows a stage in the shape of sector 3 The diagram shows a plan of a resort that to be built.
with radius 4 m. OAB is a sector with centre O.
Rajah menunjukkan sebuah pentas berbentuk sektor berjejari Rajah menunjukkan pelan bagi sebuah resort yang akan dibina.
4 m. OAB ialah sektor berpusat O.

B
0.874 rad O
4m A 1.65 km

O
The curvatue part of the stage will be decorated with It is given that the perimeter of the resort is 7.425 km.
ribbons. Calculate the length, in m, of the ribbons Find /AOB, in radians.
needed. Diberi bahawa perimeter resort itu ialah 7.425 km. Cari /AOB,
Sekeliling panjang lengkok pentas itu akan dihiasi dengan reben. dalam radian.
Hitung panjang, dalam m, reben yang diperlukan.
OA + OB + Arc length AB = 7.425
Length of ribbons 1.65 + 1.65 + (1.65)(/AOB) = 7.425
= (4)(2π − 0.874) (1.65)(/AOB) = 4.125
= (4)(5.41) /AOB = 2.5 radians
= 21.64 m

4 The shaded region in the diagram below shows a 5 In the diagram below, the shaded region shows a
lawn. OPR is a sector with centre O. stage in the shape of a segment with centre O.
Kawasan berlorek dalam rajah di bawah menunjukkan kawasan Dalam rajah di bawah, kawasan berlorek menunjukkan sebuah
halaman rumput. OPR ialah sektor berpusat O. pentas berbentuk tembereng berpusat O.
P Q
R
6m
P
2.15 rad
θ 9m
O R
4m Q O
Calculate the perimeter, in m, of the lawn. The stage is to be surrounded by ribbons. Calculate
Hitung perimeter, dalam m, bagi kawasan halaman rumput. the length, in m, of the ribbons requirement.
Q
4 Sekeliling pentas itu akan diikat dengan reben. Hitung panjang,
cos θ = 6 dalam m, reben yang diperlukan.P R
θ = 48.19°
π 180° 9 m 2.15 rad
= 48.19° × 2.15 rad = 2.15 ×
180° π
= 0.8412 rad = 123.17°
O
PQ = 6 × sin 48.19° Perimeter
= 4.472 m = 2(9)2 – 2(9)2 cos 123.17° + (9)(2.15)
Perimeter = 15.831 + 19.35
= PQ + QR + Arc length PR = 35.181 m
= 4.472 + 2 + (6)(0.8412) The length of the ribbons required is 35.18 m.
= 11.519 m

1.3 Area of Sector of a Circle / Luas Sektor Suatu Bulatan

Smart Tip
Area of minor sector
Luas sektor minor
1 2 θ in radian
= rθ θ dalam radian
r
2
Minor sector Major sector
Sektor minor θ Area of major sector
2π - θ Sektor major
r Luas sektor major
1 2
= r (2π − θ)
2

F5 Mod A+ ADDM 1(1-17).indd 7 5/1/2021 4:03:38 PM


Exercise 8 Find the area of the following shaded sectors. [Use π = 3.142]
Cari luas bagi sektor berlorek yang berikut. [Guna π = 3.142]
PL 2 Demonstrate the understanding of circular measure.

Example 8 1 2
P
P

P O 0.85 rad 78˚


7 cm Q O Q
6 cm
O 83˚
9 cm Q

Area of sector OPQ


1 ∠POQ = 78°
Solution = 2 r2 θ π
= 78° ×
∠POQ = 83° 180°
π 1
= 83° × = 2 × 72 × 0.85 = 1.362 rad
180°
= 1.449 rad = 20.83 cm2 Area of sector OPQ
1
Area of sector OPQ = r2 θ
2
Luas sektor OPQ 1
1 = 2 × 62 × 1.362
= 2 r2 θ
1 = 24.52 cm2
= 2 × 92 × 1.449
= 58.68 cm2

Common Error
1
(a) 2 r2 θ
1
= 2 × 92 × 83°
= 3 361.5 cm2
θ must be converted to 3 4
radian. P
θ mesti ditukarkan kepada

radian. 3 O 14 cm
P
98 cm
1 O 160˚
(b) 2 r2 θ
1 Q
= 2 × 92 × 1.4 Q
= 56.7 cm2 Area of sector OPQ ∠POQ = 360° – 160° = 200°
The value of θ must be in 1 π
= 2 r2 θ 200° = 200° × 180°
at least 3 decimal places.
Nilai θ mesti sekurang- 1 5 = 3.491 rad
kurangnya dalam 3 tempat = 2 × 92 × 3 π
perpuluhan. Area of sector OPQ
= 212.085 cm2
1
= 2 r2 θ
1
= 2 × 142 × 3.491
= 342.12 cm2

F5 Mod A+ ADDM 1(1-17).indd 8 5/1/2021 4:03:39 PM


Exercise 9 Given the area of the shaded sector OPQ, find the radius of each of the following.
[Use π = 3.142]
Diberi luas sektor berlorek OPQ, cari jejari bagi setiap yang berikut. [Guna π = 3.142]
PL 2 Demonstrate the understanding of circular measure.

Example 9 1 Area/Luas = 7.84 cm2 2 Area/Luas = 116 cm2

Area/Luas = 36 cm2
P
P 1.486 rad
O O
r cm
0.684 rad r cm
1.2 rad Q
O r Q
P Q

1 2 1 2
2 r θ = Area of sector OPQ 2 r θ = Area of sector OPQ
Solution 1 2 1 2
1 2 2 × r × (0.684) = 7.84 2 × r × (1.486) = 116
7.84 × 2 116 × 2
2 r θ = Area of sector OPQ r2 = 0.684 r2 = 1.486
Luas sektor OPQ
= 22.924 = 156.124
1 2
r = 12.495 cm
2 × r × (1.2) = 36 r = 4.788 cm
36 × 2
r2 = 1.2
= 60
r = 60
= 7.746 cm

3 Area/Luas = 378 cm2 4 Area/Luas = 471.28 cm2 5 Area/Luas = 535.43 cm2

P 1.034 rad
r cm P
1.034 rad 2.095 rad
O Q
Q P
O r cm O
47˚
r cm
Q
1 2 1 2
2 r θ = Area of sector OPQ 2 r θ = Area of sector OPQ π
1 2 47° × 180° = 0.8204 rad
1 2
2 × r × (2π − 1.034) = 378 2 × r × (2π − 2.095) = 471.28 1 2
1 2 1 2 2 r θ = Area of sector OPQ
2 × r × 5.25 = 378 2 × r × 4.189 = 471.28 1 2
378 × 2
r2 = 5.25 471.28 × 2
r2 = 4.189 2 × r × (2π − 0.8204) = 535.43
1 2
= 144 = 225 2 × r × 5.4636 = 535.43
r = 12 cm r = 15 cm 535.43 × 2
r2 = 5.4636
= 196
r = 14 cm

F5 Mod A+ ADDM 1(1-17).indd 9 5/1/2021 4:03:39 PM


Exercise 10 Given the area of the shaded sector OPQ, find the value of θ, in radian, for each of the following.
[Use π = 3.142]
Diberi luas sektor berlorek OPQ, cari nilai θ, dalam radian, bagi setiap yang berikut. [Guna π = 3.142]
PL 2 Demonstrate the understanding of circular measure.

Example 10 1 Area/Luas = 6.982 cm2 2 Area/Luas = 102.8 cm2

Area/Luas = 39.872 cm2 P Q


P
P
θ 48 cm
O θ
θ O
O 8 cm Q 12 cm
Q

Solution
1 2 1 2
1 2
2 r θ = Area of sector OPQ 2 r θ = Area of sector OPQ
2 r θ = Area of sector OPQ
Luas sektor OPQ 1 1
2 × 4 × θ = 6.982
2

2 × 12 × θ = 102.8
2
1 6.982 × 2
θ= 102.8 × 2
2 × 8 × θ = 39.872
2
42 θ=
39.872 × 2 122
θ= θ = 0.8728 rad θ = 1.428 rad
82
= 1.246 rad

3 Area/Luas = 128.30 cm2 4 Area/Luas = 240 cm2 5 Area/Luas = 693.76 cm2


P Q

14 cm P
P 16 cm
θ
7 cm O θ O
θ
O

Q
Q
1 2 1 2 1 2
2 r θ = Area of sector OPQ 2 r (2π − θ) = Area of sector OPQ 2 r (2π − θ) = Area of sector OPQ
1 1 1
2 × 7 × θ = 128.30
2
2 × 14 × (2π − θ) = 240 2 × 16 × (2π − θ) = 693.76
2 2
128.30 × 2 2π − θ = 2.449 2π − θ = 5.42
θ=
72 θ = 2π − 2.449 θ = 2π − 5.42
θ = 5.237 rad
= 3.835 rad = 0.864 rad

10

F5 Mod A+ ADDM 1(1-17).indd 10 5/1/2021 4:03:40 PM


Smart Tip
Area of segment PTQ = Area of sector OPTQ – Area of triangle OPQ
Luas tembereng PTQ = Luas sektor OPTQ – Luas segi tiga OPQ
1 2 1 O
O = r θ – r2 sin θ
2 2 Remember/Ingat

(
θ
θ r 1 2 1 2 1
= r (θ – sin θ) r θ – r2 sin θ
2 Q P 2 2
Q P 1 2
r sin θ
T 2 In radian In degree
Dalam radian ( rad) Dalam darjah (°)

Exercise 11 Find the area of shaded segment of each of the following. [Use π = 3.142]
Cari luas bagi tembereng berlorek bagi setiap yang berikut. [Guna π = 3.142]
PL 3 Apply the understanding of circular measure to perform simple tasks.

Example 11

Solution
180° Alternative Method
0.85 rad = 0.85 × = 48.7°
π
Area of shaded segment Area of shaded segment
Luas tembereng berlorek Luas tembereng berlorek
O 1
1 1 = 2 r2 (θ − sin θ)
8 cm 0.85
1.45 rad = 2 × r2 × θ − 2 × r2 × sin θ
1
1 1 = 2 × (8)2 (0.85 − sin 48.7°)
Q P = 2 × (8)2 × 0.85 − 2 × (8)2 × sin 48.7°
= 32(0.85 – 0.7513)
= 27.2 − 24.040 = 3.158 cm2
= 3.160 cm2

1 2

O
O
12 cm
P 3.1 rad
12 cm 1.45 rad
Q
Q P

180° 180°
1.45 rad = 1.45 × = 83.07° 3.1 rad = 3.1 × = 177.59°
π π
Area of shaded segment Area of shaded segment
1 1 1 1
= × r2 × θ − × r2 × sin θ = × r2 × θ × − × r2 × sin θ
2 2 2 2
1 1 1 1
= 2 × (12)2 × 1.45 − 2 × (12)2 × sin 83.07° = 2 × 122 × 3.1 − 2 × 122 × sin 177.59°
= 104.4 − 71.47 = 223.2 – 3.028
= 32.93 cm2 = 220.17 cm2

11

F5 Mod A+ ADDM 1(1-17).indd 11 5/1/2021 4:03:41 PM


3 4
P P

2.43 rad
Q O 68˚
O 14 cm
16 cm
Q

180° π
2.43 rad = 2.43 × = 139.21° 68° = 68° × = 1.187 rad
π 180°
Area of shaded segment Area of shaded segment
1 1 1 1
= × r2 × θ − × r2 × sin θ = × r2 × θ − × r2 × sin θ
2 2 2 2
1 1 1 1
= 2 × (14)2 × 2.43 − 2 × (14)2 × sin 139.21° = 2 × (16)2 × 1.187 − 2 × (16)2 × sin 68°
= 238.14 − 64.022 = 151.936 − 118.680
= 174.12 cm2 = 33.26 cm2

5 Q 6
P

15 cm
1.4 rad
136° O Q
O 12 cm

π 180°
136° = 136° × = 2.374 rad 1.4 rad = 1.4 × = 80.2°
180° π
Area of shaded segment
Area of shaded segment 1 1
1 1 2 
= πr2 − r2θ − r2 sin θ
2 
= × r2 × θ − × r2 × sin θ
2 2 1 1
1 1 2 
= 3.142 × 12 − × (12)2 × 1.4 − × (12)2 × sin 80.2
2
2 
= 2 × (15)2 × 2.374 − 2 × (15)2 × sin 136° = 452.448 – (100.8 – 70.95)
= 267.075 − 78.149 = 422.60 cm2
= 188.93 cm2

Exercise 12 Solve the following problems.


Selesaikan masalah yang berikut.
PL 4 Apply appropriate knowledge and skills of circular measure in the context of simple routine problem solving.

1 The diagram shows a semicircle with


a diameter of 16 cm. Let O be the centre of the semicircle.
Rajah menunjukkan sebuah semibulatan berdiameter 16 cm. ∠COB = 2 × ∠CAO
2 × 0.68 = 1.36 rad
C π − 1.36 rad = 1.782 rad
180o
1.782 × π = 102.09o

A 0.68 rad C
B
16 cm 1.782 rad
Find the area, in cm2, of the shaded region.
Cari luas, dalam cm2, bagi rantau berlorek.
0.68 rad 1.36 rad
[Use/Guna π = 3.142] A B
O 8 cm

Area of the shaded region
= area of ∆AOC + area of sector OCB
1 1
= 2 × 82 ×(sin 102.09o) + 2 × 82 × 1.36
= 31.29 + 43.52
= 74.81 cm2

12

F5 Mod A+ ADDM 1(1-17).indd 12 5/1/2021 4:03:41 PM


2 In the diagram, CAD is a sector with centre A and EBD is a semicircle with C
diameter ED. It is given that AB = 18 cm and ∠CAD = π radian.
3
Dalam rajah, CAD ialah sebuah sektor berpusat A dan EBD ialah sebuah semibulatan B
π
berdiameter ED. Diberi bahawa AB = 18 cm dan ∠CAD = radian.
3
Find/Cari
A
(a) the perimeter, in cm, of the shaded region, Applying
E D
perimeter, dalam cm, bagi rantau berlorek,
(b) area, in cm2 of the shaded region./ luas, dalam cm2, bagi rantau berlorek.

(a) Let O be the centre of semicircle EBD. AD = AO + OD


∠ABO = 90o (AC = tangent to semicircle EBD) = 36 cm + 31.18 cm
= 67.18 cm
C
π
o
90 = 2 rad
B π π 5π
∠BOD = 3 + 2 = 6
18 cm
)
Perimeter of the shaded region
A D = BC + arc length BD + arc length CD
π E O

π
3
rad
π 180o

( ) π
= (67.18 − 18) + (31.18) 6 + (67.18) 3 ()
rad = 3 × π = 60
o
3 = 49.18 + 81.64 + 70.36
AB = 201.18 cm
AO = cos 60
o

18 (b) Area of the shaded region


AO = cos 60
o
= sector ACD − ∆ABO − sector BOD
18
AO = cos 60° = 36 cm ( 1 π
) ( 1
= 2 × 67.182 × 3 − 2 × 18 × 31.18 − )
AB + BO = AO
2 2

182 + BO2 = 362


2
[ 1 5π
2 × (31.18)2 × 6 ]
BO2 = 972 = 2 363.39 − 280.62 − 1 272.76
BO = 972 = 810.01 cm2
BO = 31.18 cm
Radius = 31.18 cm

3 The diagram shows the plan of a garden. HQG is a semicircle with centre O and has a radius of 10 m. FPG is
a sector with centre P and has a radius of 18 m. Sector OQG is a lawn. The shaded region is a flower bed. The
whole flower bed has to be fenced. It is given that PQ is 10 m and ∠QOG = 1.795 rad.
F
Rajah menunjukkan pelan bagi sebuah taman. HQG ialah sebuah semibulatan berpusat O dan
mempunyai jejari 10 m. FPG ialah sebuah sektor berpusat P dan mempunyai jejari 18 m. Sektor Q
OQG ialah kawasan halaman rumput. Rantau berlorek ialah kawasan tanaman bunga. Seluruh
kawasan tanaman bunga perlu dipagarkan. Diberi bahawa PQ ialah 10 m dan ∠QOG =
1.795 rad.
Calculate/ Hitung Evaluating
H G
P O
(a) the area, in m2, of the lawn,/ luas, dalam m2, bagi kawasan halaman rumput,
(b) the length, in m, of the fence required for fencing the flower bed,
panjang, dalam m, pagar yang diperlukan untuk memagar kawasan tanaman bunga,
(c) the area, in m2, of the flower bed./ luas, dalam m2, bagi kawasan tanaman bunga.
[Use/Guna π = 3.142]
F
8m (c) Area of sector FPG – area of ∆QPO –
∠FPO = ∠QOP Q
area of sector OQG
= π – 1.795 rad
= 1.347 rad 10 m 10 m ( 1
) (
1
= 2 × 182 × 1.347 − 2 × 10 × 8 sin 77.17o − )
180 o
= 1.347 ×
= 77.17 o
π H
P
)1.347 rad )1.795 rad
8m O 10 m
G ( 1
2 × 102 × 1.795 )
= 218.21 − 39.001 − 89.75
1
(a) 2 × 102 × 1.795 = 89.75 m2 = 89.46 m2

(b) Length of the fence


= QF + arc length FG + arc length QG
= 8 + 18(1.347) + 10(1.795)
= 50.20 m

13

F5 Mod A+ ADDM 1(1-17).indd 13 5/1/2021 4:03:41 PM


1.4 Application of Circular Measures/ Aplikasi Sukatan Membulat

Exercise 13 Solve the following problems. Applying

Selesaikan masalah yang berikut.


PL 4 Apply appropriate knowledge and skills of circular measure in the context of simple routine problem solving.

1 The diagram shows the plan of a flower garden in the shape of a


semicircle with centre O and a diameter of 24 m. The sector OKL is to L
be planted with rose trees. The arc length KL is equal to the sum of the
length OL and the arc length LM.
Rajah menunjukkan pelan bagi sebuah taman bunga yang berbentuk semibulatan
berpusat O dan diameter 24 m. Sektor OKL akan ditanam pokok bunga ros. Panjang
K M
lengkok KL adalah sama dengan jumlah panjang OL dan panjang lengkok LM.
O
Wall of building
If each area of 2 m2 can be planted with 5 rose trees, find the Dinding bangunan

number of rose trees that can be planted.


Jika setiap luas 2 m2 boleh ditanam dengan 5 pokok bunga ros, cari bilangan
pokok bunga ros yang boleh ditanam.

Given diameter = 24 m, [ radius = 12 m Area to be planted with rose trees is 149.11 m2.
Given 2 m2 can plant 5 rose trees. Thus, 1 m2
Arc length KL = OL + arc length LM 5
can plant rose trees.
12 × /KOL = 12 + 12(p − /KOL) 2
/KOL = 1 + p − /KOL
Total number of rose trees to be planted
2/KOL = 1 + p 5
= 149.11 ×  
2
1
/KOL = 2   1 + p ( ) = 372.78
373 rose trees
= 2.071 rad

1
Area sector KOL = 2  × 2.071 × 122
= 149.11 m2

2 The diagram shows the plan of a park. EOF and GOH are two sectors
with common centre O. Given GO = 20 m, GE = EO and the arc length
GH = 8p m.
Rajah menunjukkan pelan bagi sebuah taman. EOF dan GOH ialah dua buah sektor
G H
dengan pusat sepunya O. Diberi GO = 20 m, GE = EO dan panjang lengkok GH =
8p m. E F
O
The shaded region will be covered with tiles. If the cost to cover an area
of 1 m2 is RM150, calculate the total cost.
Kawasan berlorek akan ditutupi dengan jubin. Jika kos untuk menutup kawasan seluas
1 m2 ialah RM150, hitung jumlah kos.

Arc length GH = GO × /GOH


8p = 20 × /GOH
2p
/GOH = 5 rad

Area of the shaded region = Area of sector OGH – Area of sector OEF
1 2p 1 2p
= 2  × 202 × 5 − 2 × 102 × 5  
= 251.36 – 62.84
= 188.52 m2

Total cost = 188.52 × RM150


= RM28 278

14

F5 Mod A+ ADDM 1(1-17).indd 14 5/1/2021 4:03:42 PM


Review 1
Paper 1

Section A 3 Diagram 3 shows two sectors, EOH and FOG with


1 Diagram 1 shows two sectors, ODE and OFG with centre O.
centre O. Rajah 3 menunjukkan dua buah sektor, EOH dan FOG
Rajah 1 menunjukkan dua buah sektor, ODE dan OFG berpusat berpusat O.
O.
F
O
D G
F H
θ
O
E
E Diagram 3 / Rajah 3
G The angle subtended at the centre O by the major
Diagram 1 / Rajah 1 arc EH is 6β radians and the perimeter of the whole
It is given that OD = DF, OF = 18 cm and the arc length diagram is (60 + 4r) cm. Given FO = r cm, EO = 3FO
FG is 36 cm. and /FOG = 2α, express
Diberi bahawa OD = DF, OF = 18 cm dan panjang lengkok FG Sudut yang dicangkum pada pusat O oleh lengkok major EH
ialah 36 cm. ialah 6β radian dan perimeter seluruh rajah ialah (60 + 4r) cm.
[Use/ Guna = 3.142] Diberi FO = r cm, EO = 3FO dan /FOG = 2α, ungkapkan
Find/ Cari (a)  in terms of ,
(a) the value of θ, in radians,  dalam sebutan ,
nilai θ, dalam radian, (b) r in terms of .
(b) the area, in cm2, of the sector ODE. r dalam sebutan .
luas, dalam cm , bagi sektor ODE.
2 [5 marks/markah]
[4 marks/markah]
4 Diagram 4 shows a sector OEF with centre O and a
2 Diagram 2 shows a sector QOR with centre O. sector GEH with centre E.
Rajah 2 menunjukkan sebuah sektor QOR berpusat O. Rajah 4 menunjukkan sebuah sektor OEF berpusat O dan
sebuah sektor GEH berpusat E.
R E
H
G
Q
S
P O
4.334 rad θ
O
Diagram 2 / Rajah 2
It is given that QP = PO = OS = SR = 7 cm and the F
reflex angle QOR = 4.334 radians. Diagram 4 / Rajah 4
Diberi bahawa QP = PO = OS = SR = 7 cm dan sudut refleks
Given OF = 12 cm, EG = 5 cm, /GEH = 0.82 radian and
QOR = 4.334 radian.
the arc length EF = 32 cm, calculate
[Use/ Guna = 3.142]
Diberi OF = 12 cm, EG = 5 cm, /GEH = 0.82 radian dan
Find/ Cari
panjang lengkok EF = 32 cm, hitung
(a) the length, in cm, of the arc QR,
(a) the value of θ, in radians,
panjang, dalam cm, bagi lengkok QR,
nilai θ, dalam radian,
(b) the area, in cm2, of the shaded region.
(b) the area, in cm2, of the shaded region.
luas, dalam cm2, bagi kawasan berlorek.
luas, dalam cm2, bagi kawasan berlorek.
[5 marks/markah]
[4 marks/markah]

15

F5 Mod A+ ADDM 1(1-17).indd 15 5/1/2021 4:03:42 PM


Paper 2
Section A
1 Diagram 1 shows a semicircle EHF with centre O and radius 8 cm. GKH is a quadrant of a circle with centre K.
Rajah 1 menunjukkan sebuah semibulatan EHF berpusat O dan berjejari 8 cm. GKH ialah sebuah sukuan bulatan berpusat K.

8 cm
3.5 cm
2.69 rad

G E K O F

Diagram 1 / Rajah 1
Given HK = 3.5 cm and /HOF = 2.69 radians.
Diberi HK = 3.5 cm dan /HOF = 2.69 radian.
[Use/ Guna = 3.142]
Calculate/ Hitung
(a) the perimeter, in cm, of the whole diagram,
perimeter, dalam cm, bagi seluruh rajah,
[4 marks/markah]
(b) the area, in cm2, of the shaded region.
luas, dalam cm2, bagi kawasan berlorek.
[3 marks/markah]

2 Diagram 2 shows a circle with centre O.


Rajah 2 menunjukkan sebuah bulatan berpusat O.
P

Q θ O S

Diagram 2 / Rajah 2
1
Given the arc length PQR is 31.5 cm and the area of the shaded region is 118 8 cm2.
1
Diberi panjang lengkok PQR ialah 31.5 cm dan luas kawasan berlorek ialah 118 cm2.
8
[Use/ Guna = 3.142]
Calculate/ Hitung
(a) the radius, in cm, of the circle,
jejari, dalam cm, bagi bulatan,
[3 marks/markah]
(b) θ, in radians,
θ, dalam radian,
[2 marks/markah]
(c) the area, in cm2, of the segment PRS.
luas dalam cm2, bagi tembereng PRS.
[3 marks/markah]

3 Diagram 3 shows a semicircle with centre O and a diameter of 26 cm. BA is a tangent to the semicircle at point A.
Rajah 3 menunjukkan sebuah semibulatan berpusat O dan berdiameter 26 cm. BA ialah tangen kepada semibulatan di titik A.
B

A D
O
Diagram 3 / Rajah 3

16

F5 Mod A+ ADDM 1(1-17).indd 16 5/1/2021 4:03:43 PM


It is given that the arc length CD is 27.3 cm.
Diberi bahawa panjang lengkok CD ialah 27.3 cm.
[Use/ Guna = 3.142]
Calculate/ Hitung
(a) /COD in radians,
/COD dalam radian,
[3 marks/markah]
(b) the area, in cm2, of the shaded region.
luas, dalam cm2, kawasan berlorek.
[4 marks/markah]

Section B
4 Diagram 4 shows a circle with centre O and radius 18 cm inscribed in a sector EFG with centre E. The straight
lines EF and EG are tangents to the circle at point H and point K respectively.
Rajah 4 menunjukkan sebuah bulatan berpusat O dan berjejari 18 cm terterap di dalam sektor EFG berpusat E. Garis lurus EF dan EG
ialah tangen kepada bulatan masing-masing pada titik H dan titik K.
P

F G
18 cm O
H K

60˚

E
Diagram 4/ Rajah 4
Calculate/Hitung
(a) the length, in cm, of the arc FG,
panjang, dalam cm, bagi lengkok FG,
[5 marks/markah]
(b) the area, in cm2, of the shaded region.
luas, dalam cm2, bagi kawasan berlorek.
[5 marks/markah]

H O TS Zo n e
1 Diagram 1 shows two circles with centres K and L respectively and touch at point P. The larger circle has a radius
of 14 cm and the smaller circle has a radius of 10 cm. The straight line FG is a common tangent to the circles at
point F and point G.
Rajah 1 menunjukkan dua buah bulatan masing-masing berpusat K dan L dan bersentuh pada titik P. Bulatan yang lebih besar mempunyai
jejari 14 cm dan bulatan yang lebih kecil mempunyai jejari 10 cm. Garis lurus FG ialah tangen sepunya kepada kedua-dua bulatan itu pada
titik F dan titik G.

P
L
K
θ
10 cm
10 cm 14 cm

F G
Diagram 1/ Rajah 1
Given ∠KLG = θ radians,
Diberi ∠KLG = θ radian,
(a) show that θ = 1.404 (round off to three decimal places), Applying

tunjukkan θ = 1.404 (bundarkan kepada tiga tempat perpuluhan),


(b) calculate the length, in cm, of the minor arc FP,
hitung panjang, dalam cm, bagi lengkok minor FP,
(c) calculate the area, in cm2, of the shaded region. Evaluating

hitung luas, dalam cm2, bagi kawasan berlorek.

17

F5 Mod A+ ADDM 1(1-17).indd 17 5/1/2021 4:03:43 PM


Learning Area: Calculus

2 Differentiation
Pembezaan
2.1 Limit and its Relation to Differentiation / Had dan Hubungannya dengan Pembezaan

Smart Tip
Limit/Had Limit idea/ First principles:
lim 𝑓(𝑥)= C can be read as “the limit of f of x, as x Idea had/ Prinsip pertama:
x→a
approaches a, is C”. dy δy
= lim , where 𝛿x and 𝛿y are small changes of x and y
had f(x) = C boleh dibaca sebagai “had f bagi x, apabila x menghampiri a, dx δx → 0 δx
x→a
ialah C”. respectively;
dy had δy
dx = δx → 0 δx , dengan keadaan 𝛿x dan 𝛿y masing-masing adalah
perubahan kecil dalam x dan y;
or/atau
dy / ℎ𝑎𝑑 f(x + δx)
x − f(x)
= f ′(x) = δxlim
dx → 0 δx → 0
δx

Exercise 1 Evaluate each of the following.


Nilaikan setiap yang berikut.
PL 2 Demonstrate the understanding of differentiation.

Example 1
1 lim / ℎ𝑎𝑑 (13 − 2x)
x→2 x→2

lim / ℎ𝑎𝑑 x − 9
2
= 13 − 2(2)
x→3 x→3 x − 3
=9
Solution
lim / ℎ𝑎𝑑 x − 9 = lim / ℎ𝑎𝑑 (x + 3)(x − 3)
2

x→3 x→3 x − 3 x→3 x→3 x−3


= lim / ℎ𝑎𝑑 (x + 3)
x→3 x→3
= (3) + 3
=6

2 lim / ℎ𝑎𝑑 (x2 − 2x − 7) 3 lim / ℎ𝑎𝑑 2x


4 lim / ℎ𝑎𝑑 x + 2
x → –3 x → –3 x→3 x→3
4x − 2 x→6 x→6
x+6
= (–3) − 2(–3) − 7
2

=9+6−7 2(3) 6+2


= 4(3) − 2 = 6+6
=8
6 3 8 2
= 10 = 5 = 12 = 3

lim / ℎ𝑎𝑑 x − 1 lim / ℎ𝑎𝑑 4 − x


2 2
lim / ℎ𝑎𝑑 x − 25
2
5 6 7
x→5 x→5
x−5 x→1 x→1
3x − 3 x→2 x→2
2x − 4
(2 − x)(2 + x)
(x − 5)(x + 5) (x − 1)(x + 1) = xlim
= xlim = xlim →2 –2(2 − x)
→5 x−5 →1 3(x − 1)
2+x 2+2
x+1 1+1 2 = xlim
= xlim = xlim –2 = –2 = –2
(x + 5) = 5 + 5 = 10 3 = 3 =3
→2
→5 →1

lim / ℎ𝑎𝑑 3x − 3 lim / ℎ𝑎𝑑 2x − 18


2 2
lim / ℎ𝑎𝑑 x − 16
2
8 9 10
x → –4 x → –4 x + 4 x→1 x→1
x−1 x→3 x→3
x−3
3(x − 1)(x + 1) 2(x − 3)(x + 3)
(x − 4)(x + 4) = xlim = xlim
= xlim →1 x−1 →3 x−3
→ –4 x+4
= xlim 3(x + 1) = 3(1 + 1) = 6 = xlim 2(x + 3) = 2(3 + 3) = 12
lim
= x → –4 x − 4 = –4 − 4 = –8 →1 →3

18

F5 Mod A+ ADDM 2(18-49).indd 18 6/1/2021 10:59:21 AM


Exercise 2 Evaluate each of the following.
Nilaikan setiap yang berikut.
PL 2 Demonstrate the understanding of differentiation.

Example 2
1 lim / ℎ𝑎𝑑 (0.8)n
n→∞ n→∞

lim / ℎ𝑎𝑑 3 lim / ℎ𝑎𝑑 2n


(a) lim / ℎ𝑎𝑑 (0.5)n (b) n→∞ (c) n→∞ = (0.8)∞
n→∞ n→∞
n→∞
2n − 5 n→∞
n−2
=0
Solution
(c) nlim / ℎ𝑎𝑑 2n ÷ n
(a) lim / ℎ𝑎𝑑 (0.5)n = (0.5)∞ → ∞ n → ∞ (n − 2)
÷n
n→∞ n→∞
=0 2
= nlim / ℎ𝑎𝑑
→∞ n→∞ 1 − 2 lim / ℎ𝑎𝑑 9
(b) nlim / ℎ𝑎𝑑 3 3
→ ∞ n → ∞ 2n − 5 = 2(∞) − 5 n
2 n→∞ n→∞  2n 
2
3 = 9
2 =∞
=∞ 1−

=0 =0
2
=
1−0 =2

lim / ℎ𝑎𝑑 4n − 3 lim / ℎ𝑎𝑑 2 lim / ℎ𝑎𝑑 3


3 n→∞ n→∞
n   4 n→∞ n→∞ n − 2 5 n→∞ n→∞ 3 + n
lim 4n 3 2 3
= n→∞
n n
−  = (∞) − 2 =3+∞
lim 3 2 =3+0
= n→∞ 4 −
n  =∞ =3
=4− 3 =4−0=4 =0

4n lim / ℎ𝑎𝑑 3n − 2 lim / ℎ𝑎𝑑 3n2 − 2


6
lim / ℎ𝑎𝑑
n→∞ n→∞ n+3 7 n→∞ n→∞ 
3 + 2n  8 n→∞ n→∞  5 + 8n  2

2 3n − 2
2

→ ∞  5 + 8n2 
3−
 
4 = nlim
= nlim n
1+
3 = nlim
→∞ →∞ 3 2 2

  
n +2 3− 3−
n n2 ∞2
= 4 2 = nlim =
3 3− →∞ 5 5
1+ ∞ +8 +8
∞ = n2 ∞2
4 3 3−0 3
+2
= 1+0 ∞ = 0+8 = 8
3−0 3
=4 = 0+2 = 2

Exercise 3 Evaluate each of the following.


Nilaikan setiap yang berikut.
PL 2 Demonstrate the understanding of differentiation.

Example 3
lim / ℎ𝑎𝑑 m − 3 m− 6
1 m→3 m→3 
m2 − 9  2 lim / ℎ𝑎𝑑
m→6 m→6 m 2
− 36 
lim / ℎ𝑎𝑑 m – 4
2

m→2 m→2
m–2  
m− 3 m− 6
= mlim
→ 3 (m + 3)(m − 3)
= mlim
→ 6 (m + 6)(m − 6)
Solution
lim / ℎ𝑎𝑑 m – 4
2
1 1
m→2 m→2
m–2   Common Error
= mlim
→3 m + 3
= mlim
→6 m + 6

lim / ℎ𝑎𝑑 (m + 2)(m – 2) 1 1


= m→2 m→2 lim / ℎ𝑎𝑑 m – 4
2
= (3) + 3 = (6) + 6
m–2 m→2 m→2
m–2
= lim / ℎ𝑎𝑑 (m + 2) (2)2 – 4 1 1
m→2 m→2 =
(2) – 2
=6 = 12
= (2) + 2
=4 0
=
0

19

F5 Mod A+ ADDM 2(18-49).indd 19 6/1/2021 10:59:22 AM


lim / ℎ𝑎𝑑 25 − m lim / ℎ𝑎𝑑 2m − 8 lim / ℎ𝑎𝑑 3m − 3
2 2 2
3 m→5 m→5
5−m   4 m→2 m→2 m−2  5 m→1 m→1 
m−1 
(5 + m)(5 − m) 2(m2 − 4) 3(m2 − 1)
= mlim = mlim = mlim
→5 5−m →2 m−2 →1 m−1
lim
= m → 5 (5 + m) 2(m + 2)(m − 2) 3(m + 1)(m − 1)
= mlim
→2 m– 2 = mlim
→1 m– 1
= 5 + (5)
lim
= m → 2 2(m + 2) lim
= m → 1 3(m + 1)
= 10
= 2[(2) + 2] = 3[(1) + 1]
= 2(4) = 3(2)
=8 =6

dy f(x + δx) – f(x) dy


Exercise 4 By using the formula dx = δlim
x→0 δx , find dx .
dy f(x + δx) – f(x) dy
Dengan menggunakan rumus = had δx , cari dx .
dx δx → 0
PL 3 Apply the understanding of differentiation to perform simple tasks.

Example 4 1 y = 4x + 5
y = 7x2
dy f(x + δx) – f(x)
= lim δx
Solution dx δx → 0
[4(x + δx) + 5] – (4x + 5)
dy lim had f(x + δx) – f(x) = δlim
= /
dx δx → 0 δx → 0 δx x→0 δx
7(x + δx)2 – 7x2 4x + 4δx + 5 – 4x – 5
= δlim / had = δlim
x→0 δx
x → 0 δx → 0 δx
7[x2 + 2xδx + (δx)2] – 7x2 4δx
= δlim / had = δlim
x → 0 δx
x → 0 δx → 0 δx
7x2 + 14xδx + 7(δx)2 – 7x2 = lim 4
= δlim / had δx
δx → 0
x → 0 δx → 0 =4
14xδx + 7(δx)2
= δlim / had
x → 0 δx → 0 δx
lim had
= δx → 0 / δx → 0 (14x + 7δx)
= 14x + 7(0)
= 14x

2 y = 2x2 3 y = x2 + x

dy f(x + δx) – f(x) dy f(x + δx) – f(x)


= lim δx = lim δx
dx δx → 0 dx δx → 0
2(x + δx)2 – 2x2 (x + δx)2 + (x + δx) – (x 2 + x)
= δlim
x→0 δx = δlim
x→0 δx
2[x2 + 2xδx + (δx)2] – 2x2 x2 + 2xδx + (δx)2 + x + δx – x2 – x
= δlim
x→0 δx = δlim
x→0 δx
2x2 + 4xδx + 2(δx)2 – 2x2 2xδx + (δx)2 + δx
= δlim
x→0 δx = δlim
x→0 δx
4xδx + 2(δx)2
= δlim
x→0 δx = δlim
x→0
(2x + δx + 1)
= δlim
x→0
(4x + 2δx) = 2x + (0) + 1
= 2x + 1
= 4x + 2(0)
= 4x

20

F5 Mod A+ ADDM 2(18-49).indd 20 6/1/2021 10:59:22 AM


Exercise 5 Determine the first derivative of each of the following by using the first principle.
Tentukan terbitan pertama bagi setiap yang berikut dengan menggunakan prinsip pertama.
PL 2 Demonstrate the understanding of differentiation.
Example 4
Example 5 1 y = 5x

y = 3x2 y = 5x  1
y + δy = 5(x + δx)
Solution
y + δy = 5x + 5δx  2
y = 3x2  1
y + δy = 3(x + δx)2 2 − 1,
y + δy = 3[x2 + 2xδx + (δx)2]
δy = 5δx
y + δy = 3x2 + 6xδx + 3(δx)2  2
δy
2 − 1, δx = 5
δy = 6xδx + 3(δx)2 lim
δy
δx → 0 δ x
=5
δy
δx = 6x + 3δx dy
lim / ℎ𝑎𝑑 δy dx = 5
δx → 0 δx → 0 δx = 6x + 3(0)

dy
dx = 6x

Smart Tip
lim / ℎ𝑎𝑑 δy = dy
δx
δx → 0 δx → 0
dx

2 y = 4x2 3
3 y= x
y = 4x2  1
3
y + δy = 4(x + δx)2 y = x  1
y + δy = 4[x2 + 2xδx + (δx)2] 3
y + δy = 4x2 + 8xδx + 4(δx)2  2 y + δy = x + δx  2

2 − 1, 2 – 1,
δy = 8xδx + 4(δx)2 3 3
δy = x + δx − x
δy
δx = 8x + 4δx 3x − 3(x + δx)
δy = x(x + δx)
lim
δy
δx → 0 δ x
= 8x + 4(0)
–3δx
dy δy = x(x + δx)
dx = 8x
δy –3
δx = x(x + δx)
lim
δy –3
δx → 0 δ x
=
x[x + (0)]
dy 3
=– 2
dx x

21

F5 Mod A+ ADDM 2(18-49).indd 21 6/1/2021 10:59:23 AM


2.2 The First Derivative / Pembezaan Peringkat Pertama
dy ,
Exercise 6 Find dx or f (x) for each of the following.
dy ,
Cari dx atau f (x) bagi setiap yang berikut.
PL 2 Demonstrate the understanding of differentiation.

Example 6
1 y = 3x6
1 dy
(a) y = 4x3 (b) f(x) = 3x5 (c) y = x2
dx = 6(3x )
6–1

Solution = 18x5
(a) y = 4x3 (b) f(x) = 3x5 1
, (c) x2 = x–2
dy f (x) = 5(3x5−1)
dx = 3(4x2 )
3−1
= 15x4 dy
dx = –2x –3
–2 – 1
= 12x
= –2x
2
= – x3
2 y = –6x4
Smart Tip dy
dx = 4(–6x )
4–1

dy
1 If y = axn, then = naxn – 1, where a and n are constants. = –24x3
dx
dy
Jika y = axn, maka dx = naxn – 1, dengan keadaan a dan n ialah pemalar.
dy
2 If y = a, then = 0, where a is a constant.
dx
dy
Jika y = a, maka dx = 0, dengan keadaan a ialah pemalar.
dy 6
3 Notation of f ’(x) is equivalent to
’(x) when y = f(x).
f x).
f( 3 y= 7
dx
dy
Tatatanda f ’(x) adalah setara dengan
dx
f x).
f(
apabila y = f(x). dy
dy dx = 0
4 The derivative of y with respect to x is written as dx .
dy
Terbitan y terhadap x ditulis sebagai dx .

2 x6 4
4 y = x3 5 f(x) = 2 6 f(x) = – x2

x6–1 f(x) = –4x–2


y = 2x–3 f ′(x) = 6 2   f ′(x) = –2(–4x–2–1)
dy = 8x–3
= –3(2x–3–1) = 3x5
dx 8
= –6x–4 = x3
6
=– 4
x

5x3 1 18x–2
7 y= x 8 y = 3x2 9 f(x) = x

5x3 x–2 f(x) = 18x–2 – 1


y = x = 5x2 y= 3
= 18x–3
dy dy x–2 – 1 f ′(x) = (–3)(18x–3 – 1)
dx = 2(5x )  
2–1
dx = (–2) 3 = –54x–4
= 10x 54
–2x–3 = – x4
= 3

= – 23
3x

22

F5 Mod A+ ADDM 2(18-49).indd 22 6/1/2021 10:59:24 AM


dy ,
Exercise 7 Determine the value of dx or f (x) for each of the following when the value of x is given.
dy ,
Tentukan nilai dx atau f (x) bagi setiap yang berikut apabila nilai x diberi.
PL 2 Demonstrate the understanding of differentiation.
Example 6
Example 7
1 y = 2x4, x = –2
x3 dy
(a) y = 6 , x = 4 (b) f(x) = 3x4, x = −2
dx = 4(2x )
4–1

= 8x3

Solution
x3 x = –2,
(a) y = 6
(b) f(x) = 3x4 dy
dy x3–1 f ′(x) = 4(3x4−1) = 8(–2)3
dx
dx = 3 6   = 12x3 = –64
x2 f ′(−2) = 12(−2)3
= 2 = −96
dy (4)2
x = 4, dx = 2
=8 2 y = 2x7, x = 1
Calculator Corner dy
dx = 7(2x )
7–1
Determine the solution of Example 7 by using scientific calculator Casio
fx-570EX. = 14x6
Menentukan penyelesaian bagi Contoh 7 menggunakan kalkulator saintifik Casio fx-570EX.
Step 1/ Langkah 1: x = 1,
Press ‘MENU’ and find 1 : Calculate (normal display), press ‘ = ’. dy
Tekan ‘MENU’ dan cari 1 : Calculate (paparan normal), tekan ‘ = ’. = 14(1)6
dx
Step 2/ Langkah 2:
= 14

SHIFT ∫ 3 x x 4 (–) 2 =

3 f(x) = 3x2 , x = 5 x6 3
4 f(x) = – 3 , x = 2 5 f(x) = x2 , x = –1
f ′(x) = 2(3x2–1)
x6–1 f(x) = 3x–2
= 6x  
f ′(x) = –6 3
f ′(x) = –2(3x–2–1)
f ′(5) = 6(5) = –2x5 = –6x–3
= 30 6
f ′(2) = –2(2)5 =– 3
x
= –64
6
f ′(–1) = –
(–1)3
6
= – (–1)
=6

6 f(x) = 9x2 , x = –4 6 1
7 y = x3 , x = 3 8 y = 5x7 , x = –2
f ′(x) = 2(9x2–1)
y = 6x–3 x–7
= 18x y= 5
dy
f ′(–4) = 18(–4) = –3(6x–3–1) dy –7(x–7–1)
dx
= –72 = –18x–4 dx = 5
–18 –7x–8
= 4 =
x 5
–7
dy –18 = 8
= 5x
dx (3)4 dy –7
2 =
=– dx 5(–2)8
9
7
=–
1 280

23

F5 Mod A+ ADDM 2(18-49).indd 23 6/1/2021 10:59:24 AM


Smart Tip
Video
If y = p(x) ± q(x), then/ Jika y = p(x) ± q(x), maka
Scan or visit https://youtu.be/
dy d d
— = — [𝑝(𝑥)] ± — [𝑞(𝑥)] Jnqe2_jt8Ac to watch a video
dx dx dx on how to differentiate a
dy function.
— = 𝑝′(𝑥) ± 𝑞′(𝑥)
dx For educational purposes only

Exercise 8 Differentiate each of the following with respect to x.


Bezakan setiap yang berikut terhadap x.
PL 2 Demonstrate the understanding of differentiation.

Example 8
1 2x7 − 8x + 9 3 1
2 4x2 +
3x + 5x − 2
4
d 4x − 5x + 9
dx (2x − 8x + 9)
7
PAK-21
d 3 1
Solution = 7(2x7 – 1) – 1(8x1 – 1) dx 4x + 4x − 5x + 9 
2
VIDEO

d = 14x6 − 8
dx (3x + 5x − 2)
4
3x–1–1
= 2(4x2 – 1) + (–1)
4 −
5x1 – 1
= 4(3x4 – 1) + (1)(5x1 – 1 )
3x –2
= 12x3 + 5 = 8x − −5
4
3
= 8x − 4x2 − 5

2 x7 2x x4 − 3
3 x5 + 4 5
x 8 − 3 +9 x

d 5 d x7 2x d x4 − 3
dx (x + 2x )
–1

dx 8 − 3 + 9  dx  x 
= 5(x5 – 1) + (–1)(2x–1 – 1) x7– 1 2x1– 1 d 3
= 5x4 − 2x–2 =7
8  − 3
= dx (x − 3x–1)
2 7x6 2 = 3(x3 – 1) – (–1)(3x–1 – 1)
= 5x4 − 2 = −3
x 8 = 3x2 + 3x–2
3
= 3x2 + 2
x

Exercise 9 Find the gradient function for each of the following.


Cari fungsi kecerunan bagi setiap yang berikut.
PL 3 Apply the understanding of differentiation to perform simple tasks.

Example 9 1 y = x4 − 3x2 + 7x − 12 2 y = 2x(4 − x2)


y = (2x − 1)2
y = x4 − 3x2 + 7x − 12 y = 2x(4 − x2)
dy y = 8x − 2x3
Solution = 4x3 − 6x + 7
dx dy
y = (2x − 1)2 = 8 − 6x2
dx
= (2x − 1)(2x − 1) Expand
= 4x2 − 4x + 1 Kembangkan
dy
dx = 8x − 4

Smart Tip
dy
is also called gradient function.
dx
dy juga dikenali sebagai fungsi kecerunan.
dx

24

F5 Mod A+ ADDM 2(18-49).indd 24 6/1/2021 10:59:26 AM


3 y = (3x + 10)2 2(2x + 9)2 5 2
4 y= 3 5 
y = 3x −
x 
y = (3x + 10)2
= (3x + 10) (3x + 10) 2 5 2

= 9x2 + 60x + 100


y = 3 (4x2 + 36x + 81)
 x
y = 3x −
dy 8 5 5
= 3 x2 + 24x + 54 = 3x − x 3x − x 
dx = 18x + 60
dy 16 5 5 25
dx = 3 x + 24 = 9x − 3x x  − 3x x  + x
2
2

= 9x2 − 30 + 25x– 2
dy
dx = 18x − 50x
–3

50
= 18x − x3

3 2 5x4 + 6 3x4 − 5x
6 y= x− x  7 y=
x2
8 y=
x3 + 2x − 1

3 2 5x4 + 6 3x4 − 5x
y= x− x y=
x2
y=
x3 + 2x − 1
3 3 = 5x + 6x–2
2
= 3x − 5x + 2x − 1
–2
= x − x x − x 
dy = 5x − 5x–2 − 1
9 dx = 10x − 12x
–3
= x2 − 6 + x2 dy
dx = 5 + 10x
–3
12
= x2 − 6 + 9x–2 = 10x − x3 10
= 5 + x3
dy
dx = 2x − 18x
–3

18
= 2x − x3

Smart Tip
dy dy du
1 It is given that y = un, then = × , where u is a function of x (Chain rule).
dx du dx
dy dy du
Diberi bahawa y = un, maka dx = du × dx , dengan keadaan u ialah fungsi bagi x (Petua rantai).

dy dv du
2 It is given that y = uv, then = u  + v  , where u and v are functions of x (Product rule).
dx dx dx
dy dv du
Diberi bahawa y = uv, maka dx = u  dx + v  dx , dengan keadaan u dan v ialah fungsi bagi x (Petua hasil darab).

du dv
v— – u—
u dy dx dx
3 It is given that y = , then = , where u and v are functions of x (Quotient rule).
v dx v2
du dv
v— – u—
u dy dx dx
Diberi bahawa y = v , maka dx = , dengan keadaan u dan v ialah fungsi bagi x (Petua hasil bahagi).
v2

25

F5 Mod A+ ADDM 2(18-49).indd 25 6/1/2021 10:59:26 AM


dy dy du
= ×
dx du dx

Exercise 10 Differentiate each of the following by using chain rule.


Bezakan setiap yang berikut dengan menggunakan petua rantai.
PL 2 Demonstrate the understanding of differentiation.

Example 10
1 y = (3x − 1)4
y = (x2 + 4x)3
y = (3x − 1)4
dy d 2
= 3(x2 + 4x)2 (x + 4x) dy d
dx dx = 4(3x − 1)3 (3x − 1)
dx dx
= 3(x2 + 4x)2(2x + 4)
= 3(x2 + 4x)2 • 2(x + 2) = 4(3x − 1)3(3)
= 6(x2 + 4x)2(x + 2) = 12(3x − 1)3

Alternative Method
du
Let/ Katakan u = x2 + 4x ∴ dx = 2x + 4
dy
y = u3 ∴ du = 3u2
dy dy du
dx = du × dx
= 3u2 × (2x + 4) = 3(x2 + 4x)2 • 2(x + 2)
= 6(x2 + 4x)2(x + 2)

2 y = (3 − 2x)7 3 y = (4x2 + 7)9

y = (3 − 2x)7 y = (4x2 + 7)9


dy d dy d
= 7(3 − 2x)6 (3 − 2x) = 9(4x2 + 7)8 (4x2 + 7)
dx dx dx dx
= 7(3 − 2x)6(–2) = 9(4x2 + 7)8(8x)
= –14(3 − 2x)6 = 72x(4x2 + 7)8

3 6
4 y= 5 y=
2x + 1 (x − 5)4
y = 3(2x + 1)–1 y = 6(x − 5)–4
dy d dy d
= –1[3(2x + 1)–2] (2x + 1) = –4[6(x − 5)–5] (x − 5)
dx dx dx dx
dy
= –3(2x + 1)–2(2) = –24(x − 5)–5 ( 1)
dx
6 24
=– =–
(2x + 1)2 (x − 5)5

26

F5 Mod A+ ADDM 2(18-49).indd 26 6/1/2021 10:59:27 AM


Exercise 11 Differentiate each of the following by using product rule.
Bezakan setiap yang berikut dengan menggunakan petua hasil darab.
PL 2 Demonstrate the understanding of differentiation.

Example 11 1 y = 3x (4x2 – 1)
du
y = (2x + 1)(3x2 − 2) u = 3x, ∴ =3
dx
dv
Solution Smart Tip v = 4x2 − 1, ∴ = 8x
dx
du
u = 2x + 1, ∴ =2 Expand the equation
dx for checking.
dy dv du
=u +v
dv Kembangkan persamaan dx dx dx
v = 3x2 − 2, ∴ = 6x
dx untuk tujuan semakan. = (3x)(8x) + (4x2 − 1)(3)
dy dv du = 24x2 + 12x2 − 3
=u +v y = (2x + 1)(3x2 − 2)
dx dx dx = 36x2 − 3
= 6x3 − 4x + 3x2 − 2
= (2x + 1)(6x) + dy
dx = 18x − 4 + 6x
2
(3x2 − 2)(2)
= 12x2 + 6x + 6x2 − 4
= 18x2 + 6x − 4

2 y = 2x4(6x2 + 7) 3 y = (2x + 1)(x2 + 4x – 3)


du du
u = 2x4, ∴ = 8x3 u = 2x + 1, ∴ =2
dx dx
dv dv
v = 6x2 + 7, ∴ = 12x v = x2 + 4x − 3, ∴ = 2x + 4
dx dx

dy dv du dy dv du
=u +v =u +v
dx dx dx dx dx dx
= (2x )(12x) + (6x2 + 7)(8x3)
4
= (2x + 1)(2x + 4) + (x2 + 4x − 3)(2)
= 24x5 + 48x5 + 56x3 = 4x2 + 8x + 2x + 4 + 2x2 + 8x− 6
= 72x5 + 56x3 = 6x2 + 18x − 2

4 y = (4x − 5)(3x2 + 8) 5 y = (x – 1)(x2 – 2x + 5)


du du
u = 4x − 5, ∴ =4 u = x − 1, ∴ =1
dx dx
dv dv
v = 3x2 + 8, ∴ = 6x v = x2 − 2x + 5, ∴ = 2x − 2
dx dx

dy dv du dy dv du
=u +v =u +v
dx dx dx dx dx dx
= (4x − 5)(6x) + (3x2 + 8)(4) = (x − 1)( 2x − 2) + (x2 − 2x + 5)(1)
= 24x2 − 30x + 12x2 + 32 = 2x2 − 2x − 2x + 2 + x2 − 2x + 5
= 36x2 − 30x + 32 = 3x2 − 6x + 7

27

F5 Mod A+ ADDM 2(18-49).indd 27 6/1/2021 10:59:27 AM


Exercise 12 Differentiate each of the following by using quotient rule.
Bezakan setiap yang berikut dengan menggunakan petua hasil bahagi.
PL 2 Demonstrate the understanding of differentiation.

Example 12
6x
1 y=
3x + 2
3x − 4
y=
2x2 − 1 du
u = 6x, ∴ =6
Solution dx
du dv
u = 3x − 4, ∴ =3 v = 3x + 2, ∴ =3
dx dx
dv du dv
v = 2x2 − 1, ∴
dx
= 4x
dy v— —
dx – u dx
=
du dv dx v2
dy v— —
dx − u dx (3x + 2)(6) – (6x)(3)
= =
dx v2 (3x + 2)2
(2x − 1)(3) − (3x − 4)(4x)
2 18x + 12 – 18x
= =
(2x2 – 1)2 (3x + 2)2
6x2 − 3 − 12x2 + 16x 12
= = (3x + 2)2
(2x2 − 1)2
16x − 6x2 − 3
= (2x2 − 1)2

7 3x − 1
2 y= 3 y=
2x − 1 4x − 3

du du
u = 7, ∴ =0 u = 3x − 1, ∴ =3
dx dx
dv dv
v = 2x − 1, ∴ =2 v = 4x − 3, ∴ =4
dx dx
du dv du dv
dy v— —
dx − u dx dy v— —
dx − u dx
= =
dx v2 dx v2
(2x − 1)(0) − (7)(2) (4x − 3)(3) − (3x − 1)(4)
= =
(2x − 1)2 (4x − 3)2
14 12x − 9 − 12x + 4
=– =
(2x − 1)2 (4x − 3)2
5
= – (4x − 3)2

5 − 2x x2 − x − 1
4 y= 5 y=
6x − 1 2x + 3

du du
u = 5 − 2x, ∴ = –2 u = x2 − x − 1, ∴ = 2x − 1
dx dx
dv dv
v = 6x − 1, ∴ =6 v = 2x + 3, ∴ =2
dx dx
du dv du dv
dy v— —
dx − u dx dy v— —
dx − u dx
= =
dx v2 dx v2
(6x − 1)(–2) − (5 − 2x)(6) (2x + 3)(2x − 1) − (x2 − x − 1)(2)
= =
(6x – 1)2 (2x + 3)2
–12x + 2 − 30 + 12x 4x − 2x + 6x − 3 − 2x2 + 2x + 2
2
= =
(6x − 1)2 (2x + 3)2
28 2x + 6x − 1
2
= – (6x – 1)2 = (2x + 3)2

28

F5 Mod A+ ADDM 2(18-49).indd 28 6/1/2021 10:59:29 AM


2.3 The Second Derivative / Pembezaan Peringkat Kedua
d2y
Exercise 13 Find for each of the following.
dx2
d2y
Cari dx2 bagi setiap yang berikut.
PL 2 Demonstrate the understanding of differentiation.

Example 13 1 y = x2(4x − 8)

y = 7x2 − 2x3 + 5x + 9 y = x2(4x − 8)


Smart Tip
= 4x3 − 8x2
Solution d2y d dy dy
y = 7x2 − 2x3 + 5x + 9
 
dx2 is equal to dx dx . dx
= 12x2 − 16x
d2y
dy d2y
dx2 adalah sama dengan
dx
= 14x − 6x2 + 5
d dy dx2 = 24x − 16
d2y  
dx dx .
dx2 = 14 − 12x

y = (x2 − 5)2
y = x4 – 3
2
3
x
y = (x2 − 5)2
= x4 − 10x2 + 25 y = x4 − 3
dy x
= 4x3 − 20x = x4 − 3x–1
dx
d2y dy
= 4x3 + 3x–2
dx2 = 12x − 20 dx
2

d2y
dx2 = 12x − 6x
2 –3

= 12x2 − 63
x

Exercise 14 Find f ″(x) for each of the following.


Cari f ″(x) bagi setiap yang berikut.
PL 2 Demonstrate the understanding of differentiation.

Example 14 1
1 f(x) = x2 +
Smart Tip x
4 d
f(x) = (3x − 5)2 f ″ (x) is equal to dx [𝑓′(𝑥)].
1
f(x) = x2 +
x
d 1
Solution f “(x) adalah sama dengan dx 𝑓′(𝑥). f ′(x) = 2x − 2
x
4 2
f(x) = (3x − 5)2 = 4(3x − 5)–2 f ″(x) = 2 + 3
x
f ′(x) = –2[4(3x − 5)–3](3)
= –24(3x − 5)–3
f ″(x) = –3[–24(3x – 5)–4](3)
216
= (3x − 5)4

2 f(x) = (3x + 1)4 3


3 f(x) =
(2x − 7)2
f(x) = (3x + 1)4
f(x) = 3(2x − 7)–2
f ′(x) = 4(3x + 1)3(3)
f ′(x) = –2[3(2x − 7)–3](2)
= 12(3x + 1)3
= –12(2x − 7)–3
f ″(x) = 3[12(3x + 1)2](3)
= 108(3x + 1)2 f ″(x) = –3[–12(2x − 7)–4](2)
72
= (2x − 7)4

29

F5 Mod A+ ADDM 2(18-49).indd 29 6/1/2021 10:59:29 AM


2.4 Application of Differentiation/ Aplikasi Pembezaan

Smart Tip
y
l = Tangent 1 l is a tangent to the curve y = f(x) at point P.
Tangen l ialah tangen kepada lengkung y = f(x) pada titik P.
y = f(x)
2 The gradient of the tangent can be determined by
dy
substituting the value of x1 into .
dx
Kecerunan tangen boleh ditentukan dengan menggantikan nilai x1 ke dalam
P(x1, y1) dy
.
dx
x
0

Exercise 15 Calculate the gradient of the tangent to the curve at the points given.
Hitung kecerunan
y tangen kepada lengkung pada titik-titik yang diberi.
PL 3 Apply the understanding of differentiation to perform Tangent
simple tasks.
Tangen
y = f(x)
Example 15
1

Given y = x2 + 5.
m

N
or
1
m Diberi y = x2 + 5.
m al
Given y = −x2 + 6. 2

Diberi y = −x2 + 6. P(x1, y1)


Point
Titik
(–3, 14) (–2, 9) (–1, 6) (0, 5) (1, 6) (2, 9) (3, 14)
x
Point
Titik
(–3, –3)0 (–2, 2) (–1, 5) (0, 6) Gradient
Kecerunan
–6 –4 –2 0 2 4 6
Gradient
Kecerunan dy
Given y = x2 + 5, = 2x
dx
dy
Given/ Diberi y = −x2 + 6, = –2x At (–3, 14),
dx dy
At/ Pada (–3, –3), Gradient =
dx
dy = 2(–3)
Gradient/ Kecerunan =
dx
= –2(–3) = –6
At (–2, 9),
=6 dy
Gradient =
dx
At/ Pada (–2, 2), = 2(–2)
dy = –4
Gradient/ Kecerunan =
dx At (–1, 6),
= –2(–2) dy
=4 Gradient =
dx
= 2(–1)
At/ Pada (–1, 5), = –2
dy At (0, 5),
Gradient/ Kecerunan =
dx dy
= –2(–1) Gradient =
dx
=2 = 2(0)
=0
At/ Pada (0, 6), At (1, 6),
dy dy
Gradient/ Kecerunan = Gradient =
dx dx
= –2(0) = 2(1)
=0 =2
At (2, 9),
dy
Point Gradient =
(–3, –3) (–2, 2) (–1, 5) (0, 6) dx
Titik = 2(2)
Gradient =4
Kecerunan
6 4 2 0
At (3, 14),
dy
Gradient =
dx
= 2(3)
=6

30

F5 Mod A+ ADDM 2(18-49).indd 30 6/1/2021 10:59:30 AM


P(x1, y1)

x
0

Smart Tip
y 1 Normal line is a line at right angles (perpendicular) to the
Tangent tangent.
Tangen
y = f(x) Garis normal ialah garis pada sudut tegak (berserenjang) dengan tangen.
2 The normal line and the tangent line are perpendicular, hence

1
m
N
or m1m2 = –1.
m
m al
2 Garis normal dan garis tangen adalah berserenjang, maka m1m2 = –1.
P(x1, y1)
dy
3 is the gradient function of the tangent, m1, of the curve
x dx
0 f x) at point P. Therefore, the gradient function of the
f(
y = f(x)
–1
normal, m2, is dy .
Remember/ Ingat: dx
Equation of the tangent/Persamaan tangen
y − y1 = m1(x − x1) dy
ialah fungsi kecerunan tangen, m1, bagi lengkung y = f(x) pada titik P.
dx
Equation of the normal/Persamaan normal –1
y − y1 = m2(x − x1) Oleh itu, fungsi kecerunan normal, m2, ialah .
dy
dx

Exercise 16 Find the gradient of the tangent, m1, and the gradient of the normal, m2, at a given point.
Hence, find the equation of the tangent and normal of each of the following.
Cari kecerunan bagi tangen, m1, dan kecerunan bagi normal, m2, pada titik yang diberi. Seterusnya, cari
persamaan tangen dan normal bagi setiap yang berikut.
PL 3 Apply the understanding of differentiation to perform simple tasks.

Example 16

Gradient Equation of the tangent Equation of the normal


Kecerunan Persamaan tangen Persamaan normal

y = 2x2 − 5x + 6; (2, 4) m1 = 3 1
m2 = −
3
Solution y − y1 = m1(x − x1)
y − y1 = m2(x − x1)
y − 4 = 3(x − 2)
dy 1
= 4x − 5 y − 4 = 3x − 6 y − 4 = − (x − 2)
dx 3
= 4(2) − 5 y = 3x − 2
3(y − 4) = –(x − 2)
=3
1 3y − 12 = –x + 2
∴ m1 = 3, m2 = – 3y = –x + 14
3

Gradient Equation of the tangent Equation of the normal


Kecerunan Persamaan tangen Persamaan normal

1 y = 3x2 − 4x + 5; (1, 4) m1 = 2 1
m2 = –
dy 2
y − 4 = 2(x − 1)
dx = 6x − 4 y − 4 = 2x − 2 y−4=–
1
(x − 1)
= 6(1) − 4 2
y = 2x + 2 2(y − 4) = –(x − 1)
=2
1 2y − 8 = –x + 1
∴ m1 = 2, m2 = – 2y = –x + 9
2

31

F5 Mod A+ ADDM 2(18-49).indd 31 6/1/2021 10:59:31 AM


Gradient Equation of the tangent Equation of the normal
Kecerunan Persamaan tangen Persamaan normal

2 y = x2 − 2x + 5; (–4, 3) m1 = –10 1
m2 =
dy 10
y − 3 = –10[x − (–4)]
dx = 2x − 2 y − 3 = –10x − 40 y−3=
1
[x − (–4)]
= 2(–4) − 2 10
y = –10x − 37 10(y − 3) = x + 4
= –10
1 10y − 30 = x + 4
∴ m1 = –10, m2 =
10 10y = x + 34

3 y = x3 + x − 4; (1, –2) m1 = 4 1
m2 = – 
dy 4
y − (–2) = 4(x − 1)
dx = 3x + 1
2
1
y + 2 = 4x − 4 y – (–2) = –  (x − 1)
= 3(1)2 + 1 4
y = 4x − 6 4(y + 2) = –(x − 1)
=4
1 4y + 8 = –x + 1
∴ m1 = 4, m2 = –  4y = –x − 7
4

12 m1 = –3 1
4 y= ; (2, 3) m2 =
x 3
dy 12 y − 3 = –3(x − 2) 1
dx = – x2 y − 3 = –3x + 6 y−3= (x − 2)
3
12 y = –3x + 9 3(y − 3) = (x − 2)
= –  2
(2) 3y − 9 = x − 2
= –3 3y = x + 7
1
∴ m1 = –3, m2 =
3

5 y = (2x − 3)4; (2, 1) m1 = 8 1


m2 = –
dy 8
y − 1 = 8(x − 2)
dx = 4(2x − 3) (2)
3
1
y − 1 = 8x − 16 y − 1 = –  (x − 2)
= 8(2x − 3)3 8
y = 8x − 15 8(y − 1) = –(x − 2)
= 8[2(2) − 3]3
8y – 8 = –x + 2
=8
8y = –x + 10
1
∴ m1 = 8, m2 = –
8

32

F5 Mod A+ ADDM 2(18-49).indd 32 6/1/2021 10:59:31 AM


Exercise 17 Solve the following problems.
Selesaikan masalah yang berikut.
PL 3 Apply the understanding of differentiation to perform simple tasks.

Example 17 3 2
1 Given the equation of a curve is y = x – 22. Find
2
Given the equation of a curve is y = 7x – x 2. 3 2
Diberi persamaan lengkung ialah y =x – 22.Cari
Find 2
dy dy
Diberi persamaan lengkung ialah y = 7x – x2. (a) at point P(4, 2),/ pada titik P(4, 2),
dx dx
Cari
dy (b) the equation of the tangent to the curve at
(a) at point P(2, 10),/ dy pada titik P(2, 10), point P,
dx dx
persamaan tangen kepada lengkung pada titik P,
(b) the equation of the tangent to the curve at (c) the equation of the normal to the curve at
point P, point P.
persamaan tangen kepada lengkung pada titik P, persamaan normal kepada lengkung pada titik P.
(c) the equation of the normal to the curve at
point P. 3
persamaan normal kepada lengkung pada titik P. (a) y = 2 x2 – 22
dy
= 3x
Solution dx
dy
(a) y = 7x – x 2 When x = 4, = 3(4)
dx
dy = 12
= 7 – 2x
dx
dy
When/Apabila x = 2, = 7 – 2(2) (b) Gradient of the tangent = 12
dx
=3 Equation of the tangent,
y – 2 = 12(x – 4)
(b) Gradient of the tangent/ Kecerunan tangen = 3
Equation of the tangent/ Persamaan tangen, y – 2 = 12x – 48
y – 10 = 3(x – 2) y = 12x – 46
y – 10 = 3x – 6
y = 3x + 4 1
(c) Gradient of the normal = –
1 12
(c) Gradient of the normal/ Kecerunan normal = – Equation of the normal,
3
Equation of the normal/ Persamaan normal, 1
y – 2 = – 12 (x – 4)
1
y – 10 = – (x – 2) 12y – 24 = –x + 4
3
3y – 30 = –x + 2 12y = –x + 28
3y = –x + 32

2 Given the equation of a curve is y = (2x – 3)3. Find


Diberi persamaan lengkung ialah y = (2x – 3)3. Cari
dy
(a) at point P(2, 1),/ dy pada titik P(2, 1),
dx dx
(b) the equation of the tangent to the curve at point P,
persamaan tangen kepada lengkung pada titik P,
(c) the equation of the normal to the curve at point P.
persamaan normal kepada lengkung pada titik P.

(a) y = (2x – 3)3 (b) Gradient of the tangent = 6


dy Equation of the tangent,
= 3(2x – 3)2 × 2
dx y – 1 = 6(x – 2)
= 6(2x – 3)2 y – 1 = 6x – 12
dy y = 6x – 11
When x = 2, = 6[2(2) – 3]2
dx
=6
1
(c) Gradient of the normal = –
6
Equation of the normal,
1
y – 1 = – (x – 2)
6
6y – 6 = –x + 2
6y = –x + 8

33

F5 Mod A+ ADDM 2(18-49).indd 33 6/1/2021 10:59:32 AM


6
3 Given the equation of a curve is y = + 8. Find
x
6
Diberi persamaan lengkung ialah y =+ 8. Cari
x
dy
(a) at point P(–2, 5),/ dy pada titik P(–2, 5),
dx dx
(b) the equation of the tangent to the curve at point P,
persamaan tangen kepada lengkung pada titik P,
(c) the equation of the normal to the curve at point P,
persamaan normal kepada lengkung pada titik P.

6
(a) y= x +8 (c) Gradient of the normal,
1 2
dy 6 =– =
dx
=– 2
x
dy 6
– ( )
3
2
3

When x = –2, =–
dx (–2)2
6 Equation of the normal,
=– 2
4 y – 5 = [x – (–2)]
3 3
=– 3(y – 5) = 2(x + 2)
2
3y – 15 = 2x + 4
3 3y = 2x + 19
(b) Gradient of the tangent = –
2
Equation of the tangent,
PAK-21
3
y – 5 = – [x – (–2)]
ACTIVITY

2
2(y – 5) = –3(x + 2)
2y – 10 = –3x – 6
2y = –3x + 4

PAK-21 Gallery Walk

Steps/ Langkah-langkah:
1 Teacher provides a set of questions involving tangent and normal to the curve on coloured cards.
Guru menyediakan beberapa set soalan melibatkan tangen dan normal kepada lengkung pada kad berwarna.

2 Students perform this activity in groups of three students. A coloured card is randomly selected for each group.
Murid melakukan aktiviti ini secara berkumpulan yang terdiri daripada tiga orang murid. Satu kad berwarna dipilih secara rawak
bagi setiap kumpulan.

3 Each group is required to answer all questions on the selected card. Write each answer on a mahjung paper.
Setiap kumpulan dikehendaki menjawab semua soalan yang terdapat pada kad yang dipilih. Tulis setiap jawapan pada kertas
mahjung.

4 The group work of each group is posted on the class’s notice board. Students are required to stand next to their
group work.
Hasil kerja setiap kumpulan ditampal pada papan kenyataan kelas. Murid-murid dikehendaki berdiri di sebelah hasil kerja
masing-masing.
5 A group is required to move to each group to evaluate the work of other groups. Once completed, other groups
need to do the same.
Satu kumpulan dikehendaki bergerak ke setiap kumpulan bagi menilai hasil kerja kumpulan yang lain. Setelah selesai, kumpulan
lain perlu melakukan langkah yang sama.

6 Teacher holds a discussion with students to enhance their understanding about solving problems involving tangent
and normal.
Guru mengadakan perbincangan dengan murid untuk menambahkan kefahaman mereka mengenai penyelesaian masalah
melibatkan tangen dan normal.

34

F5 Mod A+ ADDM 2(18-49).indd 34 6/1/2021 10:59:32 AM


Smart Tip
Turning points/ Titik pusingan The tangents to the curve y = f(x) at its turning points P and Q are parallel to
dy
the x-axis, therefore = 0.
dx
Maximum point
Titik maksimum y Tangen bagi lengkung y = f(x) pada titik pusingan P dan Q adalah selari dengan paksi-x,
dy
dy maka = 0.
=0 P dx
dx
To determine the coordinates of the turning point:
Menentukan koordinat titik pusingan:
x dy
0 1 Find/Cari .
dx
dy
=0 dy
Q dx 2 Solve = 0 to obtain the coordinates of the turning points.
dx
Minimum point dy
Selesaikan = 0 untuk memperoleh koordinat titik pusingan.
Titik minimum dx

y
Remember/ Ingat:
dy
=0
dx The maximum and minimum points are the highest and the
lowest points at certain region of the curve respectively.
dy Titik maksimum dan titik minimum masing-masing ialah titik tertinggi
dy dy >0
>0 <0 dx dan terendah kawasan tertentu bagi lengkung itu.
dx dx
dy
=0
dx
x
0

Two methods to determine the maximum and minimum points:


Dua kaedah untuk menentukan titik maksimum dan titik minimum:

d2y d2y
1 Second derivative method, /Kaedah terbitan kedua, dx2 .
dx2
d2y
(a) For maximum point, the value of 2 is negative.
dx
d2y
Bagi titik maksimum, nilai dx2 ialah negatif.

d2y
(b) For minimum point, the value of is positive.
dx2
d2y
Bagi titik minimum, nilai dx2 ialah positif.

2 Tabulating and sketching the curve/Penjadualan dan lakaran lengkung.


dy
(a) For maximum point, the value of changes from positive to negative through 0 as x increases from left to right.
dx
dy
Bagi titik maksimum, nilai berubah dari positif ke negatif melalui 0 apabila x bertambah dari kiri ke kanan.
dx
dy
(b) For minimum point, the value of changes from negative to positive through 0 as x increases from left to right.
dx
dy
Bagi titik minimum, nilai berubah dari negatif ke positif melalui 0 apabila x bertambah dari kiri ke kanan.
dx

Remember/ Ingat:
dy
Turning point/Titik pusingan: = 0.
dx
dy d2y
At maximum point/Pada titik maksimum: = 0, 2 < 0.
dx dx

dy d2y
At minimum point/Pada titik minimum: = 0, 2 > 0.
dx dx

35

F5 Mod A+ ADDM 2(18-49).indd 35 6/1/2021 10:59:33 AM


Exercise 18 Determine the turning point for each of the following.
Tentukan titik pusingan bagi setiap yang berikut.
PL 3 Apply the understanding of differentiation to perform simple tasks.

Example 18

y = 2x3 − 6x2 − 18x + 9 Alternative Method

Solution x = –1, y = 2(–1)3 – 6(–1)2 – 18(–1) + 9


y = 2x3 − 6x2 − 18x + 9 = 19
x = 3, y = 2(3)3 – 6(3)2 – 18(3) + 9
dy = –45
= 6x2 − 12x − 18
dx
Tabulating and curve sketching
At turning points, Penjadualan dan lakaran lengkung
Pada titik pusingan, dy
dy dx = 6x – 12x – 18
2

= 0, 6x2 − 12x − 18 = 0
dx x = –1,
PAK-21 ÷6, x2 − 2x − 3 = 0
(x + 1)(x – 3) = 0 x = –2 x=0
VIDEO

x = –1
x + 1 = 0 or/atau x − 3 = 0 (x < –1) (x > –1)
x = –1 x=3
x = –1, y = 2(–1)3 − 6(–1)2 − 18(–1) + 9 y = 6(–2)2 – 12(–2) – 18 y = 6(0)2 – 12(0) – 18
0
= 30 = –18
= 19
x = 3, y = 2(3)3 − 6(3)2 − 18(3) + 9
+ 0 –
= –45

Using second derivative method,


Menggunakan kaedah terbitan kedua,
d2y
dx2 = 12x − 12 Curve/Lengkung:
d2y Maximum point/Titik maksimum: (–1, 19)
x = –1, dx2 = 12(–1) − 12 = –24 < 0
x = 3,
y has a maximum value at x = –1
y mempunyai nilai maksimum di x = –1
x=2 x=4
d2y x=3
(x < 3) (x > 3)
x = 3, dx2 = 12(3) − 12 = 24 > 0
y has a minimum value at x = 3 y = 6(2)2 – 12(2) – 18 y = (4)2 – 12(4) – 18
y mempunyai nilai minimum di x = 3 0
= –18 = 30

Maximum point/Titik maksimum: (–1, 19) – 0 +


Minimum point/Titik minimum: (3, –45)

Video
Curve/Lengkung:
Scan or visit https://youtu.be/cXxW6mHhVXY Minimum point/Titik minimum: (3, −45)
to watch a video on how to find maximum
and minimum turning points.
For educational purposes only

1 y = 6x − x2 + 7

y = 6x − x2 + 7 d2y
dx2 = –2 < 0
dy
= 6 − 2x y has a maximum value at x = 3
dx
dy
=0 y = 6(3) − (3)2 + 7
dx
= 16
6 − 2x = 0
–2x = –6 The turning point is a maximum point (3, 16)
x=3

36

F5 Mod A+ ADDM 2(18-49).indd 36 6/1/2021 10:59:34 AM


1 3 8
2 y= x − x2 − 15x + 6 3 y = 2x +
3 x

1 8
y = 3 x3 − x2 − 15x + 6 y = 2x +
x
dy dy 8
= x2 − 2x − 15 =2− 2
dx dx x
At turning points, At turning points,
dy dy
=0 =0
dx dx
8
x2 − 2x − 15 = 0 2− 2 =0
x
(x + 3)(x − 5) = 0
2x2 − 8 = 0
x + 3 = 0 or x − 5 = 0
x2 − 4 = 0
x = –3 x=5
(x + 2)(x − 2) = 0
dy2
x = –2 or x = 2
dx2 = 2x − 2
d2y d2y 16
x = –3, dx2 = 2(–3) − 2 dx2 = x3
= –8 < 0 d2y 16
x = –2, dx2 =
y has a maximum value at x = –3 (–2)3
1 = –2 < 0
y = 3 (–3)3 − (–3)2 − 15(–3) + 6 y has a maximum value at x = –2
= 33 8
y = 2(–2) +
d2y (–2)
x = 5, dx2 = 2(5) − 2 = –8
= 8>0 d2y 16
x = 2, dx2 = 3
y has a minimum value at x = 5 (2)
1 = 2>0
y = 3 (5)3 − (5)2 − 15(5) + 6 y has a minimum value at x = 2
1
= –52 8
3 y = 2(2) +
(2)
Maximum point: (–3, 33) =8
1
Minimum point: (5, – 52 3 ) Maximum point: (–2, –8)
Minimum point: (2, 8)

1
4 y=5– − 4x
x

y = 5 − x–1 − 4x d2y 1 d2y –2


dx2 = –2x
–3 When x = – , 2 = = 16 > 0
2 dx 1
 
3
dy –2 –
= x–2 − 4 = 3 2
dx x
1 1
= 2 −4 1 d2y –2 y has a minimum point at x = –
x When x = 2 , dx2 = = –16 < 0 2
1 3 1 1
2  y=5−
1
 
–4 –
2
At turning points,
1  

2
dy y has a maximum point at x = =9
=0 2
dx 1 1
1
y=5−
1
−4 
2 Maximum point:  12 , 1
x2
−4=0  
2
=1 1
1 − 4x2 = 0 
Minimum point: – , 9
2 
4x − 1 = 0
2

(2x − 1)(2x + 1) = 0
1 1
x = 2 or x = –
2

37

F5 Mod A+ ADDM 2(18-49).indd 37 6/1/2021 10:59:34 AM


Exercise 19 Solve each of the following. Evaluating

Selesaikan setiap yang berikut.


PL 5 Apply appropriate knowledge and skills of differentiation in the context of complex routine problem solving.

Example 19
1 A closed box has a square base of side x cm and
height y cm. Given the volume of the box is 216 cm3.
A closed cylinder with base radius r cm and height Show that its total surface area, A, is
of h cm has a volume of 27 cm3. Show that the total Sebuah kotak tertutup mempunyai tapak segi empat sama
surface area, A, is bersisi x cm dan tinggi y cm. Diberi isi padu kotak itu ialah
Sebuah silinder tertutup dengan jejari tapak r cm dan tinggi 216 cm3. Tunjukkan bahawa jumlah luas permukaan, A, ialah
h cm mempunyai isi padu 27 cm3. Tunjukkan bahawa jumlah
864
luas permukaan, A, ialah A = 2x2 +
x
54
A= 2πr2
r + Hence, find the value of x when A is minimum.
Seterusnya, cari nilai x apabila A adalah minimum.
Hence, find the value of r when A is minimum.
Seterusnya, cari nilai r apabila A adalah minimum. Volume = length (x) × length (x) × height (y)
216 = x2y
Solution
216
Volume/Isi padu = 27 cm3 y= 2
x
πr2h = 27
27 Total surface area:
h= 2 A = (x)(x) + (x)(x) + (x)(y) + (x)(y)
πr y
+ (x)(y) + (x)(y)
Total surface area:/Jumlah luas permukaan: = 2x2 + 4xy
A = 2πrh + πr2 + πr2 216 x
27 = 2x2 + 4x
x2   x
= (2πr)
πr2 
+ 2πr2
864
A = 2x2 +
54πr x (shown)
= + 2πr2
πr2
dA 864
= 4x − 2
54 dx x
A= + 2πr2 (shown)
r 54 dA
r = 54r
–1
= 0,
dx
dA 54 dA 864
= – 2 + 4πr dr = (–1) 54r
–1–1
4x − 2 = 0
dr r x
= –54r –2
dA 4x3 − 864 = 0
= 0, 54
dr =– 2 864
r
–54 + 4πr = 0 x3 =
4
r2 = 216
–54 + 4πr3 = 0
27 x = 3 216
r3 = x =6

dA
2
1 728
27 =4+
r = 3� dx2 x3

1 728
3 =4+
r= (6)3

3
= 12 (> 0)
d2A 108
= 3 + 4π A has a minimum value at x = 6.
dr2 r
108
= + 4π
3 3
3 
2π 
108
= + 4π
27
 

= 12π (> 0)
3 .
A has a minimum value at r =

3

3 .
A mempunyai nilai minimum pada r =

3

38

F5 Mod A+ ADDM 2(18-49).indd 38 6/1/2021 10:59:36 AM


2 The diagram below shows a closed cylinder with 3 The diagram below shows a circle with radius
base radius r cm and height of h cm. The volume of 12 cm. A cylinder with height h cm is inscribed in
the cylinder is 64 cm3. the circle.
Rajah di bawah menunjukkan sebuah silinder tertutup dengan Rajah di bawah menunjukkan sebuah bulatan berjejari 12 cm.
jejari tapak r cm dan tinggi h cm. Isi padu silinder itu ialah Sebuah silinder dengan tinggi h cm terterap di dalam bulatan
64 cm3. itu.

r cm

h cm h cm

r cm

Find the height of the cylinder when its volume is


(a) Show that its total surface area, A, is maximum.
Tunjukkan jumlah luas permukaan, A, ialah Cari tinggi silinder apabila isi padunya adalah maksimum.

128
A= 2πr2
r +
(b) Hence, find the value of r when A is minimum. h cm C
Seterusnya, cari nilai r apabila A adalah minimum. 1 12 cm
2h
r cm
(a) Volume:
πr2h = 64

h =
64 12 h2
+ r2 = 122
πr2
h2 + r2 = 144
Total surface area: 4
A = 2πrh + πr2 + πr2 r2 = 144 − h
2

64 4
= (2πr) 2 + 2πr2
πr   Vcylinder = πr2h
128πr
= πh 144 − h
2
=
πr2
+ 2πr2 
4 
128 h 3
A=
r
+ 2πr2 (shown) 
= π 144h −
4 
dV 3h 2

(b)
dA –128
= 2 + 4πr dh 
= π 144 −
4 
dr r dV
dA =0
= 0, dh
dr 3h2 = 0
–128
+ 4πr = 0

π 144 −
4 
r2 3h2 = 144
–128 + 4πr3 = 0 4
32 h2 = 144 × 4
r3 = 3
π
h2 = 192
r = 3� 32   h = √192 = 8 3 cm
π

d A 256
2
d2V
dh2
= π 0 – 3h
2[ ]
= 3 + 4π
dr2 r 3h
=– π
256 2
= 3 + 4π
32
3 π
 =–
3(8 3 )
2
π
256
= + 4π = 12π (. 0) = –12 3 π (, 0)
32
 
π Volume V is maximum when h = 8 3 cm.
A has a minimum value at r = 3 �   32
π

39

F5 Mod A+ ADDM 2(18-49).indd 39 6/1/2021 10:59:36 AM


Exercise 20 Solve each of the following problems. Applying

Selesaikan setiap masalah yang berikut.


PL 5 Apply appropriate knowledge and skills of differentiation in the context of complex routine problem solving.

Example 20
1 Given y = x3 – 4x and the value of x changes at a rate
of 0.6 unit s–1. Find the rate of change in y when
The volume of a sphere decreases at a rate of x = 3.
5.4π cm3 s–1. Find the rate of change in the radius of Diberi y = x3 − 4x dan nilai bagi x berubah pada kadar
the sphere when the radius is 6 cm. 0.6 unit s−1. Cari kadar perubahan y apabila x = 3.
Isi padu sebuah sfera menyusut pada kadar 5.4π cm3 s−1. Cari
kadar perubahan jejari sfera itu apabila jejari ialah 6 cm.
dx
= 0.6
Solution dt
Common Error y = x3 − 4x
dV
= –5.4π WRONG CORRECT dy
dt SALAH BETUL = 3x2 − 4
4 dx
V = πr3 dV dV
3 = 5.4π, = –5.4π x = 3,
dt dt
dV dy
= 4πr2 The negative sign of −5.4π = 3(3)2 − 4 = 23
dr dx
dV means the decreasing of volume
r = 6, = 4π(6)2 of the sphere. dy dy dx
dr = ×
= 144π Tanda negatif pada −5.4π bermaksud dt dx dt
penyusutan isi padu sfera.
= (23) × (0.6)
dV dV dr
= × = 13.8 unit s–1
dt dr dt
dr Chain rule/Petua rantai
–5.4π = (144π) ×
dt dV dV dr
dr dt = dr × dt
= –0.0375 cm s–1
dt V = Volume/Isi padu
t = Time/Masa
r = Radius/Jejari

2 The radius of a circle increases at a rate of 0.3 cm s–1. 3 The diagram shows a square.
Find the rate of change of the area of the circle when Rajah menunjukkan sebuah segi empat sama.
the radius is 4 cm. Give the answer in terms of π.
Jejari sebuah bulatan menokok pada kadar 0.3 cm s−1. Cari kadar
perubahan luas bulatan apabila jejari ialah 4 cm. Beri jawapan
2x cm
dalam sebutan π.

dr
= 0.3 The area of the square decreases at a rate of
dt
A = πr2 3.2 cm2 s–1. Calculate the rate of change of the length
dA of the square when x = 5 cm.
= 2πr Luas segi empat sama itu menyusut pada kadar 3.2 cm2 s–1.
dr
Hitung kadar perubahan panjang sisi segi empat sama itu
r = 4, apabila x = 5 cm.
dA
= 2π(4) = 8π Area, A = 2x × 2x
dr
= 4x2
dA dA dr
= × dA
dt dr dt = 8x
dx
= (8π) × (0.3)
= 2.4π cm2 s–1 dA
x = 5, = 8(5) = 40
dx
dA
Given = 3.2
dt
dA dA dx
= ×
dt dx dt
dx
3.2 = 40 ×
dt
dx
= 0.08 cm s–1
dt

40

F5 Mod A+ ADDM 2(18-49).indd 40 6/1/2021 10:59:37 AM


Exercise 21 Solve each of the following problems. Applying

Selesaikan setiap masalah yang berikut.


PL 5 Apply appropriate knowledge and skills of differentiation in the context of complex routine problem solving.

Example 21 1 The diagram shows the water is poured into a


hemispherical container.
The diagram below shows a conical container with a Rajah menunjukkan air dimasukkan ke dalam sebuah bekas
radius of 3 cm and a height of 12 cm. berbentuk hemisfera.
Rajah di bawah menunjukkan sebuah bekas berbentuk kon yang
berjejari 3 cm dan tinggi 12 cm. 26 cm

3 cm

L cm2

12 cm
h cm
h cm
(a) Express the area of water surface, L cm2,
in terms of h.
(a) Express the volume of water, V cm3, in terms Ungkapkan luas permukaan air, L cm2, dalam sebutan h.
of h. (b) It is given that the rate of change of the height
Ungkapkan isi padu air, V cm3, dalam sebutan h. of water is 0.5 cm s−1. Find the rate of
(b) It is given that the water flows out from the hole change of the area of water surface when
of the container at a constant rate of 1.8 cm3 s−1. h = 6 cm.
Find the rate of change of the height of the water Diberi bahawa kadar perubahan tinggi air ialah
level when h = 3.6 cm. 0.5 cm s−1. Cari kadar perubahan luas permukaan air
Diberi bahawa air mengalir keluar melalui lubang bekas apabila h = 6 cm.
itu pada kadar tetap 1.8 cm3 s−1. Cari kadar perubahan
ketinggian paras air apabila h = 3.6 cm. (a) Radius of hemisphere = 13 cm
Let r = radius of the water surface
Solution
26 cm
(b) Let r = radius of the water surface
Katakan r = jejari permukaan air
r 3
=
h 12 13 − h 13
h
r= r
4
1 h cm
V = × π × r2 × h
3
1 h 2 By using Pythagoras’ theorem,
= ×π×
3 4
×h( ) (13 – h)2 + r 2 = 132
1 3 132 – 26h + h2 + r 2 = 132
= πh
48 r 2 = 26h − h2
L = πr2
dV 1 2 = π(26h − h2)
(b) = πh
dh 16 = 26πh – πh2
When/Apabila h = 3.6, Smart Tip
dV 1 dL
= π(3.6)2 (b) = 26π – 2πh
dh 16 • If V decreases, then dh
dV dL
= 0.81 π is negative. When h = 6, = 26π – 2π(6) = 14π
dt dh
dV dh
Given/Diberi = –1.8 Jika V menyusut, maka Given = 0.5
dt dV dt
dV dV dh adalah negatif.
= × dt dL dL dh
dt dh dt = ×
• If V increases, then dt dh dt
dh dV dL
–1.8 = 0.81π × is positive. = (14π) × (0.5)
dt dt dt
dh –1.8 Jika V menokok, maka = 21.99 cm2 s–1
= dV
dt 0.81π adalah positif.
dt
= –0.7074 cm s–1

41

F5 Mod A+ ADDM 2(18-49).indd 41 6/1/2021 10:59:38 AM


2 The diagram below shows the water is poured into a 3 The air from a spherical balloon is released at a
conical container with a radius of 6 cm and a height constant rate of 5π cm3 s−1. Find the rate of change of
of 18 cm. the radius of the balloon when the radius is 10 cm.


Rajah di bawah menunjukkan air dimasukkan ke dalam sebuah
bekas berbentuk kon yang berjejari 6 cm dan tinggi 18 cm.
[ 4
Volume of sphere = 3  πr 3 ]
Udara dari sebuah belon berbentuk sfera dilepaskan pada kadar
6 cm tetap 5π cm3 s−1. Cari kadar perubahan jejari belon apabila
jejari ialah 10 cm.
[ 4
]
Isi padu sfera =  πr3
3
18 cm
h cm Let V = volume of the balloon
4
V = 3  πr 3
dV
= 4πr 2
(a) Express the volume of water, V cm3, in terms dr
of h. When r = 10
Ungkapkan isi padu air, V cm3, dalam sebutan h. dV
(b) It is given that the rate of change of the height = 4π(10)2
dr
of water level is 0.72 cm s−1. Find the rate = 400π
of change of the volume of water when h = 4 cm. dV
Diberi bahawa kadar perubahan ketinggian paras air Given = –5π
dt
ialah 0.72 cm s−1. Cari kadar perubahan isi padu air dV dV dr
apabila h = 4 cm. = ×
dt dr dt
dr
–5π = 400π ×
(a) Let r = radius of the water surface dt
r 6 dr 5π
= –
h   = 18 dt 400π
h dr
r =   = –0.0125 cm s–1
3 dt
1
V = × π × r2 × h
3

1
= 3 × π × 3 × h
h 2
()
1 3
=
27 πh

dV 1
(b) = 9  πh2
dh
dV 1
When h = 4, =  π(4)2
dh 9
16
= 9π
dh
Given = 0.72
dt
dV dV dh
= ×
dt dh dt
dV 16
= 9 π × 0.72
dt
= 4.021 cm3 s–1

42

F5 Mod A+ ADDM 2(18-49).indd 42 6/1/2021 10:59:39 AM


Exercise 22 Solve each of the following problems.
Selesaikan setiap masalah yang berikut.
PL 3 Apply the understanding of differentiation to perform simple tasks.

Example 22 dy
1 Given y = x3, find the value of when x = 5.
dy dy dx
Given y = x , find the value of when x = 81.
4

dx Diberi y = x3, cari nilai apabila x = 5.


dy dx
Diberi y = 4 x , cari nilai apabila x = 81. Hence, find the approximate value of
dx
Seterusnya, cari nilai hampir bagi
Hence, find the approximate value of (a) 5.053, (b) 4.953.
Seterusnya, cari nilai hampir bagi
(a) dy
81.04 ,
4
y = x3, = 3x2
dx
(b) 4
80.96 .
When x = 5, y = (5)3 = 125
dy
Solution = 3(5)2 = 75
1 dx
y = x4 (a) x = 5, x = 5.05 – 5 = 0.05
dy 1 – 3 x3 = y
= x4
dx 4 1 (x + x)3 = y + y
When/Apabila x = 81, y = 81 4 = 3 dy
(5 + 0.05)3 = (125) + × x
dy 1 3 dx
= (81)– 4 (5.05) = 125 + (75) × 0.05
3
dx 4

( )
1 1 = 125 + 3.75
= 4 27 = 128.75
1
= 108 (b) x = 5, x = 4.95 – 5 = –0.05
x3 = y
(a) 4
81.04 (x + x)3 = y + y
dy
x = 81, x = 81.04 – 81 = 0.4 [5 + (–0.05)]3 = 125 + × x
dx
x =y
4
4.95 = 125 + (75) × (–0.05)
3

x + x = y + y
4
= 125 – 3.75 = 121.25
dy
81 + 0.04 = (3) + × x
4

dx
dy
4
81.04 = (3) +
1
( )
108 × 0.04
2 Given y = x , find the value of
Diberi y =
dy
x , cari nilai
dx
when x = 16.
apabila x = 16.
= (3) + 0.00037 dx
Hence, find the approximate value of
= 3.00037
Seterusnya, cari nilai hampir bagi
(a) 16.2 , (b) 15.8 .
(b) 4
80.96
x = 81, x = 80.96 – 81 = –0.4
dy 1 – 1
1
4
x =y y = x2,
= x2
dx 1 2
x + x = y + y
4
When x = 16, y = 16 2 = 4
dy
4
81 + (–0.04) = (3) +
dx
× x dy 1 1 1 1
= (16)– 2 = 2 4 = 8
dx 2 ( )
1

4 1
( )
80.96 = (3) + 108 × (–0.04) (a) x = 16, x = 16.2 – 16 = 0.2
= (3) – 0.00037 x =y
= 2.9996 x + x = y + y
dy
16 + 0.2 = (4) + × x
dx
Smart Tip
δy dy
16.2 = (4) +
1
8 × 0.2()
δx dx = (4) + 0.025 = 4.025
δy dy × δx (b) x = 16, x = 15.8 – 16 = –0.2
dx x =y
x + x = y + y
dy
16 + (–0.2) = (4) + × x
dx
15.8 = (4) +
1
()
8 × (–0.2)
= (4) – 0.025 = 3.975

43

F5 Mod A+ ADDM 2(18-49).indd 43 6/1/2021 10:59:40 AM


dy
3 Given y = x , find the value of when x = 64.
3

dy dx
Diberi y = 3 x , cari nilai apabila x = 64.
dx
Hence, find the approximate value of
Seterusnya, cari nilai hampir bagi
(a) 64.3 , (b) 63.7 .
3 3

dy 1 – 2
1
y = x3,
= x3 (b) x = 64, x = 63.7 – 64 = –0.3
dx 1 3
x =y
3
When x = 64, y = 64 3 = 4

( )
x + x = y + y
3
dy 1 2 1 1 1
= (64)– 3 = 3 16 = 48 dy
dx 3 3
64 + (–0.3) = (4) + × x
dx
(a) x = 64, x = 64.3 – 64 = 0.3
3
x =y
3
63.7 = (4) + ( )
1
48 × (–0.3)
3
x + x = y + y = (4) – 0.00625
dy = 3.994
64 + 0.3 = (4) +
3
× x
dx
3
64.3 = (4) +
1
48 × 0.3 ( )
= (4) + 0.00625
= 4.006

Exercise 23 Solve each of the following problems. Applying

Selesaikan setiap masalah yang berikut.


PL 5 Apply appropriate knowledge and skills of differentiation in the context of complex routine problem solving.

Example 23 1 Given y = 2x3 + 5x2 + 1, find the small corresponding


change in y when x decreases from 5 to 4.98. Hence,
Given y = x3, find the small corresponding change
find the approximate value of y after the change in x
in y when x changes from 4 to 4.003. Hence, find
has taken place.
the approximate value of y after the change in x Diberi y = 2x3 + 5x2 + 1, cari perubahan kecil dalam y apabila x
has taken place. menyusut daripada 5 kepada 4.98. Seterusnya, cari nilai hampir
Diberi y = x3, cari perubahan kecil dalam y apabila x berubah bagi y selepas perubahan dalam x berlaku.
daripada 4 kepada 4.003. Seterusnya, cari nilai hampir bagi y
selepas perubahan dalam x berlaku.
y = 2x3 + 5x2 + 1
Solution dy
= 6x2 + 10x
y = x3 dx
dy
dy x = 5, = 6(5)2 + 10(5) = 200
= 3x2 dx
dx δx = 4.98 − 5
dy
x = 4, = 3(4)2 = 48 = –0.02
dx
dy
δx = 4.003 − 4 δy ≈ × δx
dx
= 0.003 = 200(–0.02)
dy = –4
δy ≈ × δx
dx
= (48)(0.003) Approximate value of y is
= 0.144 y + δy
= [2(5)3 + 5(5)2 + 1] + (–4)
Approximate value of y is = 376 − 4
Nilai hampir bagi y ialah Check/Semak = 372
y + δy (4.003)3
= x3 + 0.144 = 64.14410803
= (4)3 + 0.144 ≈ 64.144
= 64.144

44

F5 Mod A+ ADDM 2(18-49).indd 44 6/1/2021 10:59:41 AM


6
2 Given y = , find the small corresponding 3 Given y = – 12 + 6, find the small corresponding
2x − 1 x
change in y when x increases from 2 to 2.03. Hence, change in x, in terms of k, when the value of y
find the approximate value of y after the change in x changes from 5 to 5 + k.
has taken place. 12
Diberi y = – , cari perubahan kecil dalam x, dalam
6 x +6
Diberi y = 2x – 1 , cari perubahan kecil dalam y apabila sebutan k, apabila nilai bagi y berubah daripada 5 kepada
x menokok daripada 2 kepada 2.03. Seterusnya, cari nilai 5 + k.
hampir bagi y selepas perubahan dalam x berlaku. dx
δy = (5 + k) − 5 = k = –12(6 − y)–2 (–1)
6 dy
y= δx = 2.03 − 2 12 = 12(6 − y)–2
2x − 1 y=– +6
= 0.03 x 12
y = 6(2x − 1)–1 =
dy xy = –12 + 6x (6 – y)2
dy δy ≈ × δx dx 12
= –6(2x − 1)–2(2) dx When y = 5, =
dx xy – 6x = –12 dy (6 − 5)2
4
= –12(2x − 1)–2 = – (0.03) x(y – 6) = –12 = 12
3
12 –12 dx
=– = –0.04 x= δx ≈ × δy
(2x − 1)2 y–6 dy
x = 2, Approximate value
of y is 12 = 12(k)
dy 12 x =
=– y + δy 6−y = 12k
dx [2(2) − 1]2
6 = 12(6 – y)–1
12 = + (–0.04)
=– 2 2(2) − 1
3
= 2 − 0.04
4
=– = 1.96
3

Exercise 24 Solve each of the following problems.


Selesaikan setiap masalah yang berikut.
PL 4 Apply appropriate knowledge and skills of differentiation in the context of simple routine problem solving.

Example 24 1 The radius of a circle increases from 8 cm to 8.03 cm.


Find the approximate value of the area of the circle
The radius of a hemisphere changes from 6 cm to
after the change of radius.
6.02 cm. Find the approximate value of the volume Jejari bagi sebuah bulatan bertambah daripada 8 cm kepada
after the change of radius. 8.03 cm. Cari nilai luas bulatan yang hampir selepas perubahan
Jejari bagi sebuah hemisfera berubah daripada 6 cm kepada jejari.
6.02 cm. Cari nilai isi padu yang hampir selepas perubahan
jejari.
A = πr2
dA
Solution = 2πr
dr
2 Volume of hemisphere: dA
V = 3 πr3 r = 8, = 2π(8) = 16π
Isi padu hemisfera: dr
1 4 2
dV 2 × 3 πr = 3 πr
3 3
δr = 8.03 − 8
= 2πr2
dr = 0.03
dV
r = 6, = 2π(6)2 = 72π dA
dr δA ≈ × δr
dr
δr = 6.02 − 6
= (16π)(0.03)
= 0.02
= 0.48π
dV
δV ≈ × δr Approximate value of A is
dr
A + δA
= (72π)(0.02)
= π(8)2 + 0.48π
= 1.44π
= 64.48π cm2
Approximate value of V is
Nilai isi padu V yang hampir ialah

V + δV
2
= 3 π(6)3 + 1.44π
= 145.44π cm3

45

F5 Mod A+ ADDM 2(18-49).indd 45 6/1/2021 10:59:41 AM


2 The radius of a sphere changes from 3 cm to 3 The radius of a sphere changes from 3 m to 3.06 m.
2.96 cm. Find the approximate value of the surface Find the approximate value of the volume of the
area of sphere after the change of radius. sphere after the change of radius.
Jejari bagi sebuah sfera berubah daripada 3 cm kepada 2.96 cm. Jejari bagi sebuah sfera berubah daripada 3 m kepada 3.06 m.
Cari nilai luas permukaan sfera yang hampir selepas perubahan Cari nilai isi padu sfera yang hampir selepas perubahan jejari.
jejari.
4
V = 3 πr3
A = 4πr2 dV
dA = 4πr2
= 8πr dr
dr dV
dA r = 3, = 4π(3)2 = 36π
r = 3, = 8π(3) = 24π dr
dr
δr = 3.06 − 3
δr = 2.96 − 3 = 0.06
= –0.04
dV
dA δV ≈ × δr
δA ≈ × δr dr
dr = (36π)(0.06)
= (24π)(–0.04) = 2.16π
= –0.96π
Approximate value of V is
Approximate value of A is
4πr2 + δA V + δV
= 4π(3)2 + (–0.96π) 4
= 35.04π cm2 = 3 π(3)3 + 2.16π

= 38.16π m3

4 A spherical air bubble is produced from the base 5 Jamal bought a roll of barbed wire of length 68 m.
of a lake. When the air bubble comes to the water He wants to use all the barbed wire to fence up a
surface, it expands. If the radius of the air bubble rectangular area as shown in the diagram below, to
increases from 4.5 cm to 4.7 cm, find the increase in plant sweet potatoes. Find the maximum area, in m2,
volume of the air bubble. of the sweet potatoes field. Applying

Suatu gelembung udara yang berbentuk sfera terhasil dari dasar Jamal membeli segulung dawai berduri yang panjangnya
sebuah tasik. Apabila gelembung udara naik ke permukaan 68 m. Dia ingin menggunakan kesemua dawai berduri itu
air, gelembung itu mengembang. Jika jejari gelembung itu untuk memagar suatu kawasan yang berbentuk segi empat tepat
bertambah daripada 4.5 cm kepada 4.7 cm, cari tokokan dalam seperti ditunjukkan dalam rajah di bawah, untuk menanam ubi
isi padu gelembung udara itu. keledek. Cari luas maksimum, dalam m2, bagi kawasan tanaman
ubi keledek itu.
Volume of air bubble, V
4 Old fence/Pagar lama
V = 3 πr3
dV Barbed wire ym
= 4πr2
dr Dawai berduri

When r = 4.5,
dV xm
= 4π(4.5)2 x + 2y = 68
dr
= 81π 1
 y = 2 (68 − x)
dV
δV = × δr A = xy
dr
= (81π) × (4.7 − 4.5)
= (81π) × (0.2)
[1
= x 2 (68 − x) ]
= 16.2π cm3 1
= 34x − 2 x2
dA
= 34 − x
dx
dA
When = 0,
dx
34 − x = 0
x = 34
1
y = 2 [68 − (34)] = 17
A = (34)(17) = 578 m2

46

F5 Mod A+ ADDM 2(18-49).indd 46 6/1/2021 10:59:42 AM


Review 2
Paper 1

Section A (b) Given the normal to the curve y = x2 – 7x + 6 at


1 Determine the value of/ Tentukan nilai bagi point G is parallel to the straight line y = x + 5.
4n − 12 had 4n − 12 Find the equation of the normal to the curve at
(a) nlim
→ ∞ n2 − 9
/ n → ∞ n2 − 9 , point G.
Diberi normal kepada lengkung y = x2 – 7x + 6 pada titik G
3x − 6
2
3x2 − 6
(b) xlim
→ ∞ 8 − 12x 2
/ xhad
→ ∞ 8 − 12x 2
. adalah selari dengan garis lurus y = x + 5. Cari persamaan
[4 marks/markah] normal kepada lengkung pada titik G.
[4 marks/markah]
1
2 Given x = 2 + t 2 and y = 4t 4, find 4 (a) A balloon in the shape of a sphere with a radius of
1 r cm is leaking. The volume of the balloon
Diberi x = + t2 dan y = 4t4, cari
2 decreases at a rate of 128 cm3 s−1. Find the rate of
dx
(a) dt , change of its radius when the radius is 8 cm.
dy Sebiji belon yang berbentuk sfera mempunyai jejari r cm
(b) dx in terms of x. adalah bocor. Isi padu belon berkurang dengan kadar
dy 128 cm3 s−1. Cari kadar perubahan jejari apabila jejari ialah
dalam sebutan x.
dx 8 cm.
[4 marks/markah]
[4 marks/markah]
(b) Safwan wants to fence up his farm that
Section B
is adjacent to a lake. He wants to build a
3 (a) The straight line 6x + ky = 12 is parallel to the rectangular area using 300 m of wire. If one side
2x of the fence is the lake, calculate the maximum
tangent of the curve y =
3x – 2 at point (1, 2). area, in m2, of the fence that he can build.
Determine the value of k. Safwan ingin memasang pagar di kawasan ladangnya yang
Garis lurus 6x + ky = 12 adalah selari dengan tangen bersebelahan dengan tasik. Dia ingin membina kawasan
2x
kepada lengkung y = pada titik (1, 2). Tentukan berbentuk segi empat tepat menggunakan dawai sepanjang
3x – 2
nilai k. 300 m. Jika satu sisi daripada pagar itu ialah tasik, hitung
[4 marks/markah] luas maksimum, dalam m2, pagar yang dapat dibinanya.
[4 marks/markah]

Paper 2
Section A
h
1 Given the curve y = x + , where h is a constant passes through point (−4, −5).
x
h
Diberi lengkung y = x + , dengan keadaan h ialah pemalar melalui titik (−4, −5).
x
(a) Find the value of h.
Cari nilai h.
[2 marks/markah]
(b) Find the coordinates of all the turning points of the curve.
Cari koordinat bagi semua titik pusingan bagi lengkung itu.
[4 marks/markah]
(c) Sketch the curve.
Lakarkan lengkung itu.
[2 marks/markah]

47

F5 Mod A+ ADDM 2(18-49).indd 47 6/1/2021 10:59:42 AM


2 Diagram 1 shows a rectangular garden. ABFCD is a lawn while BFC is a lake.
Rajah 1 menunjukkan sebuah taman berbentuk segi empat tepat. ABFCD ialah halaman rumput manakala BFC ialah sebuah tasik.
metre/ meter
A B

FF 2r metre/ meter

D C
Diagram 1/ Rajah 1
(a) Given the length of the fence around the lawn is 120 m. Show that the area, A m2, of the lawn is represents by
3
(
A = 120r – 2 + π r 2.
2 )
Diberi panjang pagar di sekeliling halaman rumput ialah 120 m. Tunjukkan bahawa luas, A m2, bagi halaman rumput diwakili oleh
3
(
A = 120r – 2 + π r 2.
2 )
[2 marks/markah]
(b) Find the value of r such that A is maximum.
Cari nilai r supaya A adalah maksimum.
[4 marks/markah]
(c) Find the maximum area, in m2, of the lawn.
Cari luas maksimum, dalam m2, bagi halaman rumput itu.
[Use/Guna π = 3.142]
[2 marks/markah]
21
3 Given the equation of a curve is y = 3 .
x
21
Diberi persamaan bagi suatu lengkung ialah y = .
x3
dy
(a) Find the value of when x = 2.
dx
Cari nilai dy apabila x = 2.
dx
[3 marks/markah]
21
(b) Hence, determine the approximate value of correct to two decimal places.
(1.98)3
Seterusnya, tentukan nilai hampir bagi 21 betul kepada dua tempat perpuluhan.
(1.98)3
[4 marks/markah]

4 Given the equation of a curve is y = 5 + x2(6 – x).


Diberi persamaan bagi suatu lengkung ialah y = 5 + x2(6 – x).
(a) Determine the gradient function of the curve.
Tentukan fungsi kecerunan bagi lengkung itu.
[2 marks/markah]
(b) Find the coordinates of all the turning points.
Cari koordinat bagi semua titik pusingan.
[3 marks/markah]
(c) Determine whether each of the turning points is a maximum point or a minimum point.
Tentukan sama ada setiap titik pusingan itu ialah titik maksimum atau titik minimum.
[3 marks/markah]

5 It is given that a curve y = 2x(3 – x)4 passes through the point P(2, 5).
Diberi bahawa suatu lengkung y = 2x(3 – x)4 melalui titik P(2, 5).
Find/ Cari
(a) the gradient of the tangent to the curve at point P,
kecerunan tangen kepada lengkung pada titik P,
[4 marks/markah]
(b) the equation of the normal to the curve at point P.
persamaan normal kepada lengkung pada titik P.
[3 marks/markah]

48

F5 Mod A+ ADDM 2(18-49).indd 48 6/1/2021 10:59:43 AM


6 Diagram 2 shows a conical container with a height of 15 cm and a diameter of 9 cm. The container contains some water
where the depth of water is h cm.
Rajah 2 menunjukkan sebuah bekas berbentuk kon dengan tinggi 15 cm dan diameter 9 cm. Bekas itu berisi air dengan keadaan kedalaman
air ialah h cm.
4.5 cm

r cm
15 cm

h cm

Diagram 2/ Rajah 2
(a) Express r in terms of h.
Ungkapkan r dalam sebutan h.
[2 marks/markah]
3
(b) Show that the volume, V cm3, of the water in the container is given by V = πh3.
100
3
Tunjukkan bahawa isi padu, V cm3, air di dalam bekas itu diberi oleh V = πh3.
100
[2 marks/markah]
(c) The water flows out from a small hole at the tip of the container at a constant rate of 1.8 cm3 s−1. Find the rate
of change of the height of water level when the height of water level is 6 cm.
Air mengalir keluar melalui lubang kecil di hujung bekas itu dengan kadar tetap 1.8 cm3 s−1. Cari kadar perubahan ketinggian paras
air di dalam bekas apabila ketinggian paras air ialah 6 cm.
[3 marks/markah]

Section B
7 A curve y = x3 – 6x2 + 9x – 6 passes through the point M(2, −4) and has two turning points, K(3, −6) and L. Find
Satu lengkung y = x3 – 6x2 + 9x – 6 melalui titik M(2, −4) dan mempunyai dua titik pusingan, K(3, −6) dan L. Cari
(a) the gradient of the tangent to the curve at point M,
kecerunan tangen kepada lengkung pada titik M,
[3 marks/markah]
(b) the equation of the normal to the curve at point M,
persamaan normal kepada lengkung pada titik M,
[3 marks/markah]
(c) the coordinates of L and determine whether L is a maximum point or a minimum point.
koordinat L dan tentukan sama ada L ialah titik maksimum atau titik minimum.
[4 marks/markah]

H O TS Zo n e
1
1 The daily profit, U, in RM, of a bakery is given by U = 42x − x2, where x is the number of cakes of same type
6
being sold. How many cakes must be sold to get maximum profit and what is the maximum profit of the bakery?
Applying
1
Keuntungan harian, U, dalam RM, bagi sebuah kedai kek diberi oleh U = 42x − x2, dengan keadaan x ialah bilangan kek yang sama jenis
6
dijual. Berapakah bilangan kek yang mesti dijual supaya mendapat keuntungan maksimum dan berapakah keuntungan maksimum kedai
kek tersebut?

49

F5 Mod A+ ADDM 2(18-49).indd 49 6/1/2021 10:59:43 AM


Learning Area: Calculus

3 Integration
Pengamiran
3.1 Integration as the Inverse of Differentiation / Pengamiran sebagai Songsangan Pembezaan

Smart Tip
d
Integration is a technique of finding a function, f(x) from the derivative, which is [f(x)]. This process is denoted by the sign
dx
∫ ... dx, as in ∫ f(x) dx. Integration is known as the reverse process of differentiation.
d
Pengamiran ialah suatu teknik untuk mencari suatu fungsi, f(x) daripada pembezaan, iaitu [f(x)]. Proses ini diwakilkan dengan tatatanda
dx
∫ ... dx, seperti ∫ f(x) dx. Pengamiran dikenali sebagai proses songsangan bagi pembezaan.
d
If [f(x)] = f (x), then ∫ f (x) dx = f(x)
dx
d
Jika [f(x)] = f (x), maka ∫ f (x) dx = f(x)
dx

Exercise 1 Determine the following integral based on the result of differentiation given.
Tentukan kamiran berikut berdasarkan hasil pembezaan yang diberikan.
PL 1 Demonstrate the basic knowledge of integration.

Example 1 d
1 If (7x) = 7, determine ∫ 7 dx.
d dx
(a) Given (6x) = 6, determine ∫ 6 dx. d
dx Jika (7x) = 7, tentukan ∫ 7 dx.
dx
d
Diberi (6x) = 6, tentukan ∫ 6 dx.
dx d
d (7x) = 7
(b) Given (4x2 – 3x) = 8x – 3, determine dx
dx [ ∫ 7 dx = 7x
∫ (8x – 3) dx.
d
Diberi (4x2 – 3x) = 8x – 3, tentukan ∫ (8x – 3) dx.
dx

Solution
d
(a) (6x) = 6 d
dx 2 If (4x2) = 8x, determine ∫ 8x dx.
[ ∫ 6 dx = 6x dx
d
Jika (4x2) = 8x, tentukan ∫ 8x dx.
dx
d
(b) (4x2 – 3x) = 8x – 3
dx d
(4x2) = 8x
[ ∫ (8x – 3) dx = 4x2 – 3x dx
[ ∫8x dx = 4x2

3 If
d 3
dx
(x + x2) = 3x2 + 2x, determine ∫ (3x2 + 2x) dx. 4 If ( )
d 6
dx x x
6
x
6
( )
= – 2 , determine ∫ – 2 dx.

dx ( x )
= – 6 , tentukan ∫ (– 6 ) dx.
d 3 d 6
Jika (x + x2) = 3x2 + 2x, tentukan ∫ (3x2 + 2x) dx. Jika
dx x 2
x2

d 3
dx
(x + x2) = 3x2 + 2x d 6
dx x ( ) 6
=– 2
x
[ ∫(3x2 + 2x) dx = x3 + x2
( )
6
[ ∫ – 2 dx =
x
6
x

50

F5 Mod A+ ADDM 3(50-77).indd 50 6/1/2021 11:02:19 AM


Exercise 2 Solve each of the following.
Selesaikan setiap yang berikut.
PL 2 Demonstrate the understanding of integration.

Example 2
1 Given
d 1
dx 3x + 2 (
=–
3
)
(3x + 2)2
, determine

Given
d 1
dx 2x + 1(=–
2
(2x + 1)2 )
, determine

12
(3x + 2)2
dx.
6
∫ (2x + 1)2 dx. Diberi d
1
dx 3x + 2(=–
3
)
(3x + 2)2
, tentukan ∫
12
(3x + 2)2
dx.

Diberi d
1
(
dx 2x + 1
=–
2
(2x + 1)2), tentukan ∫
6
(2x + 1)2
dx.
d 1
dx 3x + 2 ( =– )3
(3x + 2)2
Solution
3 1
d
( 1
dx 2x + 1
=– ) 2
(2x + 1)2
[ ∫–
(3x + 2)2
dx =
3x + 2
× (–4)

12 4
[ ∫–
2
dx =
1
× (–3) ∫ (3x + 2)2 dx = – 3x + 2
(2x + 1)2 2x + 12
6 3
∫ (2x + 1)2 dx = – 2x + 1

2 Given
d 2
(
dx 5x – 3
=–
10
(5x – 3)2 )
, determine 3 Given
d 3x
dx 4x + 3
2 (=
9 – 12x2
(4x2 + 3)2), determine
30 3 – 4x2
∫ (5x – 3)2 dx. ∫ (4x2 + 3)2
dx.

Diberi d
2
dx 5x – 3(=–
10
(5x – 3)2 )
, tentukan ∫
30
(5x – 3)2
dx. Diberi d
3x
(
dx 4x2 + 3
= )
9 – 12x2
(4x2 + 3)2
, tentukan ∫
3 – 4x2
(4x2 + 3)2
dx.

d 2
dx 5x – 3 ( =–
10
(5x – 3)2 ) d
(
3x
dx 4x + 3
2
= )9 – 12x2
(4x2 + 3)2
10 2
[ ∫– 9 – 12x 3x
( 13 )
2
dx = × (–3)
(5x – 3)2 5x – 3 [ ∫ (4x2 + 3)2
dx =
4x2 + 3
×
30 6
∫ (5x – 3)2 dx = – 5x – 3
∫ (4x2 + 3)2 dx = 3 × ( 4x2 + 3 )
3 – 4x2 1 3x

x
=
4x2 + 3

d d
4 Given (3x + 1)4 = 12(3x + 1)3, determine ∫6(3x + 1)3 dx. 5 Given (2x – 3)9 = 18(2x – 3)8, determine ∫(2x – 3)9 dx.
dx dx
d d
Diberi (3x + 1)4 = 12(3x + 1)3, tentukan ∫ 6(3x + 1)3 dx. Diberi (2x – 3)9 = 18(2x – 3)8, tentukan ∫ (2x – 3)9 dx.
dx dx
d d
(3x + 1)4 = 12(3x + 1)3 (2x – 3)9 = 18(2x – 3)8
dx dx
[ ∫ 12(3x + 1)3 dx = (3x + 1)4 × ( 12 ) [ ∫18(2x – 3)8 dx = (2x – 3)9 × ( 181 )
1
1 ∫(2x – 3)8 dx = (2x – 3)9
∫ 6(3x + 1)3 dx = 2 (3x + 1)4 18

51

F5 Mod A+ ADDM 3(50-77).indd 51 6/1/2021 11:02:20 AM


3.2 Indefinite Integral/ Kamiran Tak Tentu

Smart Tip
axn + 1
1 ∫ axn dx = + c, where a and c are constants, n is an integer and n ≠ –1.
n+1
axn + 1
∫ axn dx = + c, dengan keadaan a dan c ialah pemalar, n ialah integer dan n ≠ –1.
n+1
2 ∫ k dx = kx + c, where k and c are constants.
∫ k dx = kx + c, dengan keadaan k dan c ialah pemalar.

Exercise 3 Find the indefinite integral for each of the following.


Cari kamiran tak tentu bagi setiap yang berikut.
PL 2 Demonstrate the understanding of integration.

Example 3 1 ∫x3 dx 2 ∫6x2 dx


6
(a) ∫10x3 dx (b) ∫ x5 dx ∫x3 dx ∫6x2 dx
x3 + 1 6x2 + 1
Solution = +c = +c
3+1 2+1
6 x 4
6x 3
(a) ∫ 10x3 dx (b) ∫ x5 dx =
4
+c =
3
+c
10x3 + 1
= +c = ∫6x –5 dx
3+1 = 2x3 + c
10x 4
6x–5 + 1
= +c = +c
4 –5 + 1
5x4 6x–4
= +c = +c
2 –4
3
=– 4 +c
2x

3 ∫ 9 dx x 5 ∫4x5 dx 6 ∫–x6 dx
4 ∫3 dx

∫ 9 dx = 9x + c x ∫4x5 dx ∫–x6 dx
∫ 3 dx1 + 1 4x5 + 1 x6 + 1
x = +c =– +c
= +c 5+1 6+1
3(1 + 1) 4x6 x7
x 2 = +c =– +c
= +c 6 6+1
6 2 x 7
= x6 + c =– +c
3 7

4 8 dx 3 10 ∫6 x dx
7 ∫ 5 x 3 dx 8 ∫ x3 9 ∫ 4x6 dx
4 8 3 ∫6 x dx
∫ 5 x 3 dx ∫ x3 dx ∫ 4x6 dx 1
3 1
( )
4 x3 + 1
= 5 3+1 +c
= ∫8x– 3 dx
8x–3 + 1
= 4 ∫ x6 dx
= ∫6x 2 dx
6x 2
1
+1
= –3 + 1 + c 3 –6
4 x4
( )
= 5 4 +c 8x–2
= 4 ∫x dx
=
1
2 +1
+c

1
= x4 + c
5
= –2 + c
4
(
3 x–6 + 1
)
= 4 –6 + 1 + c
3
6x 2
=– 2 +c = +c
3x–5 3
x =– +c
20 2
3
3 = 4x 2 + c
=– +c
20x5 = 4 x3 + c

52

F5 Mod A+ ADDM 3(50-77).indd 52 6/1/2021 11:02:20 AM


Exercise 4 Integrate each of the following.
Kamirkan setiap yang berikut.
PL 2 Demonstrate the understanding of integration.

Example 4
1 ∫ (3x2 – 3x + 5) dx
∫(x – 1)(x + 4) dx
∫ (3x2 – 3x + 5) dx
Solution 3x3 3x2
= – + 5x + c
∫(x – 1)(x + 4) dx 3 2
3x 2

= ∫ (x2 + 3x – 4) dx = x3 – + 5x + c
Smart Tip 2
x2 + 1 3x1 + 1
= + – 4x + c ∫[f(x) g(x)] dx
2+1 1+1
x 3
3x 2 = ∫f(x) dx ∫g(x) dx
= + – 4x + c
3 2

2 ∫ (2x3 + 4x2 + 10x) dx 3 ∫ 2x2(4x – 6) dx

∫ (2x3 + 4x2 + 10x) dx ∫ 2x2(4x – 6) dx


2x4 4x3 10x2 = ∫ (8x3 – 12x2) dx
= + + +c 8x4 12x3
4 3 2 = – +c
x 4
4x 3 4 3
= + + 5x2 + c = 2x4 – 4x3 + c
2 3

∫ (x + x )(x – x ) dx ∫ (x – x )
3 3 2 2
4 5 dx

∫ (x + x )(x – x ) dx ∫ (x – x )
3 3 2 2
dx

(
= ∫ x2 –
9
x2
dx ) ( 2x )(x – 2x ) dx
=∫ x–

= ∫ (x – 2 – 2 + ) dx
= ∫ (x2 – 9x–2) dx 2 4
x 2
x3 9x–1
= – +c = ∫ (x – 4 + 4x ) dx
2 –2
3 –1
x3 9 x3 4x–1
= + +c = – 4x + +c
3 x 3 –1
x 3
4
= – 4x – +c
3 x

∫ (x2 – ) dx
1 2 (x4 + 2) (x4 + 2)
6
x2
7 ∫ x4
dx

(x4 + 2) (x4 + 2)
∫( )
1

2
x2 – dx dx
x2 x4
(x + 2x + 2x + 4)
( x1 )(x – x1 ) dx
8 4 4

= ∫ x2 – 2 =∫ dx
2 2 x4
= ∫ (x – 1 – 1 + ) dx
1 (x8 + 4x4 + 4)
4 =∫ dx
x 4 x4
= ∫ (x – 2 + x ) dx
4 –4 = ∫ (x4 + 4 + 4x–4) dx
x5 x–3 x5 4x–3
= – 2x + +c = + 4x + +c
5 –3 5 –3
x5 1 x 5
4
= – 2x – 3 + c = + 4x – 3 + c
5 3x 5 3x

53

F5 Mod A+ ADDM 3(50-77).indd 53 6/1/2021 11:02:21 AM


Smart Tip
Given ∫ (ax + b)n dx, where a and b are constants, n is an
integer and n ≠ –1.
So/Maka, ∫(ax + b)n dx = ∫ un ( dua )
1
Diberi ∫ (ax + b)n dx, dengan keadaan a dan b ialah pemalar, n ialah = ∫un du
a
integer dan n ≠ –1.

By using substitution method/ Dengan menggunakan kaedah penggantian,


= (
1 un + 1
a n+1
+c )
Let/Katakan u = ax + b
du
=
a (
1 (ax + b) n + 1
n+1
+c)
Thus/Jadi, =a (ax + b) n + 1
dx = +c
du a(n + 1)
dx =
a

Exercise 5 Integrate each of the following with respect to x.


Kamirkan setiap yang berikut terhadap x.
PL 2 Demonstrate the understanding of integration.

Example 5
1 ∫ 3(5x + 4)3 dx
∫4(2x – 3)3 dx u = 5x + 4
du
=5
Solution dx du
Given/ Diberi ∫4(2x – 3)3 dx dx =
5
Let/ Katakan u = 2x – 3
du ∫ 3(5x + 4)3 dx Alternative Method
Thus/ Jadi, =2 Alternative Method
dx du ∫3(5x + 4)3 dx
du ∫ 4(2x – 3)3 dx = ∫ 3u3
dx = 5
= 3 × (5x + 4)
3+1
2 (2x – 3)3 + 1 3 +c
=4×
2(3 + 1)
+c = ∫ u du3 5(3 + 1)
du 5
= 3 (5x + 4)4 + c
∫ 4(2x – 3) dx = ∫ 4u
( )
3 3
2 (2x – 3)4 + c 3 u4 20
= = +c
4 3 2 5 4
= ∫ u du 3
2 = u4 + c
=2
u4
4 ( )
+c
20
3
= (5x + 4)4 + c
=2 [
(2x – 3)4
4
+c ] 20

(2x – 3)4
= +c
2

2 ∫(4 – x)3 dx 3 ∫ 5(4x – 3)6 dx


u=4–x u = 4x – 3
du du
= –1 =4
dx dx du
du dx =
= dx 4
–1
du
–du = dx ∫ 5(4x – 3)6 dx = ∫ 5u6
4
∫(4 – x)3 dx = ∫ u3 (–du) 5 6
4∫
= u du
= ∫ –u3 du

=–
u4
+c
= ( )
5 u7
4 7
+c
4 5
(4 – x)4 = (4x – 3)7 + c
=– +c 28
4

Alternative Method Alternative Method


∫ (4 – x) dx
3
∫5(4x – 3)6 dx

= (4 – x) = 5 × (4x – 3)
3+1 6+1
+c +c
–1(3 + 1) 4(6 + 1)
= – (4 – x) + c = 5 (4x – 3)7 + c
4

4 28

54

F5 Mod A+ ADDM 3(50-77).indd 54 6/1/2021 11:02:22 AM


3 3
4 ∫ (4x – 1)2 dx 5 ∫ (5x – 1)3 dx
3 3
∫ (4x – 1)2 dx = ∫ 3(4x – 1)–2 dx ∫ (5x – 1)3 dx = ∫3(5x – 1)–3 dx
u = 4x – 1 u = 5x – 1
du du
=4 =5
dx du dx du
dx = dx =
4 5
du du
∫3(4x – 1)–2 dx = ∫ 3u–2 ∫3(5x – 1)–3 dx = 3 ∫ u–3
4 5
3 –2 3
= ∫ u du–3

4∫
= u du 5
=
4 –1( )
3 u–1
+c =
3 u–2
5 –2 ( ) +c

3 3
=– +c = – u–2 + c
4u 10
3 3
=– +c =– +c
4(4x – 1) 10(5x – 1)2

Alternative Method Alternative Method


∫ 3(4x – 1) dx
–2
∫3(5x – 1)–3 dx
= 3 × (4x – 1) = 3 × (5x – 1)
–2 + 1 –3 + 1
+c +c
4(–2 + 1) 5(–3 + 1)
= 3 × (4x – 1) + c = 3 × (5x – 1) + c
–1 –2

4(–1) 5(–2)
= – 3 (4x – 1)–1 + c 3
= – (5x – 1)–2 + c
4 10
=– 3 +c 3
=– +c
4(4x – 1) 10(5x – 1)2

Exercise 6 Find the equation of the curve for each of the gradient function that passes through the
following points.
Cari persamaan lengkung bagi setiap fungsi kecerunan yang melalui titik yang berikut.
PL 3 Apply the understanding of integration to perform simple tasks.

Example 6 dy
1 = 8x – 1; (0, 5)
dx
dy
= (2x –1)3; (1, 0) y = ∫ (8x –1) dx
dx
8x2
Solution y= –x+c
2
y = ∫ (2x –1)3 dx y = 4x2 – x + c
(2x – 1) 3 + 1
y= +c
2(3 + 1)
When x = 0 and y = 5
(2x – 1) 4
y= +c 5 = 4(0)2 – 0 + c
8
c=5
When x = 1 and y = 0/ Apabila x = 1 dan y = 0 y = 4x2 – x + 5
[2(1) – 1]4
0= +c
8
1
0= +c
8
1
c=–
8
(2x – 1)4 1
y= –
8 8

55

F5 Mod A+ ADDM 3(50-77).indd 55 6/1/2021 11:02:22 AM


dy dy
2 = 6x2 – 6x; (2, 9) 3 = 12x4 – 3x2 + 6; (1, 7)
dx dx
y = ∫ (6x2 – 6x) dx y = ∫ (12x4 – 3x2 + 6) dx
6x3 6x2 12x5 3x3
y = – +c y = – + 6x + c
3 2 5 3
y = 2x3 – 3x2 + c 12 5
y= x – x3 + 6x + c
5
When x = 2 and y = 9
9 = 2(2)3 – 3(2)2 + c When x = 1 and y = 7
12
c = 5 (1)5 – (1)3 + 6(1) + c
(7) =
5
y = 2x3 – 3x2 + 5 2
c = –
5
12 5 2
y = x – x3 + 6x –
5 5

dy 3 dy
4 = 2 – 1; (3, 0) 5 = (4x – 5)3; (1, 0)
dx x dx
y = ∫ (3x–2 – 1) dx y = ∫ (4x – 5)3 dx
3x –1
(4x – 5)4
y = –x+c y = +c
–1 4(4)
3 (4x – 5) 4
y = – – x + c y= +c
x 16

When x = 3 and y = 0 When x = 1 and y = 0


3
0=– – (3) + c [4(1) – 5]4
(3) 0 = +c
c=4 16
3 1
y=– –x+4 0 = +c
x 16
1
c = –
16
(4x – 5)4 1
y = –
16 16

dy 5 dy 6
6 = ; (1, 3) 7 = ; (1, 2)
dx (4x – 3)2 dx (2x + 1)3

y = ∫ 5(4x – 3)–2 dx y = ∫ 6(2x + 1)–3 dx


y = 5∫(4x – 3)–2 dx y = 6∫ (2x + 1)–3 dx

[ (4x4(–1)
– 3)
]+c [ (2x2(–2)
+ 1)
]+c
–1 –2
y=5 y=6

y = 5[ ]+c y = 6[ ]+c
(4x – 3) –1
(2x + 1) –2

–4 –4
5 3
y = – +c y = – +c
4(4x – 3) 2(2x + 1)2

When x = 1 and y = 3 When x = 1 and y = 2


5 3
3 = – +c 2 = – +c
4[4(1) – 3] 2[2(1) + 1]2
5 3
3 = – +c 2 = – +c
4(1) 18
5 13
c = 3 + c =
4 6
17 3 13
c = y = – +
4 2(2x + 1)2 6
5 17
y = – +
4(4x – 3) 4

56

F5 Mod A+ ADDM 3(50-77).indd 56 6/1/2021 11:02:23 AM


3.3 Definite Integral / Kamiran Tentu

Smart Tip
y
1 The definite integral of a function f(x) with respect to x for
the interval from x = a to x = b is written as:
Kamiran tentu bagi suatu fungsi f(x) terhadap x untuk batasan dari y = f(x)
x = a ke x = b ditulis sebagai:
b
a
f(x) dx = [g(x) + c] ba
= [g(b) + c] – [g(a) + c]
a x
= g(b) + c – g(a) – c O b
= g(b) – g(a)
y
2 Reversing the direction of the interval gives the negative
of the original definite integral. y = f(x)
Penukaran arah batasan akan menyebabkan negatif kepada
kamiran tentu yang asal.
b a
a
f(x) dx = – b
f(x) dx
x
O b a

3 Definite integral involving two continuous intervals is y


written as:
Kamiran tentu yang melibatkan dua batasan selanjar boleh ditulis y = f(x)
sebagai:
b c c
a
f(x) dx + b
f(x) dx = a
f(x) dx

a x
O b c

Exercise 7 Evaluate each of the following definite integrals.


Nilaikan setiap kamiran tentu berikut.
PL 2 Demonstrate the understanding of integration.

Example 7 4
1 2
7x dx
3
(a) –1
5x dx
2 4
(b) 1
(3x3 – x2 + 1) dx 7x dx
2

Solution =
7x2 4
2 2 [ ]
[ 5x2 ] [ ] [ ]
2 3
(a)
3
5x dx = 7(4)2 7(2)2
–1 = –
–1 2 2
=[
2 ] [ 2 ]
5(3) 5(–1)
2 2
= 56 – 14

= 42
= 20

2
(b) 1
(3x3 – x2 + 1) dx

[ ]
2
3x 4 x 3 2
2 2x2 dx
= – +x –1
4 3 1

= [
3(2)4 (2)3
4

3
+2 –
3(1)4 (1)3
4

3 ] [
+1 ] 2
2x2 dx
–1

=
34 17
3

12
=
2x 3 2
3 –1[ ]
=
119
12
=
2(2)3
3 [ ] [

2(–1)3
3 ]
=
16
3
– – ( )
2
3
=6

57

F5 Mod A+ ADDM 3(50-77).indd 57 6/1/2021 11:02:24 AM


3
3
2
(3x + 2) dx 4 0
(6x2 – 2x + 4) dx
1

2
1
(3x + 2) dx 3
(6x2 – 2x + 4) dx
0

[
3 2
] = [2x3 – x2 + 4x] 30
2
= x + 2x
2 1 = [2(3)3 – (3)2 + 4(3)] – [2(0)3 – (0)2 + 4(0)]
=
2[
3 2
(2) + 2(2) –
3 2
2
(1) + 2(1) ] [ ] = 57 – 0
= 57
7
= 10 –
2
13
=
2

5
2
(2x – 1)(x2 + 2) dx
3 4x2 + 1
6 dx
0
2 x2
2
(2x – 1)(x + 2) dx 2
0 3 4x2 + 1
2
= 0 (2x3 + 4x – x 2 – 2) dx dx
2 x2
[
2x 4 4x2 x 3
]
2 3
= + – – 2x = 2 (4 + x–2) dx
4 2 3 0

[
x4 x
] [ x–1 ]
3 2 –1 3
= + 2x2 – – 2x = 4x +
2 3 0 2

[ ] [ ] = [4x – ]
(2) 4
(2) 3
(0)4 (0)3 1 3
= + 2(2)2 – – 2(2) – + 2(0)2 – – 2(0)
2 3 2 3 x 2

= [4(3) – ] – [4(2) – ]
28 1 1
= –0
3 3 2
28 25
= =
3 6

(2x – 3x ) dx 1
–1 2 1
7 8 dx
–2 –1 (2 – x)3

–1
(4x2 – 12 + 9x–2) dx 1 1
–2 dx
(2 – x)3
[ ]
–1
4x 3 9x –1 –1
= – 12x +
3 –1 –2 =
1
(2 – x)–3 dx
[ ] [ ]
–1
4(–1)3 9 4(–2)3 9
=
3
– 12(–1) –
(–1)

3
– 12(–2) –
(–2) =[ (2 – x)–2 1
–2(–1) –1 ]
59 107
=[ ]
= – 1 1
3 6 2(2 – x) –1
2

11 1 1
= = –
6 2(2 – 1)2 2[2 – (–1)]2
4
=
9

Calculator Corner
3
Find the value of 1
(2x4 – x2 + 3) dx./ Cari nilai bagi 31 (2x4 – x2 + 3) dx.

By using scientific calculator Casio fx-570EX./ Dengan menggunakan kalkulator saintifik Casio fx-570EX.

Step 1: Press ‘MENU’ and find 1: Calculate (normal display), press ‘ = ’.


Langkah 1: Tekan ‘MENU’ dan cari 1: Calculate (paparan normal), tekan ‘ = ’.
Step 2: Key in the values and press ‘ = ’ to get the answer.
Langkah 2: Masukkan nilai dan tekan ‘ = ’ untuk mendapatkan jawapan.

∫ 2 x x 4 – x x2 + 3 1 3 =

1 412
Answer/ Jawapan:
15

58

F5 Mod A+ ADDM 3(50-77).indd 58 6/1/2021 11:02:25 AM


Exercise 8 Solve each of the following.
Selesaikan setiap yang berikut.
PL 2 Demonstrate the understanding of integration.

Example 8 5
1 Given 1
f(x) dx = 3, find the value of
Given
3
1
f(x) dx = 5, find the value of Diberi 5
1
f(x) dx = 3, cari nilai
1
Diberi 3
1
f(x) dx = 5, cari nilai (a) 5
[f(x) dx,
5
(a)
3
1
[f(x) + 2] dx, (b)
3
1
[4f(x) – x] dx (b) 1
[f(x) + 1] dx.
1
Solution (a) 5
f(x) dx
3
(a) 1 [f(x) + 2] dx =–
5
f(x) dx
1
3 3
= 1
f(x) dx + 1
2 dx = –3
= 5 + [2x ] 3
1
5
= 5 + [2(3) – 2(1)] (b) 1
[f(x) + 1] dx
=9 5
= 1 f(x) dx +
5
1 dx
1

3 = 3 + [x] 51
(b) 1
[4f(x) – x] dx = 3 + (5 – 1)
3 3
=4 f(x) dx – 1 x dx
1 =3+4
= 4(5) −
x2 3
2 1 [ ] =7

= 20 −
(3)2 (1)2
2

2 [ ]
= 20 – 4
= 16

2
2 Given
4
g(x) dx = 6, find the value of 3 Given –1
g(x) dx = 4, find the value of
1
4 2
Diberi g(x) dx = 6, cari nilai Diberi –1
g(x) dx = 4, cari nilai
1

3
[ g(x) + ] dx,
1
4 2
(a) g(x) dx, (a)
1 4 –1 6 2
[3
]
4
2
(b) g(x) + x dx. (b) [3g(x) – x2] dx.
1 2 –1

3
[ g(x) + ] dx
1
4 2
(a) g(x) dx (a)
1 4 –1 6 2
3 4 1 2 2 1
= g(x) dx = g(x) dx + dx
4 1 6 –1 –1 2
3
= (6)
4 =
1
6
(4) +
1 2
x
2 –1 [ ]
9
=
2 =
2
3
1
[1
+ (2) – (–1)
2 2 ]
2 3
= +
3
[ ] 3 2
4
(b) g(x) + x dx 13
1 2
=
3 4 4
6
= g(x) dx + 1 x dx
2 1
3
= (6) +
2
x2 4
2 1 [ ] (b)
2
–1
[3g(x) – x2] dx

=9+
(4)2 (1)2
2

2 [ ] =3
2
–1
g(x) dx –
2
–1
x2 dx

[ x3 ]
3
15 = 3(4) –
2
=9+
2 –1

=
33
2
= 12 –
(2)3 (–1)3
3

3 [ ]
= 12 – 3
=9

59

F5 Mod A+ ADDM 3(50-77).indd 59 6/1/2021 11:02:26 AM


2 5 1 1
4 Given 1
f(x) dx = 3 and 2
f(x) dx = 6, find the value of 5 Given –2
f(x) dx = 4 and –2
g(x) dx = 3, find the value
Diberi
2
f(x) dx = 3 dan
5
f(x) dx = 6, cari nilai of
1 2 1 1
5 Diberi –2
f(x) dx = 4 dan –2
g(x) dx = 3, cari nilai
(a) f(x) dx, 1 1
(a) [3f(x) + 2g(x)] dx,
[ 2
]
5 –2
(b) f(x) + 4x dx. 1
1 3 k if
(b) –2
[f(x) + kx] dx = 16.
1
5 k jika –2 [f(x) + kx] dx = 16.
(a) 1 f(x) dx
2 5
= 1 f(x) dx + f(x) dx 1
2 (a) –2
[3f(x) + 2g(x)] dx
= 3 + 6 1 1
=3 f(x) dx + 2 g(x) dx
= 9 –2 –2
= 3(4) + 2(3)
2
1 3
5
(b) f(x) + 4x dx [ ]

= 12 + 6
= 18
2 5 5
=   f(x) dx + 4 x dx
3 1 1
1

[ ]
2 x2 5 (b) –2
[f(x) + kx] dx = 16
= (9) + 4
3 2 1 f(x) dx +
1 1
kx dx = 16
–2 –2
= 6 + [2x2] 51
4+k x [ ]
2 1
= 6 + [2(5)2 – 2(1)2] = 16
2 -2
= 6 + 48
[ ]
k (1) – (–2) = 12
2 2
= 54
2 2
[ ]
k – 3 = 12
2
k = − 8

x2 3x2 + 2x d
6 Given
d
dx 3x + 1 (
= (3x + 1)2 . ) 7 Given
dx
(4x2 – 5)3 = 24x(4x2 – 5)2.
2
1 3x2 + 2x Evaluate 1 3x(4x2 – 5)2 dx.
Evaluate 2 dx.
0 (3x + 1)
Diberi d (4x2 – 5)3 = 24x(4x2 – 5)2.
Diberi d
x2
dx 3x + 1
= (
3x2 + 2x
(3x + 1)2
. )
dx
Nilaikan 21 3x(4x2 – 5)2 dx.
1 3x2 + 2x
Nilaikan 2 dx.
0 (3x + 1)
2
1
24x(4x2 – 5)2 dx = [(4x2 – 5)3] 21 (4 8)

1

0
3x2 + 2x x2 1
(3x + 1)2 dx = 3x + 1 0 [ ]
2
1
3x(4x2 – 5)2 dx
(1)2 (0)2 1
= – =  [(4x2 – 5)3] 21
3(1) + 1 3(0) + 1 8
1 1 1
= =  [(4(2)2 – 5)3] –  [(4(1)2 – 5)3]
4 8 8
=
1 331
8
 – –
1
8 ( )
1 331 1
=  +  
8 8
333
=  
2

60

F5 Mod A+ ADDM 3(50-77).indd 60 6/1/2021 11:02:26 AM


Smart Tip
y y = f(x)
Area of the shaded region between a curve and the x-axis.
Luas rantau berlorek antara lengkung dengan paksi-x.

Area of the shaded region,


Luas rantau berlorek,
b
A= a
y dx x
O a b

Exercise 9 Find the area of the shaded region between the curve and the x-axis.
Cari luas bagi rantau berlorek antara lengkung dengan paksi-x.
PL 3 Apply the understanding of integration to perform simple tasks.

Example 9
1 y

y y = x2 + 1

y = x2 − 3x

x x
O 3 O 2

Area of the shaded region


Solution 2
Area of the shaded region = 0 y dx
2
Luas rantau berlorek = (x2 + 1) dx
0

[ ]
3 3
y dx = (x – 3x) dx 2 x3 2
0 0 = +x
3
[
x3 3x 2 3
]
0
= –
3 2 0 = [
(2)3
3
+ (2) –
(0)3
3 ] [
+ (0) ]
=
3 [
(3)3 3(3)2

2

(0)3 3(0)2
3

2 ] [ ] =
14
units2
3
1
= –4
2
= –4
1
2| 1
|
= 4 units2/ unit2
2

2 y 3 y

y = x2 − 1
y = (2 + x)(3 − x)

x
O 1 x
−2 O 3

Area of the shaded region Area of the shaded region


3
=
1
y dx = –2
y dx
0
3
=
1
(x2 – 1) dx = –2
(2 + x)(3 – x) dx
0
3

[ x3 – x]
3 1 = (6 + x – x2) dx
–2
=
0
[
= 6x +
x2

x3 3
]
=[ – (1)] – [ ]
(1) 3
(0) 3
2 3 –2
– (0)

=–
3
2
3
[
= 6(3) +
32 33
2

3
– 6(–2) +
2
– ] [
(–2)2 (–2)3
3 ]
3 5
= 20 units2
Area = – | 23 | = 23 unit 2 6

61

F5 Mod A+ ADDM 3(50-77).indd 61 6/1/2021 11:02:27 AM


4 y 5 y

y = (x 2)2
y = x3 − 4x2 + 3x

x
O 1 3
x
2 O 2 4

Area of the shaded region


=
4
y dx Area of the shaded region
–2 3
=
4
(x – 2)2 dx = 1
y dx
–2 3
=
4
(x2 – 4x + 4) dx = (x3 – 4x2 + 3x) dx
1

[ ]
–2
x4 4x3 3x2 3
[
x3 4x2
]
4
= – + 4x = – +
3 2 –2
4 3 2 1
[
(3) 4(3)3 3(3)2 (1)4 4(1)3 3(1)2
] [ ]
4

=
43
3 [ – 2(4)2 + 4(4) –
(–2)3
3 ] [
– 2(–2)2 + 4(–2) ] =
4

3
+
2

4

3
+
2
9 5
= 24 units2 =– –
4 12
8
=–
3
8
Area = – = units2
3
8
3 | |

Exercise 10 Find the area of the shaded region in each of the following diagrams.
Cari luas bagi rantau berlorek dalam setiap rajah yang berikut.
PL 3 Apply the understanding of integration to perform simple tasks.

Example 10
1 y
y
y = x3 − 4x
y = 9 − x2

x
−1 O 2
x
O 3 4

Area above the x-axis


Solution =
0
–1
y dx
Area above the x-axis/ Luas di atas paksi-x 0
3 = (x3 – 4x) dx
= 0 y dx –1

[ x4 – 2x ]
4 0
=
3
(9 – x2) dx = 2
0 –1

[
= 9x –
x3 3
3 0 ] =[
(0)
4
– 2(0) ] – [
4
(–1)
4
2
4
– 2(–1)2 ]
[ ] [ ] = 0 – (– ) =
(3)3 (0)3 7 7
= 9(3) – – 9(0) –
3 3 4 4
= 18
Area below the x-axis
Area below the x-axis/ Luas di bawah paksi-x
2
4
= 3 y dx = 0
y dx
2
=
4
(9 – x ) dx
3
2 = (x3 – 4x) dx
0

[ x3 4
] [
x4
]
2
= 9x – = – 2x2
3 3 4 0

[
= 9(4) –
(4)3
3
– 9(3) – ] [
(3)3
3 ] = [
(2)4
4
– 2(2)2 –
(0)4
4 ] [
– 2(0)2 ]
= –4 – 0
44 10
= – 18 = – = –4
3 3
7
Total area/ Jumlah luas = 18 + – | 103 | Total area = + |– 4|
4
23
64 = units2
= units2/ unit2 4
3

62

F5 Mod A+ ADDM 3(50-77).indd 62 6/1/2021 11:02:29 AM


2 y 3 y
24
y= −6
x2

y = x (x + 3)(x − 1)

x
−3 O 1 x
O 1 2 3

Area above the x-axis Area above the x-axis


0 2
= –3
y dx = y dx 1
24
( )
0
= –3
x(x + 3)(x – 1) dx =
2
– 6 dx
1 x
2
0
= (x3 + 2x2 – 3x) dx
24
[ ]
–3 2
= – – 6x
=
x4
[
+
2x3 3x2 0
– ] x 1
4 3 2 –3
= –
24
[ – 6(2) – –
24
– 6(1) ] [ ]
=
4 [
(0)4 2(0)3 3(0)2
+
3

2

(–3)4 2(–3)3 3(–3)2
4
+
3

2 ] [ ]
(2)
= –24 – (–30)
(1)

= 0 – –
45
4 ( ) =6
45
= Area below the x-axis
4 3
= 2 y dx
Area below the x-axis
=
1
0
y dx
=
3 24

2 x
2 (
– 6 dx )
24
[ ]
1 3
= (x3 + 2x2 – 3x) dx
0 = – – 6x
x
[ ]
2
x4 2x3 3x2 1
=
4
+
3

2 0 = –
24
(3)[ – 6(3) – –
24
(2)
– 6(2) ] [ ]
=
4[
(1)4 2(1)3 3(1)2
+
3

2

(0)4
4
+
2(0)3
3

3(0)2
2 ] [ ] = –26 – (–24)
7 = –2
=–
12
Total area = 6 + |–2|
Total area =
45
4
+ –
7
12 | | = 8 units2

45 7
= +
4 12
71
= units2
6

4 y
Area above the x-axis
y = 8x2 − x3 − 15x 5
= 3
y dx
5
= 3
(8x2 – x3 – 15x) dx
x

[ 8x3 – x4 – 15x2 ]
O 3 5 3 4 2 5
=
3

=[
2 ] [ 3 ]
8(5) (5) 15(5)
3
8(3) (3)
4 2 3 4
15(3)2
– – – – –
Area below the x-axis 3 4 4 2

12 ( 4 )
3 125 63
= 0
y dx =– – –
3


= 0
(8x – x – 15x) dx
2 3
16
=
[ 8x3 – x4 – 15x2 ]
3 4 2 3 3
=
0

=[
8(3)
3

(3)
4

2 ] [ 3
15(3)
3

8(0) 4

(0)
4
2 3 4

15(0)2
2 ] Total area = – | 634 | + 163
63 16
63 = +
=– 4 3
4 1
= 21 units2
12

63

F5 Mod A+ ADDM 3(50-77).indd 63 6/1/2021 11:02:30 AM


5 y
y = x2 + 4x Area below the x-axis
0
= –4
y dx
0
x = –4
(x2 + 4x) dx
−5 −4
[ x3 + 2x ]
O 3 0
= 2
–4

=[ + 2(0) ] – [ + 2(–4) ]
(0) 3
(–4) 2
3
2
Area above the x-axis 3 3
–4
= y dx 32
–5 =0–
–4 3
= (x + 4x) dx
2
–5
32
[ x3 + 2x ]
3 –4 =–
= 2 3

=[
(–4)
+ 2(–4) ] – [
3
(–5)
–5

+ 2(–5) ]2
3
2
Total area =
7
3
+ –
32
3 | |
3 3 7 32
32 25 = +
= – 3 3
3 3 = 13 units2
7
=
3

Smart Tip y
Area of the shaded region between a curve and the y-axis.
x = f(y)
Luas rantau berlorek antara lengkung dengan paksi-y. Area of the shaded region,
b
Area of the shaded region, Luas rantau berlorek,
a b
Luas rantau berlorek, A= x dy
a
b
A= a
x dy x
O

Exercise 11 Find the area of the shaded region between the curve and the y-axis.
Cari luas bagi rantau berlorek antara lengkung dengan paksi-y.
PL 3 Apply the understanding of integration to perform simple tasks.

Example 11 1 y
y
5
y2 = 25 − 5x
3 y2 = 9  x
1 x
x
O O
9
2
3

Given y2 = 25 – 5x
Solution 5x = 25 – y2
Given/ Diberi y2 = 9 – x 1
x = (25 – y2)
x = 9 – y2 5 2
y
x=5–
Area of the shaded region/ Luas rantau berlorek 5
3
= –2 x dy Area of the shaded region
5
=
3
(9 – y ) dy 2 = 1 x dy
–2

[ ]
y2
( )
5
y3 3 = 5− dy
= 9y – 1 5
3 –2
[ y 5
]
3

[
= 9(3) −
(3)3
3
– 9(–2) −
3 ] [
(–2)3
] = 5y −
15 1

= 18 – –
46
3 ( ) = 5(5) −[ (5)3
15
– 5(1) −
(1)3
15 ] [ ]
50 74
1 = –
= 33 units2/unit2 3 15
3
11
= 11 units2
15

64

F5 Mod A+ ADDM 3(50-77).indd 64 6/1/2021 11:02:31 AM


2 y 3 y

4 y2 = x − 2
2

x
O x
O
−1
y2 = 4(x − 1)

Given y2 = 4(x – 1) Given y2 = x – 2


y2 = 4x – 4 x = y2 + 2
4x = y2 + 4
y2 Area of the shaded region
x= +1 2
= –1 x dy
4
2
Area of the shaded region = –1
(y2 + 2) dy

[ y3 + 2y]
4 3 2
= 0 x dy =
4 y2
( )
–1
= + 1 dy
=[ + 2(2)] – [ ]
4 (2) 3
(–1) 3
0 + 2(–1)
[
y3
] 3 3
4
= +y
3 ( 3)
12 0 20 7
= – –
[
(4)
] [
(0)3
]
3
= + (4) – + (0)
12 12 = 9 units2
1
= 9 units2
3

Smart Tip
y
Area of the shaded region between a curve and a straight line.
y = f(x)
Luas rantau berlorek antara lengkung dengan satu garis lurus.
Area of the shaded region, y = g(x)
Luas rantau berlorek,
x
b b O
A= a
f(x) dx – a
g(x) dx a b

Area
Exercise 12 Find the area of the shaded region between the curve and the straight line. Luas ra
Cari luas bagi rantau berlorek antara lengkung dengan garis lurus.

Example 12
y
y = x + 2
4
y = x2 + 4
2 Alternative Method
x Area of the shaded region
2 O 2 Luas rantau berlorek
2 1
Solution = y dx –
(2)(2)
0 2
Area of the shaded region 2
= (–x2 + 4) dx – 2
0
Luas rantau berlorek
[
x3
]
2
=
2
[(–x2 + 4) – (–x + 2)] dx = – + 4x – 2
0 3 0

=
2
(–x2 + x + 2) dx
0 = – [
(2)3
3
+ 4(2) – 0 – 2 ]
[x3 x2
]
2
= – + + 2x 1
3 2 0 = 3 units2/ unit2
3
= – [
(2)3 (2)2
3
+
2
+ 2(2) – –
(0)3 (0)2
3
+
2 ] [
+ 2(0) ]
1
= 3 units2/ unit2
3

65

F5 Mod A+ ADDM 3(50-77).indd 65 6/1/2021 11:02:33 AM


1 y 2 y
y = 4x

3
y= 3
y = x2 + 3
x
O 1
y = −x2 + x + 3
x
O
Area of the shaded region
1
= 0 [(–x2 + x + 3) – 3] dx y = 4x 1
1 y=x +3
2
2
= (–x2 + x) dx
0 Subsitute 1 into 2
= – [
x3 x2 1
3
+
2 0 ] 4x = x2 + 3
x – 4x + 3 = 0
2

= – [
(1)3 (1)2
3
+
2
– –
(0)3 (0)2
3
+
2 ] [ ] (x – 1)(x – 3) = 0
1 x = 1 or x = 3
= unit 2
6
Area of the shaded region
3
= 1 [(4x) – (x2 + 3)] dx
3
= (–x2 + 4x – 3) dx
1

[
x3
]
3
= – + 2x2 – 3x
3 1

= – [
(3)3
3
+ 2(3)2 – 3(3) – –
(1)3
3 ] [
+ 2(1)2 – 3(1) ]
=0– –
4
3 ( )
Alternative Method
4
Alternative Method = units2 When x = 1, y = 4(1) = 4
3
Area of the shaded region When x = 3, y 4(3) = 12
1
= 0
y dx – 3(1) Area of the shaded region
1
= 0
(–x2 + x + 3) dx – 3 = 1 (4 + 12) × 2 – 31 y dx
2
[ x3 x2
]
1
= – + + 3x – 3 = 16 – 31 (x2 + 3) dx
3 2
[ ]
0
= 16 – x + 3x
3 3

=[ ] [ ]
(1)3 (1)2 (0)3 (0)2
– + + 3(1) – – + + 3(0) – 3 3 1
3 2 3 2
[ ] [
= 16 – (3) + 3(3) – (1) + 3(1) ]
3 3
19 3 3
= –0–3

=
1
6
unit2
(
= 16 – 18 – 10
3
)
6 = 4 units2
3

3 y
y= x + 4
Area of the shaded region
4 Alternative Method
= 1 [(6x – x2) – (x + 4)] dx
4
When x = 1, y = (1) + 4 = 5
= (–x2 + 5x – 4) dx
1 When x = 4, y = (4) + 4 = 8
y = 6x −x2

[
x 3 5x 2
]
4
Area of the shaded region
= – + – 4x
x 3 2 1 4 1
= y dx – (5 + 8) × 3
O
= – [
(4)3 5(4)2
3
+
2 ]
– 4(4) – 4
1 2
= 1 (6x –x2) dx – 19.5
y = 6x – x
[ ]
2
1 (1) 3
5(1) 2

y=x+4 2

3
+
2
– 4(1)
[
= 3x2 –
x3 4
3 1 ]
– 19.5
Subsitute 1 into 2
6x – x2 = x + 4
8
= – –
3 ( )11
6 [
= 3(4)2 –
(4)
3
3
] [
– 3(1)2–
(1)3
3
– 19.5 ]
x – 5x + 4 = 0
(
80 8
)
2
= 4.5 units2 = – – 19.5
(x – 1)(x – 4) = 0 3 3
x = 1 or x = 4 = 4.5 units2

66

F5 Mod A+ ADDM 3(50-77).indd 66 6/1/2021 11:02:35 AM


4 5 y
y x= 5
y2 = 5x 6
5
(2, 4)
2x = (y − 2)2
4
x
O
2
−5
x
O 2

Given y2 = 5x
y2 Given 2x = (y – 2)2
x= 5 (y – 2)2
x= 2
5
2x = (y  2)2
Area = 10  6 4
Area = +
5
5 4 2
(2, 4)
5
= 5(10) – x dy
–5 1 4
5 y 2 = (2)(2) + 2 x dy
= 50 – dy 2
–5 5
4 (y – 2)2

= 50 – [ ] y2+ 1 5
5(2 + 1) –5
=2+
2
1 4 2
2
dy

= 50 – [ ]
y3 5 =2+ (y – 4y + 4) dy
2 2
15 –5
[
1 y3
]
4

= 50 – [ ]
(5)3 (–5)3 =2+ – 2y2 + 4y
– 2 3 2
15 15
= 50 –
50 =2+ [(
1 (4)3
2 3 ) (
– 2(4)2 + 4(4) –
(2)3
3
– 2(2)2 + 4(2) )]
3
=2+ ( – )
1 1 16 8
= 33 units2 2 3 3
3
=2+ ( )
1 8
2 3
4
=2+
3
1
=3 units2
3

Smart Tip
The volume of the solid generated when the region bounded by the curve y = f(x),
f x), the x-axis, the line x = a and the line
f(
x = b is revolved through 360˚ about the x-axis is
Isi padu bongkah janaan apabila suatu rantau yang dibatasi oleh lengkung y = f(x), paksi-x, garis x = a dan garis x = b dikisarkan melalui 360˚ pada
paksi x ialah
y y = f(x)

b
x Vx = π a y2 dx
O a b

67

F5 Mod A+ ADDM 3(50-77).indd 67 6/1/2021 11:02:36 AM


Area of the shaded region,
Luas rantau berlorek,
Exercise 13 Find the volume generated for each of the following diagrams when the shaded region is
revolved through 360° about the x-axis.
Cari isi padu janaan bagi setiap rajah yang berikut apabila rantau berlorek diputarkan melalui 360˚ pada paksi-x.
PL 3 Apply the understanding of integration to perform simple tasks.

Example 13 1
y
y

y = 4 − x2
4
y= x
x
−2 −1 O 2
x
O 1 3
2
Vx = π y2 dx
Solution –1
2
3
Vx = π 1 y2 dx =π –1
(4 – x2)(4 – x2) dx

()
2
3 4 2 =π (16 – 8x2 + x4) dx
=π dx –1

[ ]
1 x
= π 16x – 8x + x
3 5 2


3 16
( ) dx 3 5 –1

[( ) ( )]
1 x2
= π 16(2) – 8(2) + (2) – 16(–1) – 8(–1) + (–1)
3 5 3 5

[ ]
= π – 16
3
3 5 3 5

[ ( )]
x 1 256 203
=π – –
[ ]
16 3
= –π 15 15
x 1 3
= 30 π units3
[
= –π 16 – 16
(3) (1) ] 5

[ ]
= –π – 32
3
32
= π units3/ unit3
3

2 3
y y

y = x2 − 5x 1
y= x − x

3 x
x O 1 5
−1 O 5

5
Vx = π
3
y2 dx Vx = π 1 y2 dx
–1

(x – 1x ) dx
3 5 2
=π –1
(x2 – 5x)2 dx =π
1
3

= π (x – 2 + 1 ) dx
=π –1
(x4 – 10x3 + 25x2) dx 5
2

[ ] x 2
= π x – 10x + 25x
5 4 3 3 1

= π[ – 2x – 1 ]
5 4 3 –1 x 3 5

3 x
[( ) (
1
= π (3) – 10(3) + 25(3) – (–1) – 10(–1) +
5 4 3 5 4

= π[( (5) – 2(5) – 1 ) – ( (1) – 2(1) – 1 )]


3 3
5 4 3 5 4
3 (5) 3 (1)
25(–1)3
)] = π[
15 ( 3 )]
3 472 8
– –
=π [
711
10
– – 331
30 ( )] = 34 2 π units3
15
2
= 82 π units3
15

68

F5 Mod A+ ADDM 3(50-77).indd 68 6/1/2021 11:02:38 AM


Smart Tip
The volume of the solid generated when the region bounded by the curve x = f(y),
f y), the y-axis, the line y = a and the line
f(
y = b is revolved through 360˚ about the y-axis is
Isi padu bongkah janaan apabila suatu rantau yang dibatasi oleh lengkung x = f(y), paksi-y, garis y = a dan garis y = b dikisarkan melalui 360˚
pada paksi y ialah
y

x = f(y)
b
Vy = π a
x2 dy
b

a
x
O

Exercise 14 Find the volume generated for each of the following diagrams when the shaded region is
revolved through 360° about the y-axis.
Cari isi padu janaan bagi setiap rajah yang berikut apabila rantau berlorek diputarkan melalui 360˚ pada paksi-y.
PL 3 Apply the understanding of integration to perform simple tasks.

Example 14
1 y
y2 = 8x
y

x = (y − 3)2
x
O
3
−2
x −4
O

Solution Given y2 = 8x
x= y
3 2
Vy = π 0 x2 dy
8
3
=π (y – 3)4 dy x2 = y
4
0
64
[ ]
= π (y – 3)
4+1 3

(1)(4 + 1) 0 –2
Vy = π x2 dy
[ ]
–4
= π (y – 3)
5 3

(64y ) dx
–2 4
5 0 =π
[ ]
–4
= π ((3) – 3) – ((0) – 3)
5 5

= π[ y ]
5 –2
5 5
5(64)
[ ( )]
–4
243
= π[ – (–4) ]
=π 0– – (–2) 5 5
5
243 320 320
= π[–
320 ( 320 )]
= π units3 32
5 – – 1 024

= 3.1π units3

69

F5 Mod A+ ADDM 3(50-77).indd 69 6/1/2021 11:02:39 AM


2 y 3 y
2
y= x − 2
x = y2 − 5y
5 −2

x
O
−1
x
O

5 2
Vy = π 0 x2 dy Given y = x – 2

= π 0 (y2 – 5y)2 dy
5
2
x = y + 2
5

= π 0 (y4 – 10y3 + 25y2) dy 4
x2 = (y + 2)2
[
= π y – 10y + 25y ]
5 4 3 5

5 4 3 0 2

[ ]
Vy = π x2 dy
= π (5) – 10(5) + 25(5) – 0
5 4 3
–1
5 4 3 42
= π 2 dy
1 –1 (y + 2)
= 104 π units 3
6 [
4 2
= π – y + 2
–1
]
4 2
= –π y + 2 [–1
]
= –π 4
[ – 4
(2) + 2 (–1) + 2 ]
= –π (1 – 4)
= 3π units3

4 y 5
y
y= x + 1
2

6 y = 6 − x2

3
2
x
O
x
O

Given y = x2 + 1 When x = 0, y = 6 – (0)2 = 6


x2 = y – 1 Given y = 6 – x2
x2 = 6 – y
6
Vy = π 3 x2 dy
6
6 Vy = π 2 x2 dy
= π 3
(y – 1) dy
6
= π (6 – y) dy
y2
[ ]
6 2
= π –y
2
[ y2 ]
2 6
3
= π 6y –
= π
(6)2
[(
– (6) –
(3)2
– (3)) ( )] = π[(6(6) –
2

2 ) (
– 6(2) – (2) )]
2 2 (6) 2 2

= π 12 –
3
2 ( ) = π (18 – 10)
2

= 10.5π units3
= 8π units 3

70

F5 Mod A+ ADDM 3(50-77).indd 70 6/1/2021 11:02:41 AM


Exercise 15 Calculate the volume generated for each of the following diagrams when the shaded region is
revolved through 360° about the y-axis.
Hitung isi padu janaan bagi setiap rajah yang berikut apabila rantau berlorek diputarkan melalui 360˚ pada
paksi-y.
PL 3 Apply the understanding of integration to perform simple tasks.

Example 15 1
y
y x= 2

x = 4y − y2 12
y= x

y= 2

x x
O 3 O

Solution
x=3 1 x=2 1
12
x = 4y – y2 2 y= 2
x
Substitute 1 2
into / Gantikan 1 ke dalam 2 Substitute 1 into 2
3 = 4y – y2 12
y – 4y + 3 = 0
y= =6
2
2
(y – 1)(y – 3) = 0
2 units
y = 1 or/ atau y = 3 6
PAK-21
Volume =  4 units
3 units/ unit

ACTIVITY
3 2
Volume =  2 units/ unit
Isi padu 1
6
3
Vy = π 2 x2 dy – πr2h
Vy = π x2 dy – πr2h
( 12y ) dy – (π × 2 × 4)
1 6 2


3
(4y – y ) dy – (π × 3 × 2)
2 2 2 =π 2

1 2

= π ( 144 ) dy – 16π
3 6
=π 1
(16y2 – 8y3 + y4) dy – 18π
2 y
2

[
= π 16y – 8y + y ]
3 4 5 3

= π[– 144 ] – 16π


– 18π 6
3 4 5 1
y 2

[
= π 16y – 2y4 + y ]
3 5 3

= π[(– ) – (– 1442 )] – 16π


– 18π 144
3 5 1
6
[(
= π 16(3) – 2(3)4 + (3) – )
3 5
= π[–24 – (–72)] – 16π
3 5
= 48π – 16π
(
16(1)3 – 2(1)4 + (1)5 – 18π
3 5 )] = 32π units3

=π 153
5( – 53
15
– 18π)
= 406 π – 18π
15
136
= π units3/ unit3
15

PAK-21 Trade a Problem

Steps/ Langkah-langkah:
1 Students are divided into groups of four.
Murid-murid dibahagikan kepada kumpulan yang terdiri daripada empat orang.
2 Each group is required to construct a question related to the volume of a region revolved at the x-axis or the
y-axis. Then, each group is required to trade the question with other groups.
Setiap kumpulan dikehendaki membina satu soalan mengenai isi padu suatu rantau yang dikisarkan pada paksi-x atau paksi-y.
Kemudiam, setiap kumpulan dikehendaki menukar soalan dengan kumpulan yang lain.
3 Teacher holds a discussion with students.
Guru mengadakan perbincangan bersama-sama murid.

71

F5 Mod A+ ADDM 3(50-77).indd 71 6/1/2021 11:02:42 AM


y
2 y
x=1 3 y= 4 x2 − y2 = 25

5
1
x
O
2 x+y=8
x
O
x + y = 8 4 units
5
x = 8 – y
x2 = (8 – y)2 Volume =  4 units

x2 = 64 – 16y + y2 1

5
Vy = π 1 x2 dy – πr2h
1 unit 5
5 = π 1 (25 + y2) dy – (π × 42 × 4)
Volume = 
[ ] – 64π
3 units
= π 25y + y
3 5

2 3 1

= π[(25(5) + (5) ) – (25(1) + (1) )] – 64π


3 3

3 3
= π(
3 )
5
Vy = π 2 x2 dy – πr2h 500 76
– – 64π
5 3
= π 2 (64 – 16y + y2) dy – (π × 12 × 3)
= 424 π – 64π
[
= π 64y – 8y + y ] – 3π
3 5
2 3
3 2 232
= π units3
= π[(64(5) – 8(5) + (5) ) – (64(2) – 8(2) + (2) )] – 3π
3 3
2 2 3
3 3
= π(
3 )
485 296
– – 3π
3
= 63π – 3π
= 60π units3

4 y x=3
5 y
y = x2 + 1
y = x2 − 9 y = 2x + 1
y=6
Q(2, 5)

x P
O x
O

When x = 0, y = 2(0) + 1 = 1
3 units [ P = (0, 1)
6

Volume =  6 units 2 units
5
0
Volume =  4 units
6
Vy = π 0 x2 dy – πr2h 1
6
= π 0 (y + 9) dy – (π × 32 × 6)
Vy = π 1 x2 dy – 1 πr2h
5
( )
[
= π y + 9y – 54π ] 3
2 6

2 0 = π 1 (y – 1) dy – 1 π × 22 × 4
5

3 ( )
(6)
[(
+ 9(6) – (0) + 9(0) – 54π ) ( )]
2 2

[ ]
= π y 2 5
16
2 2 = π –y – π
= π(72 – 0) – 54π 2 1 3

[( ) (
= π (5) – (5) – (1) – (1) – 16 π )]
2 2
= 72π – 54π
= 18π units3 2 2 3

2[ ( )]
= π 15 – – 1 – 16 π
2 3
16
= 8π – π
3
= 8 π units3
3

72

F5 Mod A+ ADDM 3(50-77).indd 72 6/1/2021 11:02:44 AM


3.4 Application of Integration / Aplikasi Pengamiran
Exercise 16 Solve the following problems. Applying

Selesaikan masalah yang berikut.


PL 4 Apply appropriate knowledge and skills of integration in the context of simple routine problem solving

1 Daud is painting a wall at a rate of (25 – t) m2 per 2 Dewi invests an amount of RM5 000. The expected
hour, where t is the number of hours after he started rate of increase in the investment, RMv, is given by
painting. Given the area of the wall is 132 m2, how dv
2 = 5y + 1 200, where y is the number of years of
long will it take for him to finish painting the wall in dy
a day? investment.
Daud mengecat sebuah dinding dengan kadar (25 – t) m2 per Dewi melabur sebanyak RM5 000. Kadar pertambahan
jam, dengan keadaan t ialah bilangan jam selepas dia mula dv
nilai pelaburan, RMv, diberi oleh 2 = 5y + 1 200, dengan
mengecat. Diberi luas dinding itu ialah 132 m2, berapa lamakah dy
masa yang diperlukan untuk dia habis mengecat dinding itu keadaan y ialah bilangan tahun melabur.
dalam sehari? 5y2
(a) Show that v = + 600y + 5 000.
4
dA 5y
2

Given = 25 – t such that A = area of the wall. Tunjukkan bahawa v = + 600 y + 5 000.
dt 4
(b) Find the minimum number of years required for
Time taken, j, to paint 132 m2 wall is
j the total amount of investment to be at least three
(25 – t) dt = 132
0 times the initial amount of investment.

[25t − 12 t ] Cari bilangan tahun minimum yang diperlukan supaya


j
2
= 132 jumlah nilai pelaburan menjadi sekurang-kurangnya tiga kali
0

[ ] [ ]
1 1 nilai pelaburan awal.
25j − j2 – 25(0) − (0)2 = 132
2 2
1 dv
25j − j2 = 132 (a) 2 = 5y + 1 200
2 dy
1 2 dv 5y
j – 25j + 132 = 0 = + 600
2 2 dy 2
j – 50j + 264 = 0
5y2
(j – 6)(j – 44) = 0 v = + 600y + c
4
j – 6 = 0 or j – 44 = 0 When y = 0, v = 5 000
j = 6 j = 44
5(0)2
5 000 = + 600(0) + c
Since j < 24, thus j = 6 hours. 4
c = 5 000
5y2
[ v= + 600y + 5 000
4
(b) Total amount of investment triple
= 5 000 × 3
= 15 000
5y2
+ 600y + 5 000 = 15 000
4
5y2
+ 600y – 10 000 = 0
4
5y + 2 400y – 40 000 = 0
2

–b b2 – 4ac
y =
2a
(2 400)2 – 4(5)(–40 000)
y = –(2 400)
2(5)
y = 16.125 or y = −496

Since y > 0, the minimum number of years


required for the total amount of investment
to be at least three times the initial amount of
investment is 17.

73

F5 Mod A+ ADDM 3(50-77).indd 73 6/1/2021 11:02:45 AM


Review 3
Paper 1

Section A (a) Find the value of m and of n.


8 5 Cari nilai m dan nilai n.
1 Given 4
f(y) dy = 2 , find 6 6
(b) Given f(x) dx = 11, find the value of f(x) dx.
8 5 –4 1
Diberi f(y) dy = , cari Diberi
6
f(x) dx = 11, cari nilai
6
f(x) dx.
4 2 –4 1
4
(a) 8
6f(y) dy, [4 marks/markah]
8
(b) the value of m such that 4
[m – f(y)] dy = 9.5.
m
8 4 (a) Given 2
(4x – 7) dx = 10, find the value of m.
nilai m dengan keadaan 4
[m – f(y)] dy = 9.5. m
Diberi (4x – 7) dx = 10, cari nilai m.
[5 marks/markah] 2
[3 marks/markah]
2 (a) Diagram 1 shows a shaded region bounded by the (b) Diagram 3 shows a shaded region bounded by the
curve y = g(x), y-axis and the straight line OP. curve y = 3x – x 2 and the x-axis.
Rajah 3 menunjukkan suatu rantau berlorek yang dibatasi
Rajah 1 menunjukkan satu rantau berlorek yang dibatasi
oleh lengkung y = 3x – x2 dan paksi-x.
oleh lengkung y = g(x), paksi-y dan garis lurus OP.
y
y
y = 3x − x2
y = g(x)

P(5, 1)
x
O
x
O Diagram 3/ Rajah 3
Diagram 1/ Rajah 1 Find the volume generated, in terms of π, when
5
the shaded region is revolved through 360°
It is given that 0 g(x) dx = 9, find the area, in about the x-axis.
units2, of the shaded region. Cari isi padu janaan, dalam sebutan π, apabila rantau
5
Diberi bahawa 0 g(x) dx = 9, cari luas, dalam unit2, bagi berlorek itu diputarkan melalui 360° pada paksi-x.
rantau berlorek. [3 marks/markah]
[2 marks/markah]
(b) The graph of y = h(x) passes through the point 5 Diagram 4 shows a part of the curve y = x2 + 1 intersects
d with the straight line x + y = 7 at point F.
(2, −16). Given [h(x)] = 3x2 + x – 26, find h(x) in
dx Rajah 4 menunjukkan sebahagian daripada lengkung y = x2 + 1
terms of x. bersilang dengan garis lurus x + y = 7 pada titik F.
Graf y = h(x) melalui titik (2, −16). Diberi y
d
[h(x)] = 3x2 + x – 26, cari h(x) dalam sebutan x. y = x2 + 1
dx
[3 marks/markah]
F
3 Diagram 2 shows a curve y = f(x). x + y= 7
Rajah 2 menunjukkan lengkung y = f(x).
E
y x
O G
y = f(x) Diagram 4/ Rajah 4
Find/ Cari
(a) the coordinates of point F,
koordinat titik F,
x
4 O 1 6 [2 marks/markah]
(b) the area of the shaded region.
Diagram 2/ Rajah 2
luas bagi rantau berlorek.
The area of the shaded region is given by [3 marks/markah]
n 13
f(x) dx = .
m 4
n 13
Luas bagi rantau berlorek diberi oleh f(x) dx = .
m 4

74

F5 Mod A+ ADDM 3(50-77).indd 74 6/1/2021 11:02:46 AM


Section B (b) Diagram 6 shows the curve y = 8x – x2 and the
6 (a) Diagram 5 shows the curve y = h(x). The straight straight line y = 7.
line y = 5 is a tangent to the curve. Rajah 6 menunjukkan lengkung y = 8x – x2 dan garis lurus
Rajah 5 menunjukkan lengkung y = h(x). Garis lurus y = 5 y = 7.
ialah tangen kepada lengkung itu. y

y
y = h(x) y= 7

y = 8x − x2

y= 5 x
O
x
O Diagram 6/ Rajah 6
Diagram 5/ Rajah 5
Find the area, in units2, of the shaded region.
Given the gradient function of the curve is Cari luas, dalam unit2, bagi rantau berlorek.
dy [4 marks/markah]
= 2x – 4, find the equation of the curve.
dx
Diberi fungsi kecerunan bagi lengkung itu ialah
dy
= 2x – 4, cari persamaan bagi lengkung itu.
dx
[4 marks/markah]

1 Given
Paper 2
Section A
1 Diagram 1 shows a part of the curve y = 6x – x2 and the straight line 2x + y = 16.
Rajah 1 menunjukkan sebahagian daripada lengkung y = 6x – x2 dan garis lurus 2x + y = 16.

y 2x + y = 16
K

y = 6x − x2
x
O 2 4

Diagram 1/ Rajah 1
(a) Find the coordinates of point K and of point L,
Cari koordinat titik K dan titik L,
[3 marks/markah]
(b) Calculate the area, in units2, of the shaded region.
Hitung luas, dalam unit2, bagi rantau yang berlorek.
[4 marks/markah]

Section B
2 Diagram 2 shows the curve y = 16 − x2 and the straight line FH is a tangent to the curve at point G.
Rajah 2 menunjukkan lengkung y = 16 − x2 dan garis lurus FH ialah tangen kepada lengkung itu pada titik G.
y

G
y = 16  x2
p

x
O 9 H
Diagram 2/ Rajah 2
Given the gradient of the straight line FH is −2. Find
Diberi kecerunan garis lurus FH ialah −2. Cari
(a) the coordinates of point G,
koordinat titik G,
[2 marks/markah]

75

F5 Mod A+ ADDM 3(50-77).indd 75 6/1/2021 11:02:47 AM


(b) the area, in units2, of the shaded region,
luas, dalam unit2, bagi rantau yang berlorek,
[5 marks/markah]
(c) the value of p if the region bounded by the curve y = 16 – x2, the x-axis and the straight line y = p is revolved
through 180° about the y-axis, the volume generated is 78π units3.
nilai p jika rantau yang dibatasi oleh lengkung y = 16 – x2, paksi-x dan garis lurus y = p diputarkan melalui 180° pada paksi-y, isi
padu janaan ialah 78π unit3.
[3 marks/markah]

3 Diagram 3 shows the straight line DE intersects the curve x = y2 + 1 at point E.


Rajah 3 menunjukkan garis lurus DE bersilang dengan lengkung x = y2 + 1 pada titik E.
y

D
y= 7 − x

E(5, 2) x = y2 + 1
2

x
O F(m, 0) 5

Diagram 3/ Rajah 3
Find/ Cari
(a) the value of m,
nilai m,
[2 marks/markah]
(b) the area, in units2, of the shaded region,
luas, dalam unit2, bagi rantau yang berlorek,
[5 marks/markah]
(c) the volume generated, in terms of π, when the region bounded by the curve x = y2 + 1, the straight line x = 5 and
the x-axis is revolved through 360° about the x-axis.
isi padu janaan, dalam sebutan π, apabila rantau yang dibatasi oleh lengkung x = y2 + 1, garis lurus x = 5 dan paksi-x diputarkan
melalui 360° pada paksi-x.
[3 marks/markah]

H O TS Zo n e
1 Diagram 1 shows a circle with centre O and radius r units.
Rajah 1 menunjukkan sebuah bulatan berpusat O dan berjejari r unit.
y

x
O

Diagram 1/ Rajah 1
(a) Show that the equation of the circle is x + y2 = r2. 2

Tunjukkan bahawa persamaan bulatan itu ialah x2 + y2 = r2.


(b) Find the volume generated, in terms of π and r, when the shaded region is revolved through 360° about the
x-axis. Applying

Cari isi padu janaan, dalam sebutan π dan r, apabila rantau berlorek diputarkan melalui 360° pada paksi-x.

76

F5 Mod A+ ADDM 3(50-77).indd 76 6/1/2021 11:02:47 AM


2 Diagram 2 shows a straight line PQ on a Cartesian plane.
Rajah 2 menunjukkan garis lurus PQ pada satah Cartes.
y

P(0, h)

x
O Q(r, 0)

Diagram 2/ Rajah 2
(a) Express the equation of PQ in the form of x = my + c.
Ungkapkan persamaan PQ dalam bentuk x = my + c.
(b) Calculate the volume generated, in terms of π, r and h, when the shaded region is revolved through 360° about
the y-axis. Applying

Hitung isi padu janaan, dalam sebutan π, r dan h, apabila rantau berlorek diputarkan melalui 360° pada paksi-y.

3 Diagram 3 shows a shaded region bounded by the curve y = x2 + 2, the straight line y = k and the y-axis.
Rajah 3 menunjukkan rantau berlorek yang dibatasi oleh lengkung y = x2 + 2, garis lurus y = k dan paksi-y.
y
y = x2 + 2

y= k

x
O

Diagram 3/ Rajah 3
Given the volume generated when the shaded region is revolved through 360° about the y-axis is 18π units3.
Find the value of k. Applying

Diberi isi padu janaan apabila rantau berlorek diputarkan melalui 360° pada paksi-y ialah 18π unit3. Cari nilai k.

77

F5 Mod A+ ADDM 3(50-77).indd 77 6/1/2021 11:02:48 AM


Learning Area: Statistics

4 Permutation and Combination


Pilih Atur dan Gabungan
4.1 Permutation / Pilih Atur

Smart Tip
Multiplication rule is defined as the number of ways for two or more events can occur. If a certain event can occur in m
ways and another event can occur in n ways, then the number of ways for both events can occur is m × n ways.
Petua pendaraban didefinisikan sebagai bilangan cara bagi dua atau lebih peristiwa boleh berlaku. Jika suatu peristiwa boleh berlaku dalam m cara dan
suatu peristiwa lain boleh berlaku dalam n cara, maka bilangan cara kedua-dua peristiwa boleh berlaku ialah m × n cara.

Exercise 1 Solve each of the following.


Selesaikan setiap yang berikut.
PL 2 Demonstrate the understanding of permutation and combination.

Example 1
1 In a chess competition, team X has 10 players and
An undergraduate from Jengka wishes to go back team Y has 6 players. Find the number of ways a
to his hometown at Kota Kinabalu. There are 5 bus single game can be held.
Dalam suatu pertandingan catur, pasukan X mempunyai
companies that provide transport from Jengka to
10 orang pemain dan pasukan Y mempunyai 6 orang pemain.
Kuala Lumpur and 3 airlines that provide flight
Cari bilangan cara perlawanan individu boleh diadakan.
from Kuala Lumpur to Kota Kinabalu. Find the
number of ways he can go home.
Seorang mahasiswa dari Jengka ingin pulang ke kampungnya di Number of ways = 10 × 6 = 60
Kota Kinabalu. Terdapat 5 buah syarikat bas yang menyediakan
perjalanan dari Jengka ke Kuala Lumpur dan 3 buah syarikat
penerbangan yang menyediakan penerbangan dari Kuala
Lumpur ke Kota Kinabalu. Cari bilangan cara dia boleh pulang
ke kampungnya.

Solution
Number of ways/ Bilangan cara
=5×3
= 15

2 If a man has 4 trousers and 3 shirts, how many ways 3 How many ways to form a pair of mixed double
can he wear the trousers and the shirts? player from 5 female players and 7 male players?
Jika seorang lelaki mempunyai 4 helai seluar panjang dan 3 helai Berapakah bilangan cara untuk membentuk sepasang pemain
baju, berapakah bilangan cara dia boleh memakai seluar panjang beregu campuran daripada 5 orang pemain perempuan dan 7
dan baju itu? orang pemain lelaki?

Number of ways = 4 × 3 = 12 Number of ways = 5 × 7 = 35

78

F5 Mod A+ ADDM 4(78-90).indd 78 6/1/2021 11:08:51 AM


4 A woman has 5 handbags, 4 shawls and 7 skirts. 5 The diagram below shows the roads that connecting
How many ways can she match the three items at four towns, A, B, C and D.
one time? Rajah di bawah menunjukkan jalan yang menghubungkan
Seorang wanita mempunyai 5 buah beg tangan, 4 helai empat buah bandar, A, B, C dan D.
selendang dan 7 helai skirt. Berapakah bilangan cara dia boleh
padankan ketiga-tiga item itu pada satu masa?

A B C D
Number of ways = 5 × 4 × 7 = 140

Khalid wishes to drive from town A to town D. Find


the number of ways that he can travel from town A
to town D.
Khalid ingin memandu dari bandar A ke bandar D. Cari
bilangan cara perjalanan dari bandar A ke bandar D yang dia
boleh lalui.

Number of ways = 2 × 4 × 4 = 32 PAK-21

VIDEO
PAK-21
Smart Tip

ACTIVITY
1 The number of permutations of n different objects is given by n!, such that
Bilangan pilih atur bagi n objek yang berbeza diberi oleh n!, dengan keadaan
n! = n × (n – 1) × (n – 2) ... × 3 × 2 × 1
2 n! is read as n factorial.
n! dibaca sebagai n faktorial.
3 0! = 1.

PAK-21 Simultaneous Round Table


Calculator Corner

Find the value of 8!./ Cari nilai bagi 8!. Steps/ Langkah-langkah:
1 Students are divided into groups of four.
By using scientific calculator Casio fx-570EX. Murid-murid dibahagikan kepada kumpulan yang terdiri daripada
Dengan menggunakan kalkulator saintifik Casio
empat orang.
fx-570EX.
2 Teacher gives a word to each group.
Step 1: Press ‘MENU’ and find 1: Calculate Guru memberikan satu perkaatan kepada setiap kumpulan.
(normal display), press ‘ = ’. Example/ Contoh : “CUTE”
Langkah 1: Tekan ‘MENU’ dan cari 1: Calculate
3 Each group is required to arrange all the letters from the
(paparan normal), tekan ‘ = ’.
word without repetition.
Step 2: Key in the values and press ‘ = ’ to get
Setiap kumpulan dikehendaki menyusun semua huruf dalam
the answer.
perkataan itu tanpa ulangan huruf.
Langkah 2: Masukkan nilai dan tekan ‘ = ’ untuk
mendapatkan jawapan. 4 In the group, the first student write the first arrangement of
–1 the letters in a paper. Then, the first student passes the
8 SHIFT x =
paper to the other students in the group in clockwise rotation
Answer/ Jawapan: 40 320 so that each member of the group can write the answers.
Dalam kumpulan, murid pertama menulis susunan huruf di atas
kertas. Kemudian, murid pertama mengedarkan kertas kepada
murid seterusnya dalam kumpulan mengikut susunan jam supaya
setiap ahli kumpulan dapat menulis jawapan.

5 Students are required to repeat the process until there is no


Video more possibilities of the arrangement.
Murid-murid dikehendaki mengulang proses itu sehingga tiada lagi
Scan or visit https://youtu.be/BrG24qYV_Vw
susunan huruf yang mungkin.
to watch a video for more explanation and
example of permutations of n different objects. 6 Teacher holds a discussion with students.
For educational purposes only Guru mengadakan perbincangan bersama-sama murid.

79

F5 Mod A+ ADDM 4(78-90).indd 79 6/1/2021 11:08:52 AM


Exercise 2 Without using a calculator, find the value for each of the following.
Tanpa menggunakan kalkulator, cari nilai bagi setiap yang berikut.
PL 2 Demonstrate the understanding of permutation and combination.

Example 2 1 2! 4! 2 5! 0!
8!
(a) 3! (b) 6!
2! 4! 5! 0!
Solution = (2 × 1) × (4 × 3 × 2 × 1) = (5 × 4 × 3 × 2 × 1) × 1
8! 8 × 7 × 6! = 2 × 24 = 120
(a) 3! = 3 × 2 × 1 = 6 (b) 6! = 6! = 48
=8×7
= 56

6! 8! 7! 2! 9! 5!
3 4 5 6
3! 6! 2! 5! 7! 4!
6! 6 × 5 × 4 × 3! 8! 8 × 7 × 6!
3! = 3! 6! 2! = 6! × 2 × 1 7! 2! 9! 5!
=6×5×4 56 5! 7! 4!
= 120 = 2 7 × 6 × 5! × 2 × 1 9 × 8 × 7! × 5 × 4!
= =
= 28 5! 7! × 4!
=7×6×2×1 =9×8×5
= 84 = 360

Exercise 3 Solve each of the following.


Selesaikan setiap yang berikut.
PL 2 Demonstrate the understanding of permutation and combination.

Example 3 1 Find the number of ways to 2 Find the number of ways to


arrange 6 books on a bookshelf. arrange all the letters from
Find the number of ways to
Cari bilangan cara menyusun 6 buah the word ‘WATER’ without
arrange all the letters from
buku di atas rak buku. repetition.
the word ‘FACTORS’ without
Cari bilangan cara menyusun semua
repetition.
n=6 huruf daripada perkataan ‘WATER’
Cari bilangan cara menyusun semua
Number of ways = 6! tanpa ulangan.
huruf daripada perkataan ‘FACTORS’
= 720
tanpa ulangan.
n=5
Number of ways = 5!
Solution
= 120
Number of letters
Bilangan huruf, n = 7
Number of ways/ Bilangan cara
= 7!
= 5 040

3 Find the number ways to form 4-digit numbers 4 Find the number of different ways to arrange all the
using the digits 1, 3, 4 and 5 if no digit can be letters in the word ‘FLOWER’.
repeated. Cari bilangan cara berlainan menyusun semua huruf daripada
Cari bilangan cara membentuk nombor 4 digit menggunakan perkataan ‘FLOWER’.
digit 1, 3, 4 dan 5 jika digit-digit itu tidak berulang.
n=6
n=4 Number of ways = 6!
Number of ways = 4! = 720
= 24

80

F5 Mod A+ ADDM 4(78-90).indd 80 6/1/2021 11:08:52 AM


Smart Tip
Number of permutations of n different objects taken from r objects at a time is given by
Bilangan pilih atur bagi n objek yang berbeza diambil r objek pada satu masa diberi oleh
n n!
Pr = , where/dengan keadaan r  n
(n – r)!

Calculator Corner
7 7
Find the value of P4./ Cari nilai P4.

By using scientific calculator Casio fx-570EX.


Video
Dengan menggunakan kalkulator saintifik Casio fx-570EX.
Scan or visit https://youtu.
Step 1: Press ‘MENU’ and find 1: Calculate (normal display), press ‘ = ’. be/0NAASclUm4k to watch a
Langkah 1: Tekan ‘MENU’ dan cari 1: Calculate (paparan normal), tekan ‘ = ’. video for more explanation
of permutations and
Step 2: Key in the values and press ‘ = ’ to get the answer. combinations. PAK-21
Langkah 2: Masukkan nilai dan tekan ‘ = ’ untuk mendapatkan jawapan.

VIDEO
For educational purposes only

7 SHIFT × 4 =

Answer/ Jawapan: 840

Exercise 4 Solve each of the following.


Selesaikan setiap yang berikut.
PL 2 Demonstrate the understanding of permutation and combination.
PL 3 Apply the understanding of permutation and combination to perform simple tasks.

Example 4 1 (a) Without using a calculator, find the value of


5 Tanpa menggunakan kalkulator, cari nilai
(a) Without using a calculator, find the value of P2. 7 9
5 (i) P3, (ii) P2.
Tanpa menggunakan kalkulator, cari nilai P2. n
(b) Determine the number of ways to form 3-digit (b) Show that Pn = n!.
n
numbers from the digits 2, 3, 4, 6, 8 and 9 Tunjukkan bahawa Pn = n!.
without repetition.
Tentukan bilangan cara untuk membentuk nombor 3 digit (a) (i) 7P3 (ii)
9
P2
daripada digit 2, 3, 4, 6, 8 dan 9 tanpa ulangan. 7! 9!
= (7 – 3)! = (9 – 2)!
Solution 7! 9!
= 4! =
5 5! 5 5! 7!
(a) P2 = (5 – 2)! (b) P3 = (5 – 3)! 7 × 6 × 5 × 4! 9 × 8 × 7!
= =
5! 5! 4! 7!
= 3! = 2! =7×6×5 =9×8
5 × 4 × 3! 5 × 4 × 3 × 2! = 210 = 72
= 3! = 2!
n
=5×4 =5×4×3 (b) (i) Pn
= 20 = 60 n!
= (n – n)!
n!
= 0!
n!
= 1
= n!

81

F5 Mod A+ ADDM 4(78-90).indd 81 6/1/2021 11:08:52 AM


2 A 3-digit code is to be formed from the following 3 Find the number of ways to form 2-digit numbers
characters: using the following digits if repetition of digits is
Satu kod tiga digit akan dibentuk daripada aksara berikut: not allowed.
Cari bilangan cara untuk membentuk nombor 2 digit
F G H 7 8 9 menggunakan digit-digit berikut jika ulangan digit tidak
Find the number of ways the code can be formed. dibenarkan.
Cari bilangan cara kod itu dapat dibentuk. 2 3 4 6 9

6 6! 5 5!
P3 = P2 =
(6 – 3)! (5 – 2)!
= 120 = 20

Smart Tip
The number of permutation of n objects involving identical objects is given by
Bilangan pilih atur bagi n objek yang melibatkan objek secaman diberi oleh
n!
P=
a! b! c! ...
where a, b and c, … is the number of objects for each identical object.
dengan keadaan a, b dan c, … ialah bilangan objek bagi setiap objek secaman.

Exercise 5 Find the number of ways to arrange all the letters from the following words.
Cari bilangan cara menyusun semua huruf daripada perkataan yang berikut.
PL 2 Demonstrate the understanding of permutation and combination.

Example 5 1 EMCEE 2 LITTLE


(a) TREES (b) CAFETERIA
5 letters, 3E 6 letters, 2L, 2T
Number of ways Number of ways
Solution 5! 6!
(a) 5 letters, 2E (b) 9 letters, 2A, 2E = =
3! 2! 2!
5 huruf, 2E 9 huruf, 2A, 2E 120
Number of ways Number of ways = 6 = 180
Bilangan cara Bilangan cara = 20
5! 9!
= = 2! 2!
2!
120
= = 90 720
2
= 20

3 COCOON 4 SELFLESS 5 EYELEVEL

6 letters, 2C, 3O 8 letters, 3S, 2E, 2L 8 letters, 4E, 2L


Number of ways Number of ways Number of ways
6! 8! 8!
= = =
2! 3! 3! 2! 2! 4! 2!
= 60 = 1 680 = 840

82

F5 Mod A+ ADDM 4(78-90).indd 82 6/1/2021 11:08:53 AM


Exercise 6 Solve each of the following.
Selesaikan setiap yang berikut.
PL 3 Apply the understanding of permutation and combination to perform simple tasks.

Example 6
1 Find the number of ways 5-digit numbers can be
Find the number of ways all the letters from the formed from the digits below if the digits 5 and 7
word ‘FORMULA’ can be arranged with the are always separated and no digits can be repeated.
Cari bilangan cara nombor 5-digit yang boleh dibentuk
condition that the arrangements start with a vowel.
daripada digit-digit di bawah jika digit 5 dan digit 7 sentiasa
Cari bilangan cara semua huruf daripada perkataan
berasingan dan tiada digit yang berulang.
‘FORMULA’ boleh disusun dengan syarat susunan bermula
dengan huruf vokal. 4 5 6 7 8
Solution Number of ways when digits 5 and 7 are always
The vowels are O, U, A together
Huruf vokal ialah O, U, A = 4! × 2
O = 48
6 × 5 × 4 × 3 × 2 × 1 = 6! Number of ways when digits 5 and 7 are always
U separated
= 5! – 48
6 × 5 × 4 × 3 × 2 × 1 = 6! = 120 – 48
A = 72
6 × 5 × 4 × 3 × 2 × 1 = 6!

Number of ways/ Bilangan cara


= 6! + 6! + 6!
= 720 + 720 + 720
= 2 160

Alternative Method
Number of ways
Bilangan cara
3 6
= P1 × P6
= 2 160

2 There are 6 books on a bookshelf. What is the 3 Find the number of ways 4-digit even numbers
number of ways to arrange the books if two of the can be formed from the digits 3, 4, 5 and 7 without
bestselling books must be arranged side by side? repetition.
Terdapat 6 buah buku di atas rak buku. Berapakah bilangan Cari bilangan cara nombor genap 4 digit yang boleh dibentuk
cara untuk menyusun buku-buku itu jika dua buah buku yang daripada digit-digit 3, 4, 5 dan 7 tanpa ulangan.
terlaris mesti disusun bersebelahan?
Even number must end with digit 4.
5! × 2! = 120 × 2 Number of ways for three digits before digit 4
= 240
4
3×2×1=6

83

F5 Mod A+ ADDM 4(78-90).indd 83 6/1/2021 11:08:53 AM


4 The diagram below shows 2 letter cards and 4 5 Find the number of ways 4-digit odd numbers can
number cards. be formed using the digits 4, 5, 7 and 9 without
Rajah di bawah menunjukkan 2 kad huruf dan 4 kad nombor. repetition.
Cari bilangan cara nombor ganjil 4 digit yang boleh dibentuk
3 P 4 Q 5 6 menggunakan digit-digit 4, 5, 7 dan 9 tanpa ulangan.
Find the number of codes that can be formed using
all the characters if each code must start with a letter Odd numbers must end with digits 5, 7 and 9.
and without repeating any of the characters. Number of ways for three digits with the last
Cari bilangan kod yang dapat dibentuk menggunakan semua digit ‘5’
aksara itu jika setiap kod mesti bermula dengan huruf dan tiada
aksara yang berulang. 5
3×2×1=6
Number of ways for 5 characters with the first letter
‘P’ Number of ways for three digits with the last
P digit ‘7’

5 × 4 × 3 × 2 × 1 = 120 7
3×2×1=6
Number of ways for 5 characters with the first letter
‘Q’ Number of ways for three digits with the last
Q digit ‘9’
5 × 4 × 3 × 2 × 1 = 120 9
3×2×1=6
Number of ways
= 120 + 120
Number of ways
= 240
=6+6+6
= 18

Exercise 7 Solve each of the following.


Selesaikan setiap yang berikut.
PL 4 Apply appropriate knowledge and skills of permutation and combination in the context of simple routine problem solving.

Example 7 Example 8
The diagram shows eight letter cards. Find the number of ways to arrange eight
Rajah menunjukkan lapan kad huruf. shareholders to sit at a round table in a meeting.
Cari bilangan cara menyusun lapan orang pemegang
F E S T I V A L saham untuk duduk di sebuah meja bulat dalam suatu
Find/ Cari mesyuarat.
(a) the number of ways to arrange four letter cards
from all the cards, Solution
bilangan cara menyusun 4 kad huruf daripada semua kad itu, Number of ways
(b) the number of arrangements if the arrangement Bilangan cara
of the 4 letter cards begins with a vowel. = (8 – 1)!
bilangan susunan jika susunan 4 kad huruf itu bermula = 7!
dengan huruf vokal. = 5 040

Solution
8
(a) P4 = 1 680
(b) Vowel/Huruf vokal: E, I, A
1 7
E = P1 × P3 = 210
1 7
Smart Tip
I = P1 × P3 = 210
1 7 Number of permutations of n different
A = P1 × P3 = 210 objects in a circle arrangement = (n – 1)!
Number of arrangements/Bilangan susunan Bilangan pilih atur bagi n objek yang berbeza
dalam susunan bulatan = (n – 1)!
= 210 + 210 + 210
= 630

84

F5 Mod A+ ADDM 4(78-90).indd 84 6/1/2021 11:08:54 AM


1 5 boys and 2 girls are to be seated in a row. Find the 2 4 male teachers and 3 female teachers are to be
number of ways they can be seated if 2 girls want to arranged in a row for a photograph session. Find
sit together. the number of ways they can be seated if the 3
5 orang murid lelaki dan 2 orang murid perempuan ditempatkan female teachers must be seated together.
sebaris. Cari bilangan cara mereka boleh duduk jika 2 orang 4 orang guru lelaki dan 3 orang guru perempuan disusun
murid perempuan itu ingin duduk bersebelahan. sebaris untuk sesi fotografi. Cari bilangan cara mereka
boleh duduk jika 3 orang guru perempuan itu mesti duduk
bersebelahan.

G G B B B B B
Number of ways = 2! × 5! × 6 F F F M M M M
= 1 440 Number of ways = 3! × 4! × 5
= 720

3 Ali has 4 watermelons, 5 papayas and 2 apples. Find 4 4 rattan chairs and 5 wooden chairs are required
the number of ways if he wants to arrange all the to be arranged in a row. Find the number of
fruits in a row and the fruits of the same kind must arrangements that can be formed if 3 rattan chairs
be placed together. must be arranged together.
Ali mempunyai 4 biji tembikai, 5 biji betik dan 2 biji epal. Cari 4 buah kerusi rotan dan 5 buah kerusi kayu perlu disusun
bilangan cara jika dia ingin menyusun semua buah itu sebaris sebaris. Cari bilangan susunan jika 3 buah kerusi rotan mesti
dan buah yang sama jenis diletakkan bersebelahan. disusun bersebelahan.

Number of ways = (4! × 5! × 2!) × 3! R R R


= 34 560 4
Number of ways = P3 × P6
6

= 17 280

5 Find the number of ways to arrange five students to 6 What is the number of ways to arrange seven
sit at a round table for the group work. teachers to sit at a round table in a meeting?
Cari bilangan cara menyusun lima orang murid untuk duduk Berapakah bilangan cara menyusun tujuh orang guru untuk
di sebuah meja bulat untuk kerja berkumpulan. duduk di sebuah meja bulat dalam suatu mesyuarat?

Number of ways Number of ways


= (5 – 1)! = (7 – 1)!
= 4! = 6!
= 24 = 720

4.2 Combination / Gabungan

Smart Tip
1 A combination is a selection of several objects from a set such that the arrangement of the objects is not taken into
consideration. The number of combinations of r objects chosen from n different objects at a time is given by
Gabungan ialah pemilihan beberapa objek daripada suatu set dengan keadaan susunan objek itu tidak diambil kira. Bilangan gabungan bagi r objek
yang dipilih daripada n objek yang berbeza pada satu masa diberi oleh
n n!
Cr =
r!(n – r)!
n n
2 The relationship between permutation, Pr and combination, Cr is
n n
Hubungan antara pilih atur, Pr dan gabungan, Cr ialah
n n
Cr × r! = Pr

85

F5 Mod A+ ADDM 4(78-90).indd 85 6/1/2021 11:08:54 AM


Calculator Corner
8 8
Find the value of C3./ Cari nilai C3.

By using scientific calculator Casio fx-570EX.


Dengan menggunakan kalkulator saintifik Casio fx-570EX.

Step 1: Press ‘MENU’ and find 1: Calculate (normal display), press ‘ = ’.


Langkah 1: Tekan ‘MENU’ dan cari 1: Calculate (paparan normal), tekan ‘ = ’.
Step 2: Key in the values and press ‘ = ’ to get the answer.
Langkah 2: Masukkan nilai dan tekan ‘ = ’ untuk mendapatkan jawapan.

8 SHIFT  3 =
Answer/ Jawapan: 56

Exercise 8 Mark () for the situation that involves combination.


Tandakan () pada situasi yang melibatkan gabungan.
PL 2 Demonstrate the understanding of permutation and combination.

Situation Answer
Situasi Jawapan

Constructing a password from given digits.


1
Membina kata laluan daripada digit-digit yang diberi.

Choosing 5 committee members from 10 students.


2 
Memilih 5 ahli jawatankuasa daripada 10 orang murid.

Forming 4-digit number from 9 digits.


3
Membentuk nombor 4 digit daripada 9 digit.

Choosing 4 representatives from 20 students for debate competition.


4 
Memilih 4 orang wakil daripada 20 orang murid untuk pertandingan perbahasan.

Arranging the students to sit on a long bench.


5
Menyusun murid untuk duduk di sebuah bangku panjang.

Arranging the sitting positions at a round table.


6
Menyusun tempat duduk di sebuah meja bulat.

Mixing at least two colours.


7 
Mencampurkan sekurang-kurangnya dua warna.

Exercise 9 Solve each of the following.


Selesaikan setiap yang berikut.
PL 3 Apply the understanding of permutation and combination to perform simple tasks.

Example 9 1 Without using a calculator, find 1 n


n
Show that Cr =
r! × Pr.
6 2
Without using a calculator, find the value of C4.
n 1 n
8 Tanpa menggunakan kalkulator, cari Tunjukkan bahawa Cr = × Pr.
the value of C6. 6
r!
Tanpa menggunakan kalkulator, cari nilai C4.
8
nilai C6. n n!
Cr = r!(n – r)!
6 6!
C4 = 4!(6 – 4)! 1 n!
Solution = r! × (n – r)!
8 8! 6!
(a) C6 = 6!(8 – 6)! = 4! 2! 1 n
= r! × Pr
8! 6 × 5 × 4!
= 6! 2! = 4! 2!
8 × 7 × 6! 6×5
= 6! 2! =
2×1
8×7 = 15
= 2 × 1 = 28

86

F5 Mod A+ ADDM 4(78-90).indd 86 6/1/2021 11:08:54 AM


9 9 n n 12 12
3 If Cm = C5, find the value of m. 4 Given Cr = C3, express n in 5 Given C7 = Cm, determine the
9 9
Jika Cm = C5, cari nilai m. terms of r. value of m.
n n 12 12
Diberi Cr = C3, ungkapkan n dalam Diberi C7 = Cm, tentukan nilai m.
m=9–5 sebutan r.
=4 m = 12 – 7
n = r + 3, since =5
r+3 (r + 3)! (r + 3)!
Cr = r!(r + 3 – r)! = r! 3!
r+3 (r + 3)! (r + 3)!
C3 = 3!(r + 3 – 3)! = 3! r!

Exercise 10 Solve each of the following.


Selesaikan setiap yang berikut.
PL 3 Apply the understanding of permutation and combination to perform simple tasks.

Example 10 1 Determine the number of ways to select 3 cakes


from 10 cakes.
Find the number of ways to choose 3 durians from
Tentukan bilangan cara memilih 3 biji kek daripada 10 biji kek.
7 durians.
Cari bilangan cara memilih 3 biji durian daripada 7 biji durian.
Number of ways
10
= C3
Solution
10!
n = 7, r = 3 = 3!(10 – 3)!
Number of ways/ Bilangan cara 10!
7 = 3! 7!
= C3
7! = 120
= 3!(7 – 3)!
7!
= 3! 4!
= 35

2 There are 10 deer in a park. If 4 deer are to be given 3 There are 13 people in a room. If each person is
away to a zoo, how many ways are there to select asked to shake hand with each other exactly once,
the deer? how many handshakes would there be?
Terdapat 10 ekor rusa di sebuah taman. Jika 4 ekor rusa akan Terdapat 13 orang di dalam sebuah bilik. Jika setiap orang
diberikan ke sebuah zoo, berapakah bilangan cara pemilihan diminta untuk bersalam antara satu sama lain sekali sahaja,
rusa itu boleh dilakukan? berapakah bilangan cara bersalaman yang dapat dilakukan?

Number of ways Number of ways


10 13
= C4 = C2
10! 13!
= 4!(10 – 4)! = 2!(13 – 2)!
10! 13!
= 4! 6! = 2! 11!
= 210 = 78

87

F5 Mod A+ ADDM 4(78-90).indd 87 6/1/2021 11:08:54 AM


4 The diagram shows 7 points on a piece of cardboard.
Rajah menunjukkan 7 titik di atas sekeping kadbod.
If a triangle can be formed by joining any three points, calculate the number
of ways the triangles can be formed.
Jika sebuah segi tiga boleh dibentuk dengan menyambung mana-mana tiga titik, hitung bilangan
cara segi tiga yang dapat dibentuk.

Number of ways
7
= C3
7!
= 3!(7 – 3)!
7!
= 3! 4!
= 35

Exercise 11 Solve each of the following.


Selesaikan setiap yang berikut.
PL 4 A pply appropriate knowledge and skills of permutation and combination in the context of simple routine problem solving.

Example 11 1 A group of 4 students is to be chosen from 3 boys


and 3 girls to participate in a performance. Find
A school prefect committee that consists of 8 the number of differents ways the students can be
members is to be chosen from 6 boys and 5 girls. selected if
Find the number of different committees that can be Sekumpulan 4 orang murid akan dipilih daripada 3 orang
formed if murid lelaki dan 3 orang murid perempuan untuk menyertai
Sebuah jawatankuasa pengawas yang terdiri daripada 8 suatu persembahan. Cari bilangan cara berlainan murid
orang ahli perlu dipilih daripada 6 orang murid lelaki dan 5 tersebut boleh dipilih jika
orang murid perempuan. Hitung bilangan ahli jawatankuasa (a) there is no restriction,
berlainan yang boleh dibentuk jika tiada syarat,
(a) there is no restriction, (b) at least one boy must be selected.
tiada syarat, sekurang-kurangnya seorang murid lelaki mesti dipilih.
(b) 4 boys and 4 girls are required in the committee.
6
4 orang murid lelaki dan 4 orang murid perempuan (a) C4 = 15 ways
diperlukan dalam jawatankuasa itu.
3 3 3 3 3 3
(b) C1 × C3 + C2 × C2 + C3 × C1
Solution =3×1+3×3+1×3
11
(a) C8 = 165 ways/ cara =3+9+3
6 5
(b) C4 × C4 = 15 × 5 = 15 ways
= 75 ways/ cara

2 There are 10 scouts participating in a camp. 3 Johan wants to mix 5 different watercolours to
Determine the number of ways produce new colours. Find the number of new
Terdapat 10 orang pengakap yang menyertai satu perkhemahan. colours that can be produced.
Tentukan bilangan cara Johan ingin mencampurkan 5 warna air berbeza untuk
(a) 5 scouts are chosen to cook, menghasilkan warna baharu. Cari bilangan warna baharu yang
5 orang pengakap dipilih untuk memasak, mungkin yang boleh dihasilkan.
(b) not more than 4 scouts are chosen to set up a
tent. The colours that can be mixed:
tidak lebih daripada 4 orang pengakap dipilih untuk 2 from 5 or 3 from 5 or 4 from 5 or 5 from 5
memasang khemah.
The total number of new colours
10 5 5 5 5
(a) C5 = 252 ways = C2 + C3 + C4 + C5
(b)
10 10 10
C1 + C2 + C3 + C4
10 = 10 + 10 + 5 + 1
= 26
= 10 + 45 + 120 + 210
= 385 ways

88

F5 Mod A+ ADDM 4(78-90).indd 88 6/1/2021 11:08:55 AM


4 Hashim has 14 books of different titles. 10 of the 5 A group that consist of 3 students are to be chosen
books are in English and the rest are in Malay. He from 4 boys and 6 girls to form a debate team. Find
wants to choose 6 books from his collection. Find the the number of different ways to form the debate
number of ways to choose at least 2 books in Malay. team which consists of
Hashim mempunyai 14 buah buku dengan tajuk yang berlainan. Satu kumpulan yang terdiri daripada 3 orang murid akan
10 buah buku adalah dalam Bahasa Inggeris dan yang selebihnya dipilih daripada 4 orang murid lelaki dan 6 orang murid
adalah dalam Bahasa Melayu. Dia ingin memilih 6 buah buku perempuan untuk membentuk satu pasukan bahas. Cari
daripada koleksinya. Cari bilangan cara memilih sekurang- bilangan cara berlainan untuk membentuk pasukan bahas yang
kurangnya 2 buah buku dalam Bahasa Melayu. terdiri daripada
(a) 2 girls and 1 boy,
4 10 4 10 4 10
( C2 × C4) + ( C3 × C3) +( C4 × C2) 2 orang murid perempuan dan seorang murid lelaki,
= (6 × 210) + (4 × 120) + (1 × 45) (b) at least 2 boys.
= 1 260 + 480 + 45 sekurang-kurangnya 2 orang murid lelaki.
= 1 785 ways 6 4
(a) C2 × C1 = 15 × 4
= 60 ways
4 6 4 6
(b) ( C2 × C1) + ( C3 × C0)
= (6 × 6) + (4 × 1)
= 36 + 4
= 40 ways

Review 4
Paper 1

Section A Satu kod lima aksara akan dibentuk menggunakan


n semua angka dan huruf itu tanpa ulangan. Kod itu mesti
1 (a) Find the value of P0.
n mengandungi 3 angka diikuti dengan 2 huruf. Berapakah
Cari nilai bagi P0.
bilangan kod berlainan yang boleh dibentuk?
[1 mark/markah]
[3 marks/markah]
(b) Diagram 1 shows five number cards and three
letter cards.
Rajah 1 menunjukkan lima keping kad angka dan tiga
2 Diagram 2 shows seven cards of different letters.
Rajah 2 menunjukkan tujuh keping kad berlainan huruf.
keping kad huruf.

9 8 7 6 4 M N P C O M B I N E
Diagram 2/ Rajah 2
Diagram 1/ Rajah 1
A code of 5 characters is to be formed using Find the number of ways to arrange
Cari bilangan cara menyusun
all the numbers and letters without repetition.
(a) four letters from all the cards,
The code must consists of 3 numbers followed
empat huruf daripada semua kad itu,
by 2 letters. How many different codes can be
(b) if the arrangements in (a) begin with a consonant.
formed? jika susunan di (a) bermula dengan huruf konsonan.
[5 marks/markah]

89

F5 Mod A+ ADDM 4(78-90).indd 89 6/1/2021 11:08:55 AM


3 Diagram 3 shows six letter cards. 4 A committee that consists of 7 members is to be
Rajah 3 menunjukkan enam keping kad huruf. selected from 6 teachers and 5 students. Find the
number of different committees that can be formed if
P U N C A K Satu jawatankuasa yang terdiri daripada 7 orang ahli akan
Diagram 3/ Rajah 3 dipilih daripada 6 orang guru dan 5 orang murid. Cari bilangan
jawatankuasa berlainan yang dapat dibentuk jika
Calculate the number of ways to arrange all the letters (a) there is no restriction,
without repetition if tiada syarat,
Hitung bilangan cara menyusun semua huruf itu tanpa ulangan (b) the number of teachers must exceed the number
jika of students.
(a) there is no restriction, bilangan guru mesti melebihi bilangan murid.
tiada syarat, [4 marks/markah]
(b) the first letter and the last letter are consonants.
huruf pertama dan huruf terakhir ialah konsonan.
[4 marks/markah]

Section B
5 (a) Saiful has seven marbles of different colours. He wants to arrange all the marbles in a row. Find the number of
ways the marbles can be arranged if the white marble is not placed next to the red marble.
Saiful mempunyai tujuh biji guli yang berlainan warna. Dia ingin menyusun semua guli itu sebaris. Cari bilangan cara guli itu boleh
disusun jika guli putih tidak diletak bersebelahan dengan guli merah.
[4 marks/markah]
(b) Diagram 4 shows seven cards.
Rajah 4 menunjukkan tujuh keping kad.

3 4 5 6 7 X Y
Diagram 4/ Rajah 4
A code that consists of numbers and letters is to be formed using five cards. How many different codes can be
formed if each code must contain at least 3 numbers that are arranged side by side.
Satu kod yang terdiri daripada angka dan huruf akan dibentuk menggunakan lima keping kad. Berapakah bilangan kod berlainan yang
dapat dibentuk jika setiap kod mesti mengandungi sekurang-kurangnya 3 angka yang disusun bersebelahan.
[4 marks/markah]

H O TS Zo n e
1 Find the number of arrangements of all the letters from the word ‘REMAINS’ if the vowels are always placed in
the odd places. Applying

Cari bilangan susunan bagi semua huruf daripada perkataan ‘REMAINS’ jika huruf vokal sentiasa berada pada kedudukan ganjil.

2 Diagram 1 shows eight points on the straight line AB and four points on the straight line CD.
Rajah 1 menunjukkan lapan titik pada garis lurus AB dan empat titik pada garis lurus CD.

A B

C D

Diagram 1/ Rajah 1
If a triangle can be formed by joining three points from the two straight lines, calculate the number of triangles
that can be formed. Applying

Jika sebuah segi tiga boleh dibentuk dengan menyambungkan tiga titik daripada dua garis lurus itu, hitung bilangan segi tiga yang dapat
dibentuk.

90

F5 Mod A+ ADDM 4(78-90).indd 90 6/1/2021 11:08:55 AM


Learning Area: Statistics

5 Probability Distribution
Taburan Kebarangkalian
5.1 Random Variable/ Pemboleh Ubah Rawak

Smart Tip
A random variable refers to a variable with numerical values as a result of a random phenomenon. The random variable
is denoted by X.
Pemboleh ubah rawak merujuk kepada suatu pemboleh ubah dengan nilai berangka hasil daripada suatu fenomena rawak. Pemboleh ubah rawak
diwakili oleh X.

Exercise 1 List all the values of the random variable for each of the following situations.
Senaraikan semua nilai pemboleh ubah rawak bagi setiap situasi yang berikut.
PL 2 Demonstrate the understanding of probability distribution.

Example 1 1 A coin is tossed six times. X represents the number


of times of obtaining tails.
A fair dice is tossed four times. X represents the Sekeping duit syiling dilambungkan sebanyak enam kali.
number of times to get the number 6. X mewakili bilangan kali mendapat angka.
Sebiji dadu adil dilambungkan sebanyak empat kali. X mewakili
bilangan kali mendapat nombor 6.
X = {0, 1, 2, 3, 4, 5, 6}
Solution
X = {0, 1, 2, 3, 4}

2 A chess player plays five games. X represents the 3 A sample of eight bulbs are selected from a bulb
number of times he wins. factory. X represents the number of defected bulbs.
Seorang pemain catur bermain sebanyak lima permainan. Satu sampel 8 biji mentol dipilih daripada sebuah kilang mentol.
X mewakili bilangan kali dia menang. X mewakili bilangan mentol yang rosak.

X = {0, 1, 2, 3, 4, 5} X = {0, 1, 2, 3, 4, 5, 6, 7, 8}

Smart Tip
1 A discrete random variable has countable values of the random variable.
Pemboleh ubah rawak diskret mempunyai nilai pemboleh ubah yang boleh dibilang.
2 A continous random variable has values of the random variable in a certain interval.
Pemboleh ubah rawak selanjar mempunyai nilai pemboleh ubah yang berada dalam suatu selang tertentu.

91

F5 Mod A+ ADDM 5(91-112).indd 91 5/1/2021 3:48:19 PM


Exercise 2 Determine whether each of the following events is a discrete random variable or a continuous
random variable.
Tentukan sama ada setiap peristiwa yang berikut ialah pemboleh ubah rawak diskret atau pemboleh ubah rawak
selanjar.
PL 2 Demonstrate the understanding of probability distribution.

Event Answer
Peristiwa Jawapan

The number of raining days in a week.


1 Discrete
Bilangan hari hujan dalam satu minggu.

The duration to complete a task.


2 Continuous
Tempoh masa untuk menyiapkan suatu tugasan.

The number of defected smartphones.


3 Discrete
Bilangan telefon bimbit yang rosak.

The age of a group of tourists.


4 Continuous
Umur bagi sekumpulan pelancong.

The mass of workers in a factory.


5 Continuous
Jisim pekerja di sebuah kilang.

The number of males in a family.


6 Discrete
Bilangan lelaki dalam sebuah keluarga.

The height of students in a school.


7 Continuous
Tinggi murid di sebuah sekolah.

The number of times a football team wins in a tournament.


8 Discrete
Bilangan kali sebuah pasukan bola sepak menang dalam suatu pertandingan.

The household income in a state.


9 Continuous
Pendapatan isi rumah di sebuah negeri.

The total financial debt.


10 Continuous
Jumlah hutang kewangan.

Smart Tip
1 If X represents a discrete random variable with the values of r1, r2, ..., rn then, the probability for each value is
P(X = r1), P(X = r2),… , P(X = rn ).
Jika X mewakili pemboleh ubah rawak diskret dengan nilai r1, r2, ..., rn maka, kebarangkalian bagi setiap nilai ialah P(X = r1), P(X = r2),…,
P(X = rn).
n
2 P(X = ri) = 1 where each P(X = ri) > 0.
i=1
n

i=1
P(X = ri) = 1 dengan keadaan setiap P(X = ri) > 0.

92

F5 Mod A+ ADDM 5(91-112).indd 92 5/1/2021 3:48:20 PM


Exercise 3 Write down X in set notations. Hence, draw a tree diagram to represent all the possible outcomes
of X.
Tuliskan X dalam bentuk tatatanda set. Seterusnya, lukis gambar rajah pokok untuk mewakili semua kesudahan
yang mungkin bagi X.
PL 2 Demonstrate the understanding of probability distribution.

Example 2
1 There are three blue balls and two green balls in a
bag. Two balls are drawn at random from the bag
A coins is tossed three times. X represents the
and X represents the number of times of getting
number of times of getting tails.
Sekeping duit syiling dilambungkan sebanyak tiga kali. X
blue ball.
mewakili bilangan kali mendapat angka. Terdapat tiga biji bola biru dan dua biji bola hijau di dalam
sebuah beg. Dua biji bola diambil secara rawak dari beg itu dan
Solution X mewakili bilangan kali mengambil bola biru.
X = {0, 1, 2, 3 }
Let H is getting heads and T is getting tails. X = {0, 1, 2}
Katakan H ialah mendapat gambar dan T ialah mendapat Let B is getting blue ball and G is getting green ball.
angka.
First Second Outcome X=r
drawn drawn
First Second Third Outcome X=r
toss toss toss Kesudahan B BB 2
Lambungan Lambungan Lambungan B BB 2
pertama kedua ketiga
B
H HHH 0 G BG 1
H G BG 1
T HHT 1
H G GG 0
H HTH 1
T B GB 1
T HTT 2 G
H THH 1 B GB 1
H
T THT 2 B GB 1
T
H TTH 2
T
T TTT 3 X = {BB, BB, BG, BG, GG, GB, GB, GB}

X = {HHH, HHT, HTH, HTT, THH, THT, TTH, TTT}

2 A fair dice is tossed three times. X represents the number of times of getting number 4.
Sebiji dadu adil dilambungkan sebanyak tiga kali. X mewakili bilangan kali mendapat nombor 4.

X = {0, 1, 2, 3}
Let 4 is getting the number ‘4’ and 4’ is not getting the number ‘4’.

First Second Third Outcome X=r


toss toss toss
4 444 3
4
4 444 2
4
4 444 2
4
4 444 1
4 444 2
4
4 444 1
4
4 444 1
4
4 444 0

X = {444, 444’, 44’4, 44’4’, 4’44, 4’44’, 4’4’4, 4’4’4’}

93

F5 Mod A+ ADDM 5(91-112).indd 93 5/1/2021 3:48:20 PM


Exercise 4 Solve each of the following.
Selesaikan setiap yang berikut.
PL 3 Apply the understanding of probability distribution to perform simple tasks.

Example 3
The diagram shows a spinning wheel that is divided into four sectors of equal size. One sector is in yellow and
three sectors are in green.
Rajah menunjukkan sebuah roda putar yang dibahagikan kepada empat sektor yang sama besar. Satu sektor berwarna kuning dan tiga
sektor berwarna hijau.
Green
Hijau

Green Yellow
Hijau Kuning

Green
Hijau

The wheel is spun three times. Given X is the number of times of getting yellow sector. Complete the table below.
Hence, draw the graph of probability distribution of X.
Roda itu diputarkan sebanyak tiga kali. Diberi X ialah bilangan kali mendapat sektor berwarna kuning. Lengkapkan jadual di bawah.
Seterusnya, lukis graf taburan kebarangkalian bagi X.

X=r 0 1 2 3
P(X = r)

Solution
Let Y is getting yellow sector and G is getting green sector.
Katakan Y ialah mendapat sektor kuning dan G ialah mendapat sektor hijau.
P(X = r)
First spin Second spin Third spin Outcome Probability X=r
Putaran Putaran Putaran Kesudahan Kebarangkalian
pertama kedua ketiga

1 1
0.4
4 Y YYY 64 3
1 Y
4 3 3
G YYG 64 2
4
Y 0.3
1 3
3 4 Y YGY 64 2
1
4
4 G
3 9
G YGG 64 1
4
0.2
1 3
4 Y GYY 64 2
3 1 Y
4 4 3 9
G GYG 64 1
4 0.1
G
1 9
3 4 Y GGY 64 1
4 G
3 27
G GGG 64 0
4 0 r
0 1 2 3
P(X = 0) = P(GGG)
27
=
64
= 0.4219
P(X = 1) = P(YGG) + P(GYG) + P(GGY) P(X = 3) = P(YYY)
9 9 9 1
= + + =
64 64 64 64
27 = 0.0156
=
64
= 0.4219 X=r 0 1 2 3
P(X = 2) = P(YYG) + P(YGY) + P(GYY) P(X = r) 0.4219 0.4219 0.1406 0.0156
3 3 3
= + +
64 64 64
9
=
64
= 0.1406

94

F5 Mod A+ ADDM 5(91-112).indd 94 5/1/2021 3:48:21 PM


1 The diagram shows a spinning wheel having three sectors of equal size. Two sectors are in red and one sector
in blue.
Rajah menunjukkan roda putar yang mempunyai tiga sektor yang sama saiz. Dua sektor berwarna merah dan satu sektor berwarna
biru.

Red
Blue Merah
Biru

Red
Merah

The wheel is spun three times and X represents the number of times of obtaining red sector. Complete the table
below. Hence, draw the graph of probability distribution of X.
Roda itu diputarkan sebanyak tiga kali dan X mewakili bilangan kali mendapat sektor berwarna merah. Lengkapkan jadual di bawah.
Seterusnya, lukis graf taburan kebarangkalian bagi X.

X=r 0 1 2 3
P(X = r)

Let R is getting red sector and B is getting blue sector.

First Second Third Outcome Probability X=r P(X = r)


spin spin spin
2 8
3 R RRR 27 3
2 R 0.5
3 4
1 B RRB 2
27
3
R
2 4
1 3 R RBR 27 2
2
3
3 B 0.4
2
1 B RBB 1
27
3
2 4
3 R BRR 27 2
1 2 R 0.3
3 3 2
1 B BRB 1
27
3
B
2 2
1 3 R BBR 27 1
0.2
3 B
1
1 B BBB 0
27
3

1 0.1
P(X = 0) = P(BBB) = 27  
= 0.0370
2 2 2
P(X = 1) = P(RBB) + P(BRB) + P(BBR) = 27 + 27 + 27 r
0
6 0 1 2 3
= 27
= 0.2222
4 4 4
P(X = 2) = P(RRB) + P(RBR) + P(BRR) =
27 + 27 + 27
12
= 27
= 0.4444
8
P(X = 3) = P(RRR) = 27
= 0.2963

X=r 0 1 2 3

P(X = r) 0.0370 0.2222 0.4444 0.2963

95

F5 Mod A+ ADDM 5(91-112).indd 95 5/1/2021 3:48:21 PM


2 A coin is tossed twice. Given X represents the number of times of obtaining heads.
Complete the table below. Hence, draw the graph to show the probability distribution of X.
Sekeping duit syiling dilambungkan sebanyak dua kali. Diberi X mewakili bilangan kali mendapat gambar. Lengkapkan jadual di
bawah. Seterusnya, lukis graf untuk menunjukkan taburan kebarangkalian X.

X=r 0 1 2
P(X = r)

Let H is obtaining heads and T is obtaining tails.


First Second Outcome Probability X=r
toss toss
1 1
2 H HH 4 2
1 H
2 1
1 T HT 1
4
2
1 1
1 2 H TH 4 1
2 T
1 P(X = r)
1 T TT 0
4
2

1
P(X = 0) = P(TT) = 4 0.50
= 0.25
1 1
P(X = 1) = P(HT) + P(TH) = 4 + 4
1 0.25
=2
= 0.5
1
P(X = 2) = P(HH) = 4
= 0.25 0 r
0 1 2

X=r 0 1 2

P(X = r) 0.25 0.5 0.25

5.2 Binomial Distribution / Taburan Binomial

Smart Tip
1 An experiment that only has two outcomes, either ‘success’ or ‘failure’ is known as a Bernoulli trial. If a Bernoulli trial is
repeated for n times, it is known as the binomial experiment.
Suatu eksperimen yang mempunyai dua kesudahan sahaja, iaitu ‘kejayaan’ atau ‘kegagalan’ dikenali sebagai percubaan Bernoulli. Jika percubaan
Bernoulli dilakukan sebanyak n kali, ia dikenali sebagai ekperimen binomial.
2 A binomial random variable is the number of success, r from n Bernoulli trials. The probability distribution for binomial
random variable is known as binomial distribution.
Pemboleh ubah rawak binomial ialah bilangan kejayaan, r daripada n percubaan Bernoulli. Taburan kebarangkalian bagi pemboleh ubah rawak
binomial dikenali sebagai taburan binomial.
3 A binomial random variable X in n trials, where the probability of ‘success’ is p can be written as:
Pemboleh ubah rawak binomial X dalam n percubaan, dengan keadaan kebarangkalian ‘kejayaan’ ialah p boleh ditulis sebagai:
X ~ B(n, p)
4 The probability of obtaining r successes in a binomial distribution is given by
Kebarangkalian memperoleh r kejayaan bagi suatu taburan binomial diberi oleh
X = r) = nCrpr qn – r, r = 1, 2, 3,…, n
P(X
where/dengan keadaan
n = number of trials/bilangan percubaan
p = probability of success/kebarangkalian kejayaan
q = probability of failure/kebarangkalian kegagalan

96

F5 Mod A+ ADDM 5(91-112).indd 96 5/1/2021 3:48:22 PM


Exercise 5 Solve each of the following.
Selesaikan setiap yang berikut.
PL 3 Apply the understanding of probability distribution to perform simple tasks.

Example 4 1 In a research, it is found that the probability of rain


1
In a basket of rambutans, it is found that 40% of the will fall in Kamunting on a certain day is 3 . Find
rambutans are yellow. If 8 rambutans are chosen at the probability that in a certain week, the rain will
random, find the probability that fall
Di dalam sebakul rambutan, didapati bahawa 40% daripada Dalam suatu kajian, didapati bahawa kebarangkalian hujan
rambutan itu berwarna kuning. Jika 8 biji rambutan dipilih 1
akan turun di Kamunting pada suatu hari tertentu ialah .
secara rawak dari bakul itu, cari kebarangkalian bahawa 3
Cari kebarangkalian bahawa dalam suatu minggu tertentu,
(a) at least 3 rambutans are yellow,
sekurang-kurangnya 3 biji rambutan berwarna kuning, hujan akan turun
(b) exactly 6 rambutans are yellow. (a) exactly 3 days,/ tepat 3 hari,
tepat 6 biji rambutan berwarna kuning. (b) less than 3 days,/ kurang daripada 3 hari,
(c) more than 5 days./ lebih daripada 5 hari.
Solution
n = 8, p = 0.4, q = 0.6 1 2
n = 7, p = ,q=
X = Number of yellow rambutans 3 3
Bilangan rambutan berwarna kuning X = Number of rainy days
X ~ B(8, 0.4)
( )
X ~ B 7,
1
3
(a) P(X > 3) = 1 − [P(X = 0) + P(X = 1) + P(X = 2)]
( 13 ) ( 23 ) = 0.2561
3 4
(a) P(X = 3)= 7C3
= 1 − [8C0(0.4)0(0.6)8 + 8C1(0.4)1(0.6)7
+ 8C2(0.4)2(0.6)6] (b) P(X , 3)
= 1− [0.0168 + 0.0896 + 0.2090] = P(X = 0) + P(X = 1) + P(X = 2)
( )( ) ( )( )
1 0 2 7 7 1 1 2 6 7 1
( ) ( 23 )
2 5
7
= 0.6846 = C0 + C1 + C2
3 3 3 3 3
(b) P(X = 6) = 8C6(0.4)6(0.6)2 = 0.05853 + 0.2048 + 0.3073
= 0.0413 = 0.5706
(c) P(X . 5)
= P(X = 6) + P(X = 7)
( )( )
1 6 2 1 7 1 7 2
( )( )
0
7
= C6 + C7
3 3 3 3
= 0.006401 + 0.000457
= 0.006858

2 It is found that the probability Intan wins in a story 3 A coin is tossed 7 times, find the probability that
2 Sekeping duit syiling dilambungkan sebanyak 7 kali, cari
telling competition is . If she takes part in
5 kebarangkalian bahawa
7 competitions, find the probability that (a) not obtaining tails,
Didapati bahawa kebarangkalian Intan memenangi dalam tidak mendapat angka,
2
suatu pertandingan bercerita ialah . Jika dia mengambil
5
(b) obtaining tails three times.
bahagian dalam 7 pertandingan, cari kebarangkalian bahawa tiga kali mendapat angka.
(a) she wins exactly 4 times,
dia menang tepat 4 kali. n = 7, p =0.5, q = 0.5
(b) she loses less than twice. X = Number of times obtaining tails
dia kalah kurang daripada dua kali. X ~ B(7, 0.5)

2 3 (a) P(X = 0) = 7C0 (0.5)0(0.5)7


n = 7, p = ,q=
5 5 = 0.007813
X = Number of times winning
X ~ B 7,( )
2
5
(b) P(X = 3) = 7C3 (0.5)3(0.5)4
= 0.2734

( 25 ) ( 35 ) = 0.1935
4 3
(a) P(X = 4) = 7C4

(b) P(X = 6) + P(X = 7)

( )( )
2 6 3 1 7 2
( ) ( 35 )
7 0
= 7C6 + C7
5 5 5
= 0.0172 + 0.001638
= 0.01884

97

F5 Mod A+ ADDM 5(91-112).indd 97 5/1/2021 3:48:22 PM


Exercise 6 Solve each of the following.
Selesaikan setiap yang berikut.
PL 3 Apply the understanding of probability distribution to perform simple tasks.

Example 5 1 There are 4 cats in a house. A cat is chosen at


random from the house. X represents the number of
The analysis of results of SPM examination in a
male cats.
school shows that 20% of the candidates obtained
Terdapat 4 ekor kucing di dalam sebuah rumah. Seekor kucing
grade A in Additional Mathematics. A sample of 5 dipilih secara rawak dari rumah itu. X mewakili bilangan
candidates is chosen at random from the school. X kucing jantan.
represents the number of candidates who obtained (a) Find P(X = 0), P(X = 1), P(X = 2), P(X = 3) and
grade A in Additional Mathematics. P(X = 4).
Analisis keputusan peperiksaan SPM di sebuah sekolah Cari P(X = 0), P(X = 1), P(X = 2), P(X = 3) dan P(X = 4).
menunjukkan bahawa 20% daripada calon mendapat gred A
(b) Construct a binomial probability distribution
dalam mata pelajaran Matematik Tambahan. Satu sampel yang
terdiri daripada 5 orang calon dipilih secara rawak dari sekolah
table and draw the graph of the distribution.
Bina satu jadual taburan kebarangkalian binomial dan
itu. X mewakili bilangan calon yang mendapat gred A dalam
lukis graf bagi taburan itu.
mata pelajaran Matematik Tambahan.
(a) List all the possible outcomes of X.
( 12 ) ( 12 ) = 161
0 4
Senaraikan semua kesudahan yang mungkin bagi X. (a) P(X = 0) = 4C0
(b) Calculate the probability of each outcome.
P(X = 1) = C ( ) ( ) =
Hitung kebarangkalian bagi setiap kesudahan. 1 1
4
1
4 13
=
(c) Hence, draw a graph to represent the 2 2 1 16 4
P(X = 2) = C ( ) ( ) =
binomial distribution of X. 1 1
4 6
2
3 2
=
Seterusnya, lukis satu graf untuk mewakili taburan 2 2 2 16 8
P(X = 3) = C ( ) ( ) =
binomial bagi X. 1 1
4
3
4 11
=
2 2 3 16 4
Solution
P(X = 4) = C ( ) ( ) =
1 1
4
4
1 0

(a) X = {0, 1, 2, 3, 4, 5} 2 2 4 16
(b) P(X = 0) = 5C0(0.2)0(0.8)5 = 0.3277 (b)
P(X = 1) = 5C1(0.2)1(0.8)4 = 0.4096
P(X = 2) = 5C2(0.2)2(0.8)3 = 0.2048 X=r 0 1 2 3 4
P(X = 3) = 5C3(0.2)3(0.8)2 = 0.0512
1 1 3 1 1
P(X = 4) = 5C4(0.2)4(0.8)1 = 0.0064 P(X = r)
16 4 8 4 16
P(X = 5) = 5C5(0.2)5(0.8)0 = 0.0003

(c)
P(X = r) P(X = r)

0.5 3
8
5
16
0.4 1
4
3
16
0.3 1
8
1
16
0.2
0 r
0 1 2 3 4

0.1

0 r
0 1 2 3 4 5

98

F5 Mod A+ ADDM 5(91-112).indd 98 5/1/2021 3:48:23 PM


2 In a survey, it is found that only 70% of the number 3 The diagram below shows the graph of a binomial
of houses in a rural village have electricity. If distribution of X.
6 houses are chosen at random, determine the Rajah di bawah menunjukkan graf bagi suatu taburan
binomial distribution for the number of houses binomial X.
that have electricity. Hence, draw a graph of the P(X = r)
binomial distribution.
Dalam satu tinjauan, didapati bahawa hanya 70% daripada 10
bilangan rumah di sebuah kampung di kawasan pedalaman 20
yang mempunyai elektrik. Jika 6 buah rumah dipilih secara 7
20
rawak, tentukan taburan binomial bagi bilangan rumah yang
mempunyai elektrik. Seterusnya, lukis graf bagi taburan k
1
binomial itu. 20

( 107 ) ( 103 ) = 0.000729


0 6
0 r
P(X = 0) = 6C0 0 1 2 3

Find/ Cari
( 107 ) ( 103 ) = 0.0102
1 5
6
P(X = 1) = C1 (a) P(X  2),
(b) the value of k./ nilai k.
P(X = 2) = C ( ) ( ) = 0.0595
76 3 2 4

10 10 2
(a) P(X  2) = P(X = 0) + P(X = 1) + P(X = 2)
P(X = 3) = C ( ) ( ) = 0.1852
76 3 3 3
1 7 10
10 10 3 = 20 + 20 + 20
P(X = 4) = C ( ) ( ) = 0.3241
76 3 4 2
9
10 10 4 = 10

P(X = 5) = C ( ) ( ) = 0.3025
76 3 5 1

10 10 5 10 7 1
(b) k = 1 − 20 − 20 − 20
P(X = 6) = C ( ) ( ) = 0.1176
76 3 6 0
2
10 10 6
= 20
1
= 10

P(X = r)

0.4

0.3

0.2

0.1

0 r
0 1 2 3 4 5 6

Smart Tip
Mean,  = np Variance, s2 = npq Standard deviation, s = npq
Min  = np Varians s 2 = npq Sisihan piawai, s = npq

99

F5 Mod A+ ADDM 5(91-112).indd 99 5/1/2021 3:48:24 PM


Exercise 7 Solve each of the following.
Selesaikan setiap yang berikut.
PL 3 Apply the understanding of probability distribution to perform simple tasks.

Example 6
1 X is a discrete random variable such that
X is a discrete random variable such that X ~ B(n, p). X ~ B(300, 0.25). Find the mean and the standard
If the mean and the standard deviation of X are 120 deviation of X.
X ialah pemboleh ubah rawak diskret dengan keadaan
and 2 5 respectively, find the value of n.
X ialah pemboleh ubah rawak diskret dengan keadaan X ~ B(300, 0.25). Cari min dan sisihan piawai bagi X.
X ~ B(n, p). Jika min dan sisihan piawai bagi X masing-masing
ialah 120 dan 2 5 , cari nilai n. n = 300, p = 0.25, q = 1 – 0.25 = 0.75
Mean = np
Solution = (300)(0.25)
Mean/Min = np = 120 = 75
Standard deviation = npq
Standard deviation/ Sisihan piawai = npq
= (300)(0.25)(0.75)
2 5 = 120q = 7.5
20 = 120q
1
q=
6
1 5
Thus/ Maka, p =1− 6 = 6
5
( )
n 6 = 120
n = 144

2 A discrete random variable X has a binomial 3 A discrete random variable X ~ B(n, p) has a mean
distribution, that is X ~ B(600, p). It is given that the of 60 and a standard deviation of 3 5 . Find the
standard deviation is 12 and p , 0.5. Determine the value of n.
value of p. Suatu pemboleh ubah rawak diskret X ~ B(n, p) mempunyai
Suatu pemboleh ubah rawak diskret X bertaburan binomial, min 60 dan sisihan piawai 3 5 . Cari nilai n.
iaitu X ~ B(600, p). Diberi bahawa sisihan piawai ialah 12 dan
p , 0.5. Tentukan nilai p. Mean = np = 60
Standard deviation = npq
n = 600, Given p and q = 1 – p
3 5 = 60q
Standard deviation = npq
(3 5 )2 = 60q
12 = npq
45 = 60q
12 = (600)(p)(1 – p) 3
600p(1 − p) = 122 q=
4
600p(1 − p) = 144 ( 24) 3 1
25p(1 − p) = 6 Therefore, p = 1 − =
4 4
25p – 25p2 = 6
25p2 – 25p + 6 = 0
n ( )
1
4
= 60
n = 240
(5p − 2)(5p – 3) = 0
2 3
p = or p =
5 5 2
Given p < 0.5, [ p =
5

100

F5 Mod A+ ADDM 5(91-112).indd 100 5/1/2021 3:48:24 PM


Exercise 8 Solve each of the following problems.
Selesaikan setiap masalah yang berikut.
PL 4 Apply appropriate knowledge and skills of probability distribution in the context of simple routine problem-solving.

Example 7

In a singing competition, 3 out of 7 participants failed to proceed to the final.


Dalam suatu pertandingan nyanyian, 3 daripada 7 orang peserta gagal ke peringkat akhir.
(a) If 4 participants are chosen at random, find the probability that not more than 2 participants failed to proceed
to the final.
Jika 4 orang peserta dipilih secara rawak, cari kebarangkalian bahawa tidak lebih daripada 2 orang peserta gagal ke peringkat akhir.
(b) If there are 15 participants in the competition, find the mean and the standard deviation of the number of
participants who failed to proceed to the final.
Jika terdapat 15 orang peserta dalam pertandingan itu, cari min dan sisihan piawai bagi bilangan peserta yang gagal ke peringkat
akhir.

Solution
(a) Let X = Number of participants who failed to proceed to the final
Katakan X = Bilangan peserta yang gagal ke peringkat akhir

X ~B 4, ( 37 )
P(X  2) = P(X = 0) + P(X = 1) + P(X = 2)

( )( )
3 0 4 4 4 3 1 4 3 4 3
( )( ) ( ) ( 47 )
2 2
= 4C0 + C1 + C2
7 7 7 7 7
= 0.1066 + 0.3199 + 0.3599
= 0.7864

(b) X ~B 15, ( 3
7 ) 3
Mean/Min = np =15 × = 6.429
7
Standard deviation/ Sisihan piawai = npq
3 4
= 15 × ×
7 7
= 1.917

1 It is found that 80% of the candidates who took the culinary test will pass the test.
Didapati bahawa 80% daripada calon yang mengambil ujian memasak akan lulus ujian itu.
(a) In a sample that consists of 8 candidates, find the probability that at least 3 candidates passed the test.
Dalam suatu sampel yang terdiri daripada 8 orang calon, cari kebarangkalian bahawa sekurang-kurangnya 3 orang calon lulus
ujian itu.
(b) If there are 200 candidates who sat for the test on a certain day, determine the mean and the standard
deviation of the number of candidates who passed the test.
Jika terdapat 200 orang calon yang menduduki ujian itu pada suatu hari tertentu, tentukan min dan sisihan piawai bagi bilangan
calon yang lulus ujian itu.

(a) p = 0.8, q = 0.2, n = 8


Let X = Number of candidates who passed the test
P(X > 3) = 1 − P(X = 0) − P(X = 1) − P(X = 2)
= 1 − 8C0(0.8)0(0.2)8 − 8C1(0.8)1(0.2)7 − 8C2(0.8)2(0.2)6
= 1 − 0.00000256 − 0.00008192 − 0.00114688
= 0.9988

(b) μ = np
= (200)(0.8)
= 160
s = npq
= 200(0.8)(0.2)
= 32
= 5.657

101

F5 Mod A+ ADDM 5(91-112).indd 101 5/1/2021 3:48:24 PM


2 In a research in a district, it is found that two out of five Form 5 students in the district are in boarding school.
Dalam satu kajian di sebuah daerah, didapati bahawa dua daripada lima orang murid Tingkatan 5 di daerah itu bersekolah di sekolah
berasrama.
(a) If eight Form 5 students are selected at random from the district, find the probability that at least two
students are in boarding school.
Jika lapan orang murid Tingkatan 5 dipilih secara rawak dari daerah itu, cari kebarangkalian bahawa sekurang-kurangnya dua
orang murid bersekolah di sekolah berasrama.
(b) If there are 3 600 Form 5 students in the district, calculate the mean and the standard deviation of the
students who are in boarding school.
Jika terdapat 3 600 orang murid Tingkatan 5 di daerah itu, hitung min dan sisihan piawai bagi bilangan murid yang bersekolah di
sekolah berasrama.

2
(a) n = 8, p =
= 0.4, q = 1 – 0.4 = 0.6
5
Let X = Number of students who are in boarding school
P(X > 2) = 1− [P(X = 0) + P(X = 1)]
= 1 − [8C0(0.4)0(0.6)8 + 8C1(0.4)1(0.6)7]
= 1 − [0.01680 + 0.08958 ]
= 0.8936

(b) μ = np
= (3 600)(0.4)
= 1 440
s = npq
= (3 600)(0.4)(0.6)
= 864
= 29.39

3 In a basketball game, it is found that Johan attains 40% of success of the total trials he did.
Dalam suatu permainan bola keranjang, didapati bahawa Johan mendapat 40% kejayaan daripada jumlah percubaan yang dia lakukan.
Calculate/ Hitung
(a) the probability that Johan attains exactly 5 successes from 6 trials,
kebarangkalian bahawa Johan mendapat tepat 5 kejayaan daripada 6 percubaan,
(b) the number of trials he has to make so that the probability of success is more than 0.99.
bilangan percubaan yang perlu dia lakukan supaya kebarangkalian kejayaan lebih daripada 0.99.

(a) n = 6 , p = 0.4, q = 1 – 0.4 = 0.6


Let X = Number of success
P(X = 5) = 6C5(0.4)5(0.6)1
= 0.03686

(b) P(X > 1) . 0.99
1 – P(X = 0) . 0.99
1 − nC0(0.4)0(0.6)n . 0.99
1 × 1 × 0.6n , 0.01
0.6n , 0.01
log 0.6n , log 0.01
n log 0.6 , log 0.01
n(–0.2218) , –2
0.2218n . 2
n . 9.017
[ n = 10

102

F5 Mod A+ ADDM 5(91-112).indd 102 5/1/2021 3:48:24 PM


4 In a guessing game, the probability of getting the correct guessing is p.
Dalam suatu permainan meneka, kebarangkalian memperoleh tekaan yang betul ialah p.
(a) Find the number of times of guessing and the value of p so that the mean and the standard deviation are 35
and 2 7 respectively.
Cari bilangan kali tekaan dan nilai p supaya min dan sisihan piawai masing-masing ialah 35 dan 2 7 .
(b) If 12 times of guessing is done, find the probability that 3 times of guessing is correct.
Jika 12 kali tekaan dilakukan, cari kebarangkalian bahawa 3 kali tekaan adalah betul.

(a) Mean = np = 35
Standard deviation = 2 7
np(1 – p) = 2 7
np(1 – p) = (2 7 )2

35(1 – p) = 28
28
1 – p = 35  
4
1 – p = 5  
1
p = 5   or 0.2

np = 35
n(0.2) = 35
n = 175

(b) Let X = Number of times the guessing is correct


P(X = 3) = 12C3(0.2)3(0.8)9
= 0.2362

5 In a residential area, 18% of the residents are senior citizens.


Di sebuah kawasan penempatan, 18% daripada penduduknya ialah warga emas.
(a) If 9 residents are selected at random from the area, find the probability that at least three of them are senior
citizens.
Jika 9 orang penduduk dipilih secara rawak dari kawasan itu, cari kebarangkalian bahawa sekurang-kurangnya tiga orang ialah
warga emas.
(b) If the variance of the number of senior citizens is 2 214, how many residents are there in the residential
area?
Jika varians bagi bilangan warga emas ialah 2 214, berapakah bilangan penduduk di kawasan penempatan itu?

(a) p = 0.18, q = 0.82, n = 9


Let X = Number of senior citizens
P(X > 3) = 1 – P(X = 0) – P(X = 1) – P(X = 2)
= 1 – 9C0(0.18)0(0.82)9 – 9C1(0.18)1(0.82)8 – 9C2(0.18)2(0.82)7
= 1 – 0.1676 − 0.3312 – 0.2908
= 0.2104

(b) npq = 2 214


n(0.18)(0.82) = 2 214
2 214
n = (0.18)(0.82)  
= 15 000

103

F5 Mod A+ ADDM 5(91-112).indd 103 5/1/2021 3:48:24 PM


5.3 Normal Distribution / Taburan Normal

Smart Tip
1 A normal distribution has continuous random variables. The normal distribution, X can be written as X ~ N(, s2),
where  is mean and s2 is variance.
Suatu taburan normal mempunyai pemboleh ubah rawak selanjar. Taburan normal, X boleh ditulis sebagai X ~ N(, s2), dengan keadaan  ialah
min and s2 ialah varians.
2 The probability of X from X = a to X = b is written as P(a , X , b). The area under a graph represents the probability of X.
Kebarangkalian bagi X dari X = a hingga X = b ditulis sebagai P(a , X , b). Luas di bawah graf mewakili kebarangkalian bagi X.
ff(
f(x)
x)
x =

P(a , X , b)

0 x
a b
3 The total area under a graph which is equal to the total probability of all the values of X is 1.
Jumlah luas di bawah graf yang bersamaan dengan jumlah kebarangkalian bagi semua nilai X ialah 1.
X–
4 A continous random variable X of a normal distribution can be converted to the standard score, Z, which is Z = .
X–. s
Suatu pemboleh ubah rawak selanjar X bagi taburan nomal boleh ditukarkan kepada skor piawai, Z, iaitu Z =
s

Exercise 9 Find the area of each of the shaded regions based on the given diagram.
Cari luas bagi setiap rantau berlorek berdasarkan rajah yang diberikan.
PL 2 Demonstrate the understanding of probability distribution.

Example 8
1 f(x)

f(x)

0.132
0.526
0 x
 u
0 x
−c  c (a) f(x)

(a) f(x)

0 x
 u

0 x
 c (b) f(x)
(b) f(x)

0 x
 u
0 x
 c
(a) P( , X , u) = 0.5 – 0.132 = 0.368
Solution (b) P(X , u) = 1 – 0.132 = 0.868
1 – 0.526
(a) P(X . c) = = 0.237
2
(b) P(X , c) = 1 – 0.237 = 0.763

104

F5 Mod A+ ADDM 5(91-112).indd 104 5/1/2021 3:48:26 PM


2 f(x) 3 f(x)

0.864
0.158

0 x 0 x
 a −m  m

(a) f(x) (a) f(x)

0 x 0 x
−a   m

(b) f(x) (b) f(x)

0 x 0 x
−a  −m 

(a) P(X , –a) = P(X . a) = 0.158 1 – 0.864


(a) P(X . m) = = 0.068
(b) P(X . –a) = 1 – 0.158 = 0.842 2
(b) P(X . –m) = 1 – 0.068 = 0.932

Exercise 10 Determine the probability of each of the following by using the standard normal distribution
table.
Tentukan kebarangkalian bagi setiap yang berikut dengan menggunakan sifir taburan normal piawai.
PL 2 Demonstrate the understanding of probability distribution.

Example 9
1 P(Z . 0.812)
P(–0.7 , Z , 1.7) f(z)

Solution
f(z)
Area = 0.2085

Area A Area B z
Luas A Luas B 0 0.812
= 0.2420 = 0.0446
P(Z . 0.812) = 0.2085
z
−0.7 0 1.7

P(–0.7 , Z , 1.7)
= 1−Area A – Area B
1 – Luas A – Luas B
= 1 − 0.2420 − 0.0446
= 0.7134

105

F5 Mod A+ ADDM 5(91-112).indd 105 5/1/2021 3:48:27 PM


2 P(Z , –1.21) 3 P(Z , 0.76)

f(z) f(z) f(z)

Area Area
= 0.1131 = = 0.1131
Area = 0.2236
z z
1.21 0 0 1.21 z
0 0.76

P(Z , –1.21) = P(Z . 1.21) P(Z , 0.76) = 1 − P(Z . 0.76)


= 0.1131 = 1 − 0.2236
= 0.7764

4 P(0 , Z , 2.55) 5 P(–1.2 , Z , 0.8)


f(z) f(z)

Area
= 0.00539 Area = 0.1151 Area = 0.2119

z z
0 2.55 1.2 0 0.8

P(0 , Z , 2.55) = 0.5 − P(Z . 2.55) P(–1.2 , Z , 0.8) = 1 – P(Z , −1.2) – P(Z . 0.8)
= 0.5 − 0.00539 = 1 – P(Z . 1.2) – P(Z . 0.8)
= 0.4946 = 1 – 0.1151 – 0.2119
= 0.6730

6 P(0.3 , Z , 2.2) 7 P(–1.4 , Z , –0.6)


f(z) f(z) f(z)

=
z z z
0 0.3 2.2 1.4 0.6 0 0 0.6 1.4
f(z) f(z) f(z) f(z)

Area A Area B
= = 0.3821  Area A  Area B
= 0.0139
= 0.2743 = 0.0808
z z =
z z
0 0.3 0 2.2 0 0.6 0 1.4

P(0.3 , Z , 2.2) = Area A − Area B P(–1.4 , Z , –0.6) = Area A − Area B


= P(Z . 0.3 ) − P(Z . 2.2) = P(Z . 0.6) − P(Z . 1.4)
= 0.3821− 0.0139 = 0.2743 − 0.0808
= 0.3682 = 0.1935

106

F5 Mod A+ ADDM 5(91-112).indd 106 5/1/2021 3:48:27 PM


Exercise 11 Find the value of k for each of the following.
Cari nilai k bagi setiap yang berikut.
PL 3 Apply the understanding of probability distribution to perform simple tasks.

Example 10
1 P(Z . k) = 0.0548 2 P(Z . k) = 0.3156
P(k , Z , 1.07) = 0.6281
P(Z . k) = 0.0548 P(Z . k) = 0.3156
k = 1.6 k = 0.48
Solution
P(k , Z , 1.07) = 0.6281
P(Z . k) – P(Z . 1.07) = 0.6281
P(Z . k) – 0.1423 = 0.6281
P(Z . k) = 0.7704

For/ Untuk k , 0,
1 – P(Z , k) = 0.7704
P(Z , k) = 0.2296
k = − 0.74

3 P(Z . k) = 0.1453 4 P(Z . k) = 0.7881 5 P(Z . k) = 0.8729

P(Z . k) = 0.1453 P(Z . k) = 0.7881 P(Z . k) = 0.8729


k = 1.057 1 – P(Z , k) = 0.7881 1 – P(Z , k) = 0.8729
P(Z , k) = 0.2119 P(Z , k) = 0.1271
k = –0.8 k = –1.14

6 P(Z . k) = 0.9538 7 P(0.4 , Z , k) = 0.2256 8 P(k , Z , 1.247) = 0.7712

P(Z . k) = 0.9538 P(0.4 , Z , k) = 0.2256 P(k , Z , 1.247) = 0.7712


1 – P(Z , k) = 0.9538 P(Z . 0.4) – P(Z . k) = 0.2256 P(Z . k) – P(Z . 1.247) = 0.7712
P(Z , k) = 0.0462 0.3446 – P(Z . k) = 0.2256 P(Z . k) – 0.1062 = 0.7712
k = –1.683 P(Z . k) = 0.119 P(Z . k) = 0.8774
k = 1.18 1 – P(Z , k) = 0.8774
P(Z , k) = 0.1226
k = –1.162

Smart Tip
(
P(X . k)= P Z . k – 
s )
where  = mean and s = standard deviation.
dengan keadaan  = min dan s = sisihan piawai.

107

F5 Mod A+ ADDM 5(91-112).indd 107 5/1/2021 3:48:28 PM


Exercise 12 Solve each of the following.
Selesaikan setiap yang berikut.
PL 3 Apply the understanding of probability distribution to perform simple tasks.
PL 4 Apply appropriate knowledge and skills of probability distribution in the context of simple routine problem-solving.

Example 11 1 The heights of Form 5 students in a school are


normally distributed with a mean of 160 cm and
Given the masses of chickens in a farm are normally a variance of 25 cm2. If a student is selected at
distributed with a mean of 2.3 kg and a variance of random, find the probability that the height of the
0.64 kg2. If a chicken is chosen at random, determine student is more than 165 cm.
the probability that the mass of chicken is more than Ketinggian murid Tingkatan 5 di sebuah sekolah bertaburan
2 kg. normal dengan min 160 cm dan varians 25 cm2. Jika seorang
Diberi jisim ayam di sebuah ladang bertaburan normal dengan
murid dipilih secara rawak, cari kebarangkalian bahawa
min 2.3 kg dan varians 0.64 kg2. Jika seekor ayam dipilih secara
rawak, tentukan kebarangkalian bahawa jisim ayam itu lebih ketinggian murid itu lebih daripada 165 cm.
daripada 2 kg.
 = 160, s2 = 25, s = 5
Solution X = The heights of the students
 = 2.3, s2 = 0.64, s = 0.8 X ~ N(160, 52)
X = The masses of chickens/Jisim ayam
X ~ N(2.3, 0.82)
P(X . 165) = P Z . (
165 – 160
5 )
( )
2 – 2.3 = P(Z . 1)
P(X . 2) = P Z . = 0.1587
0.8
= P(Z . − 0.375)
= 1 – P(Z . 0.375)
= 1 – 0.3538
= 0.6462

2 The masses of durians in an orchard are normally 3 Given the lengths of chillies in a farm are normally
distributed with a mean of 2.8 kg and a variance of distributed with a mean of 8 cm and a standard
2
0.8 kg . Find the probability that a durian chosen deviation of 2.5 cm. If a chilli is randomly selected,
randomly will have a mass more than 3 kg. determine the probability that the length of chilli is
Jisim durian di sebuah dusun bertaburan normal dengan more than 7 cm.
min 2.8 kg dan varians 0.8 kg2. Cari kebarangkalian bahawa Diberi panjang cili di sebuah kebun bertaburan normal dengan
sebiji durian yang dipilih secara rawak mempunyai jisim lebih min 8 cm dan sisihan piawai 2.5 cm. Jika sebiji cili dipilih
daripada 3 kg. secara rawak, tentukan kebarangkalian bahawa panjang cili itu
lebih daripada 7 cm.
 = 2.8, s2 = 0.8, s = 0.8 = 0.8944
X = The masses of the durians  = 8, s = 2.5
X ~ N(2.8, 0.89442) X = The lengths of the chillies
P(X . 3) = P Z . ( 3 – 2.8
) X ~ N(8, 2.52)
0.8944
= P(Z . 0.2236) P(X . 7) = P Z . (
7–8
2.5 )
= 0.4115 = P(Z . –0.4)
= 1 – P(Z . 0.4)
= 1 – 0.3446
= 0.6554

108

F5 Mod A+ ADDM 5(91-112).indd 108 5/1/2021 3:48:28 PM


4 Given X ~ N(7, 1.22), determine P(X . 5.4). 5 Given X ~ N(4, 22), determine P(X . 4.5).
2
Diberi X ~ N(7, 1.2 ), tentukan P(X . 5.4). Diberi X ~ N(4, 22), tentukan P(X . 4.5).

P(X . 5.4) = P Z . ( 5.4 – 7


1.2 ) P(X . 4.5) = P Z . (4.5 – 4
2 )
= P(Z . –1.333) = P(Z . 0.25)
= 1 – P(Z . 1.333) = 0.4013
= 1 – 0.0913
= 0.9087

Exercise 13 Solve each of the following.


Selesaikan setiap yang berikut.
PL 4 Apply appropriate knowledge and skills of probability distribution in the context of simple routine problem-solving.

Example 12 1 An orchard produces rambutans. The rambutans


are being classified into two grades and sold at the
The masses of durians produced in an orchard are
night market. The table below shows the grades of
normally distributed with a mean of  kg and a the rambutans according to their masses.
variance of 0.16 kg2. Sebuah kebun menghasilkan buah rambutan. Buah rambutan
Jisim buah durian yang dihasilkan di sebuah kebun mempunyai itu dikelaskan kepada dua gred dan dijual di pasar malam.
taburan normal dengan min μ kg dan varians 0.16 kg2.
Jadual di bawah menunjukkan gred buah rambutan mengikut
(a) It is given that 7.93% of the number of durians
jisimnya.
produced are more than 2.5 kg, find the value
of . Grade/ Gred A B
Diberi bahawa 7.93% daripada bilangan buah durian yang
dihasilkan itu berjisim lebih daripada 2.5 kg, cari nilai μ. Mass/ Jisim, g X . 35 25 , X , 35
(b) The durians that have masses between 2 kg and It is given that the masses of rambutans are
3 kg will be sold at the local market. Calculate normally distributed with a mean of 30 g and a
the probability that a durian chosen at random standard deviation of 8 g.
will be sold at the local market. Diberi bahawa jisim buah rambutan itu bertaburan normal
Buah durian yang berjisim antara 2 kg dan 3 kg akan
dengan min 30 g dan sisihan piawai 8 g.
dijual di pasar tempatan. Hitung kebarangkalian bahawa
(a) If a rambutan is chosen at random from the
sebiji durian yang dipilih secara rawak akan dijual di pasar
tempatan. orchard, calculate the probability that it is of
grade A.
Jika sebiji buah rambutan dipilih secara rawak dari kebun
Solution
itu, hitung kebarangkalian bahawa buah itu adalah gred A.
(a) s = 0.16 = 0.4
(b) Find the number of grade B rambutans if the
P(X . 2.5) = 0.0793
orchard produced 500 rambutans.
P Z. (2.5 – 
0.4
= 0.0793 ) Cari bilangan rambutan gred B jika kebun itu
menghasilkan 500 biji rambutan.
2.5 – 
= 1.41
0.4  = 30 g, s = 8
 =1.936
(a) P(X . 35) = P Z . ( 35 – 30
8 )
(b) P(2 , Z , 3) =
2 – 1.936
0.4 (,Z,
3 – 1.936
0.4 ) = P(Z . 0.625) = 0.2660

= P(0.16 , Z , 2.66)
= P(Z . 0.16) – P(Z . 2.66)
(b) P(25 , X , 35) = P ( 25 –8 30 , Z , 35 –8 30 )
= 0.4364 – 0.00391 = P(–0.625 , Z , 0.625)
= 0.4325 = 1 – P(Z . 0.625) – P(Z , –0.625)
= 1 – 0.2660 – 0.2660
= 0.468
n(A)
P(A) =
n(S)
x
0.468 =
500
x = 234
[ 234 grade B rambutans

109

F5 Mod A+ ADDM 5(91-112).indd 109 5/1/2021 3:48:28 PM


2 The mass of a group of scout members in a camp is normally distributed with a mean of 60 kg and a standard
deviation of 10 kg. Find the probability that a scout member selected at random has a mass between 45 kg and
75 kg.
Jisim sekumpulan ahli pengakap yang menyertai perkhemahan bertaburan normal dengan min 60 kg dan sisihan piawai 10 kg. Cari
kebarangkalian bahawa seorang ahli pengakap yang dipilih secara rawak mempunyai jisim antara 45 kg dan 75 kg.

 = 60, s = 10
P(45 , X , 75)
=P (
45 – 60
10
,Z,
75 – 60
10 ) −1.5 1.5
z

= P(–1.5 , Z , 1.5)
= 1 − P(Z , −1.5) − P(Z . 1.5)
= 1 − 0.0668 − 0.0668
= 0.8664

3 The diagram shows a standard normal distribution graph and f(z)


P(k , Z , 0.8) = 0.7146.
Rajah menunjukkan graf taburan normal piawai dan P(k , Z , 0.8) = 0.7146.
Find the value of P(Z . k).
Cari nilai P(Z . k).
z
k O 0.8
P(k , Z , 0.8) = 0.7146 P(Z . k) = P(Z . − 1.45)
1 – P(Z , k) – P(Z . 0.8) = 0.7146 = 1 – P(Z . 1.45)
1 − P(Z < k) − 0.2119 = 0.7146 = 1 – 0.0735
1− 0.2119 − 0.7146 = P(Z , k) = 0.9265
P(Z , k) = 0.0735
k = −1.45

Review 5
Paper 1

Section A Find/ Cari


1 A discrete random variable X has a binomial (a) the value of u,
distribution, which is X ~ (4, p). Diagram 1 shows the nilai u,
graph of binomial distribution of X. (b) P(X . 1).
Pemboleh ubah rawak diskret X bertaburan binomial, iaitu [4 marks/markah]
X ~ (4, p). Rajah 1 menunjukkan graf taburan binomial bagi X.
2 Diagram 2 shows the graph of normal distribution,
P(X = x)
X ~ N(, s2).
3
Rajah 2 menunjukkan graf bagi taburan normal, X ~ N(, s2).
8
f(x)
1 H
4

u
0 x
0 1 2 3 4 G
0 x
Diagram 1/ Rajah 1 14 19
Diagram 2/ Rajah 2

110

F5 Mod A+ ADDM 5(91-112).indd 110 5/1/2021 3:48:29 PM


Given GH is the axis of symmetery of the graph, find (b) Express P(X . 0) in terms of r and s.
Diberi GH ialah paksi simetri bagi graf itu, cari Ungkapkan P(X . 0) dalam sebutan r dan s.
(a) the value of , [4 marks/markah]
nilai ,
(b) the value of P(9  X  19) if the area of the 4 Diagram 4 shows the graph of standard normal
shaded region is 0.294. distribution.
nilai P(9  X  19) jika luas rantau berlorek ialah 0.294. Rajah 4 menunjukkan graf bagi taburan normal piawai.
[4 marks/markah] f(z)

3 Diagram 3 shows the graph of binomial distribution


X ~ (3, p).
Rajah 3 menunjukkan graf bagi taburan binomial X ~ (3, p).
P(X = x) z
k 0
r Diagram 4/ Rajah 4
64
125
It is given that P(k , Z , 0) = 0.2764.
s Diberi bahawa P(k , Z , 0) = 0.2764.
(a) Find the value of k.
27
125 Cari nilai k.
0 x (b) The continuous random variable X is normally
0 1 2 3
distributed with a mean of  and a standard
Diagram 3/ Rajah 3 deviation of 4.5. Find the value of  if the z-score
(a) Find the value of p. of X = 57.2 is k.
Cari nilai p. Pemboleh ubah rawak selanjar X bertaburan secara normal
dengan min  dan sisihan piawai 4.5. Cari nilai  jika
skor-z bagi X = 57.2 ialah k.
[4 marks/markah]

Section B
5 (a) A discrete random variable X has a binomial distribution, which is X ~ (12, p). If the mean is 3, find
Suatu pemboleh ubah rawak diskret X mempunyai taburan binomial, iaitu X ~ (12, p). Jika min ialah 3, cari
(i) the value of p,
nilai p,
(ii) P(X  3).
[4 marks/markah]
(b) The diameters of oranges in an orchard are normally distributed with a mean of 6.4 cm and a standard
deviation of 2 cm. Calculate
Diameter buah oren di sebuah kebun bertaburan normal dengan min 6.4 cm dan sisihan piawai 2 cm. Hitung
(i) the probability that an orange chosen at random from the orchard has a diameter more than 7.5 cm,
kebarangkalian bahawa sebiji buah oren yang dipilih secara rawak dari kebun itu mempunyai diameter lebih daripada 7.5 cm,
(ii) the value of h if 40% of the oranges have diameters more than h cm.
nilai h jika 40% daripada buah oren itu mempunyai diameter lebih daripada h cm.
[4 marks/markah]

Paper 2
Section A
1 The diameters of concrete cylindrical pillars produced in a factory are normally distributed with a mean of 40 cm and a
standard deviation of s cm. It is given that the probability of a pillar selected at random from the factory has diameter
more than 43.6 cm is 0.0548.
Diameter tiang silinder konkrit yang dihasilkan di sebuah kilang bertaburan normal dengan min 40 cm dan sisihan piawai s cm. Diberi
bahawa kebarangkalian sebatang tiang yang dipilih secara rawak dari kilang itu mempunyai diameter lebih daripada 43.6 cm ialah 0.0548.
Find/ Cari
(a) the value of s,
nilai s,
[2 marks/markah]
(b) P(40 , X , 44),
[3 marks/markah]
(c) the value of k such that P(X . k) = 0.8085.
nilai k dengan keadaan P(X . k) = 0.8085.
[3 marks/markah]

111

F5 Mod A+ ADDM 5(91-112).indd 111 5/1/2021 3:48:29 PM


2 It is found that 55% of the Form 5 students in a school passed in an Additional Mathematics test. A sample of six Form
5 students is chosen at random.
Didapati bahawa 55% daripada murid Tingkatan 5 di sebuah sekolah lulus dalam ujian Matematik Tambahan. Satu sampel yang terdiri
daripada enam orang murid Tingkatan 5 dipilih secara rawak.
(a) Find the standard deviation of the probability distribution.
Cari sisihan piawai bagi taburan kebarangkalian itu.
[2 marks/markah]
(b) Find the probability that more than four students from the sample passed the test.
Cari kebarangkalian bahawa lebih daripada empat orang murid daripada sampel lulus ujian itu.
[3 marks/markah]
(c) What is the minimum number of sample of students needed so that the minimum mean of the number of
students who passed the test is 9?
Berapakah bilangan minimum sampel murid yang diperlukan supaya min minimum murid yang lulus ujian itu ialah 9 orang?
[3 marks/markah]

Section B
3 (a) In a survey conducted in a state, it is found that 60% of the graduates are unemployed. If 12 graduates from the
state is selected at random, find the probability that
Dalam satu kajian yang dijalankan di sebuah negeri, didapati bahawa 60% daripada siswazah menganggur. Jika 12 orang siswazah
dari negeri itu dipilih secara rawak, cari kebarangkalian bahawa
(i) exactly 8 graduates are unemployed,
tepat 8 orang siswazah menganggur,
(ii) at least 10 graduates are unemployed.
sekurang-kurangnya 10 orang siswazah menganggur.
[5 marks/markah]
(b) There are 400 students who participate in a cross-country running. The time taken to complete the run is
normally distributed with a mean of 50 minutes and a standard deviation of 15 minutes. The participants that
complete the run more than 60 minutes will not be given a medal.
Seramai 400 orang murid menyertai larian merentas desa. Masa yang diambil untuk menamatkan larian itu bertaburan normal
dengan min 50 minit dan sisihan piawai 15 minit. Peserta yang menamatkan larian lebih daripada 60 minit tidak akan diberikan
pingat.
(i) If a participant is chosen at random, find the probability that the participant does not get the medal.
Jika seorang peserta dipilih secara rawak, cari kebarangkalian bahawa peserta itu tidak mendapat pingat.
(ii) The medals are only awarded to the first 30 participants that complete the run. If Johan completed his run
in 27 minutes, is he qualified for the medal?
Pingat hanya diberikan kepada 30 orang peserta terawal yang menamatkan larian. Jika Johan menamatkan lariannya dalam masa
27 minit, adalah dia layak mendapat pingat itu?
[5 marks/markah]

H O TS Zo n e
1 The lecture hours in a college start at 0830 hours. On a certain day, it is found that the arrival of students in the
college are normally distributed with a mean of 0815 hours and a standard deviation of 12 minutes.
Waktu kuliah di sebuah kolej bermula pada jam 0830. Pada suatu hari tertentu, didapati bahawa waktu ketibaan pelajar di kolej itu
bertaburan normal dengan min jam 0815 dan sisihan piawai 12 minit.
(a) Find the percentage of the students who are late to the college.
Cari peratus pelajar yang lewat ke kolej.
(b) Determine the time if 60% of the students have arrived in the college. Applying

Tentukan waktu jika 60% daripada pelajar telah tiba di kolej.

112

F5 Mod A+ ADDM 5(91-112).indd 112 5/1/2021 3:48:29 PM


Learning Area: Trigonometry

6 Trigonometric Functions
Fungsi Trigonometri
6.1 Positive Angles and Negative Angles / Sudut Positif dan Sudut Negatif

Smart Tip
1 Positive angle is an angle measured in the anticlockwise direction from the positive x-axis.
Sudut positif ialah suatu sudut yang diukur mengikut lawan arah jam dari paksi-x yang positif.
2 Negative angle is an angle measured in the clockwise direction from the positive x-axis.
Sudut negatif ialah suatu sudut yang diukur mengikut arah jam dari paksi-x yang positif.
y y

x
O θ°
θ°
x
O

Positive angle Negative angle


Sudut positif Sudut negatif

Exercise 1 Represent each of the following angles in Cartesian plane. Hence, determine which quadrant
each of the angles lies in the Cartesian plane.
Wakilkan setiap sudut yang berikut dalam satah Cartes. Seterusnya, tentukan sukuan yang manakah setiap sudut
itu berada dalam satah Cartes.
PL 1 Demonstrate the basic knowledge of trigonometric functions.

Example 1

p
(a) 600° (b) − 4 rad

Solution
(a) y (b)
y

x
60°
π x

4 4

600° − 360° = 240°


p
600° makes one anticlockwise rotation of 360° The angle − 4 lies in the fourth quadrant.
and another rotation of 240°. The angle 240° lies
in the third quadrant. Hence, 600° lies in the third −
p p
( 7p
4 is equivalent to 2p − 4 or 4 . )
quadrant. p
Sudut − terletak di sukuan empat.
Sudut 600° berputar mengikut lawan arah jam satu putaran 4
360° dan diikuti dengan putaran 240° lagi. 240° berada di
sukuan ketiga. Maka, 600° berada di sukuan ketiga.
p
− adalah setara dengan 2p −
4
p
4 (
atau
7p
4
. )

113

F5 Mod A+ ADDM 6(113-137).indd 113 5/1/2021 3:48:57 PM


11 2 –640°
1 p rad
5 y y
−640°

11π
5
π
5 80°
x x

11 p
p − 2p = 720° – 640° = 80°
5 5
First quadrant First quadrant

3 840° 4 – 7 p rad
y 6
y

120° 6

60° 7
π
x 6
x

7 5
840° – 720° = 120° 2p − p= p
6 6
Second quadrant Second quadrant

6.2 Trigonometric Ratios of Any Angle/ Nisbah Trigonometri bagi Sebarang Sudut

Smart Tip
y Adjacent side
Sisi bersebelahan x 1
cot θ/ kot θ = =
r
P(x, y) Opposite side y = tan θ
r Sisi bertentangan
y
θ°
r x
−r O x

−r
Hypotenus
Opposite side Hipotenus Opposite to θ
Sisi bertentangan y Bertentangan dengan θ
sin θ = =
Hypotenuse r θ°
Hipotenus
Adjacent to θ
Adjacent side Bersebelahan dengan θ
Sisi bersebelahan x
cos θ/ kos θ = =
Hypotenuse r Quadrant II/ Sukuan II Quadrant I/ Sukuan I
Hipotenus sin θ = sin(180° − θ) sin θ
Opposite side cos θ = − cos(180° − θ) cos θ/ kos θ
kos θ = − kos(180° − θ) tan θ
Sisi bertentangan y
tan θ = = tan θ = − tan(180° − θ)
Adjacent side x
Sisi bersebelahan
sin (+) All (+)/ Semua (+)
Hypotenuse
Hipotenus r 1 tan (+) cos (+)/ kos (+)
cosec θ/ kosek θ = =
Opposite side y = sin θ
Sisi bertentangan Quadrant III/ Sukuan III Quadrant IV/ Sukuan IV
sin θ = − sin (180° + θ) sin θ = − sin (360° − θ)
Hypotenuse cos θ = − cos (180° + θ) cos θ = cos (360° − θ)
Hipotenus r 1 kos θ = − kos (180° + θ) kos θ = kos (360° − θ)
sec θ/ sek θ = =
Adjacent side x = cos θ/kos θ tan θ = tan (180° + θ) tan θ = − tan (360° − θ)
Sisi bersebelahan

114

F5 Mod A+ ADDM 6(113-137).indd 114 5/1/2021 3:48:58 PM


Exercise 2 Solve each of the following without using a calculator.
Selesaikan setiap yang berikut tanpa menggunakan kalkulator.
PL 2 Demonstrate the understanding of trigonometric functions.

Example 2 24
1 Given cos θ = , where 180°  θ  360°, find the
25
3 value of
If sin θ = 5 , where 90°  θ 180°, find the value of 24
3 Diberi kos θ = , dengan keadaan 180°  θ  360°, cari
Jika sin θ = , dengan keadaan 90°  θ  180°, cari nilai 25
5 nilai
(a) tan θ, (b) sec θ/sek θ. (a) sin θ, (b) cot θ/ kot θ.
Solution y
y

24 x
5 O θ
3
θ 25
x
−4 O −7

By using Pythagoras’s theorem 7


Dengan menggunakan teorem Pythagoras (a) sin θ = –
25
3 3
(a) tan θ = –4 = − 4
7
4 (b) tan θ = –
(b) cos θ/kos θ = − 5 24
1
1 4 cot θ =
sec θ/sek θ = =−
5 7
4 – 24

5
24
=–
7

5 5
Given tan x =
12 and x is not in the first quadrant, Given tan x = − 7 , where 90°  x  270°, find the
2 3
find the value of value of
5 5
Diberi tan x = dan x tidak berada dalam sukuan pertama, Diberi tan x = – , dengan keadaan 90°  x  270°, cari nilai
12 7
cari nilai (a) cosec x/ kosek x, (b) sec x/sek x.
(a) sin x, (b) cosec x/kosek x. y
y
5
74
5
−12 x
x x
x O −7 O
13

−5 5
(a) sin x =
74
1
–5 cosec x =
(a) sin x = 13 5
74
5
= − 13 74
=
5
1
(b) cosec x = –7 7
5 (b) cos x = =–
– 13 74 74
1
13 sec x =
=− 7
5 –
74
74
=–
7

115

F5 Mod A+ ADDM 6(113-137).indd 115 5/1/2021 3:48:59 PM


Smart Tip
The formulae of the complementary angles/ Rumus sudut pelengkap:
• sin θ = cos (90° − θ)/ kos θ (90° − θ)
)/kos
• cos θ/ kos θ = sin (90° − θ)
/kos
• tan θ = cot (90° − θ)/ kot (90° − θ)
• cot θ/ kot θ = tan (90° − θ)
• sec θ = cosec (90° − θ)/ sek θ = kosek(90° − θ)
)/sek
• cosec θ = sec (90° − θ)/ kosek θ = sek(90° − θ)

Exercise 3 Given sin 50° = 0.766 and cos 50° = 0.6428, find the value of
Diberi sin 50° = 0.766 dan kos 50° = 0.6428, cari nilai
PL 2 Demonstrate the understanding of trigonometric functions.

Example 3
1 sin 40° 2 tan 40°
cos 40°/ kos 40°
sin 40° = cos (90° − 40°) tan 40° = cot (90° − 40°)
Solution = cos 50° = cot 50°
cos 40°/ kos 40° = 0.6428 cos 50°
= sin(90° − 40°) = sin 50°
= sin 50° 0.6428
= 0.766 = 0.766
= 0.8392

3 sec 40°/ sek 40° 4 cosec 40°/ kosek 40° 5 cot 40°/ kot 40°

sec 40° = cosec (90° − 40°) cosec 40˚ = sec (90˚ − 40˚) cot 40° = tan (90° − 40°)
= cosec 50° = sec 50˚ = tan 50°
1 1 sin 50°
= sin 50° = cos 50° = cos 50°
1 1 0.766
= 0.766 = 0.6428 = 0.6428
= 1.305 = 1.556 = 1.192

Smart Tip
The values of the trigonometric ratios of the special angles 30°, 45° and 60° are:
Nilai nisbah trigonometri bagi sudut-sudut khas 30°, 45° dan 60° ialah:

30° Ratio
2 2 Nisbah cos θ cosec θ sec θ cot θ
sin θ tan θ
3 Angle kos θ kosek θ sek θ kot θ
Sudut
60° 1 2
1 3
1 1 30° 2 3
2 2 3 3
1 1
45° 45° 1 2 2 1
2 2
2 3 1 2 1
1 60° 2
2 2 3 3 3
45°
1

116

F5 Mod A+ ADDM 6(113-137).indd 116 5/1/2021 3:48:59 PM


Exercise 4 Find the value of each of the following by using the values of the trigonometric ratios of the
special angles.
Cari nilai bagi setiap yang berikut dengan menggunakan nilai nisbah trigonometri bagi sudut-sudut khas.
PL 2 Demonstrate the understanding of trigonometric functions.

Example 4 1 cos 150°/ kos 150° 2 tan 225°

sin 210° cos 150° tan 225°


Solution = − cos (180° – 150°) = tan (225° – 180°)
sin 210° = − cos 30° = tan 45°
= −sin(210° − 180°) 3 =1
=−
= − sin 30° 2
1
=−2

3 cosec 135°/ kosek 135° 4 cot 45°/ kot 45° 4


5 sin 3 p
cosec 135° cot 45° = 1
= cosec (180° – 135°) 4
sin 3 p
= cosec 45° = sin 240°
= 2 = – sin (240° – 180°)
= – sin 60°
3
=−
2

6
4 4
cos 3 p/ kos 3 p 7 ( 5
) ( )
5
sec − 4 p / sek − 4 p
8 cosec (−300°)/ kosek (−300°)

cosec (−300°)
4
cos p
3 ( 5
sec − 4 p ) = cosec (–360 – (–300°))
= cosec (–60°)
= cos 240° = sec (–225°) = – cosec (60°)
= – cos (240° – 180°) = sec (–225° – (–180°)) 2
= – cos 60° = sec (–45°) =−
1 3
=− = – sec (45°)
2
=– 2

6.3 Graphs of Sine, Cosine and Tangent Functions / Graf Fungsi Sinus, Kosinus dan Tangen

Smart Tip
1 y = sin x 2 y = cos x/ kos x 3 y = tan x

y y y

1 1

x x x
0 0 90° 180° 270° 360° 0 90° 180° 270° 360°
90° 180° 270° 360°

1 1

x 0° 90° 180° 270° 360° x 0° 90° 180° 270° 360° x 0° 90° 180° 270° 360°
y 0 1 0 –1 0 y 1 0 –1 0 1 y 1 ∞ 0 ∞ 0

117

F5 Mod A+ ADDM 6(113-137).indd 117 5/1/2021 3:49:00 PM


Exercise 5 Draw the graph for each of the following for 0 < x < 2p.
Lukis graf bagi setiap yang berikut untuk 0 < x < 2p.
PL 3 Apply the understanding of trigonometric functions to perform simple tasks.

Example 5 1 y = cos x – 1/y = kos x – 1


y = − 4 sin x
p 3p
x 0 p 2p
Solution 2 2

p 3p y 0 –1 –2 –1 0
x 0 p 2p
2 2
y
y 0 –4 0 4 0

y
x
O   3 2
4
2 2

1
x
O π π 3π 2π
2 2
2
−4

2 y = tan x + 1 3 y = 3 sin x

p 3p p 3p
x 0 p 2p x 0 p 2p
2 2 2 2

y 1 ∞ 1 ∞ 1 y 0 3 0 –3 0

y y

1
x
x O π π 3π 2π
O π π 3π 2π 2 2
2 2
−3

4 y = − tan x 5 y = − 4 cos x/ y = − 4 kos x

p 3p p 3p
x 0 p 2p x 0 p 2p
2 2 2 2

y 0 ∞ 0 ∞ 0 y –4 0 4 0 –4

y y

x x
O   3 2 O π π 3π 2π
2 2 2 2

−4

118

F5 Mod A+ ADDM 6(113-137).indd 118 5/1/2021 3:49:01 PM


Exercise 6 Draw the graph for each of the following for 0 < x < 2p.
Lukis graf bagi setiap yang berikut untuk 0 < x < 2p.
PL 3 Apply the understanding of trigonometric functions to perform simple tasks.

Example 6
1 y = sin 2x
y = |cos 2x|/ y = |kos 2x| 2p
Period = 2 = p
Solution Number of cycles = 2
2p Amplitude = 1
Period/ Kala = 2 = p
Number of cycles/ Bilangan kitaran = 2
Amplitude/ Amplitud = 1 p p 3p 5p 3p 7p
x 0 p 2p
4 2 4 4 2 4
p p 3p 5p 3p 7p y 0 1 0 –1 0 1 0 –1 0
x 0 p 2p
4 2 4 4 2 4
y
y 1 0 1 0 1 0 1 0 1
1
y

x
1
O π π 3π π 5π 3π 7π 2π
4 2 4 4 2 4
−1

x
O   3  5 3 7 2
4 2 4 4 2 4

2 y = |cos x|/ y = |kos x| 3 y = |tan x|

p 3p p 3p
x 0 p 2p x 0 p 2p
2 2 2 2
y 1 0 1 0 1 y 0 ∞ 0 ∞ 0

y y

x
O π π 3π 2π
2 2

−1 x
O   3 2
2 2

4 y = |tan 2x| 5 y = −|cos 2x|/ y = −|kos 2x|


2p 2p
Period = 2 = p , Number of cycles = 2 Period = 2 = p , Number of cycles = 2

p p 3p 5p 3p 7p p p 3p 5p 3p 7p
x 0 p 2p x 0 p 2p
4 2 4 4 2 4 4 2 4 4 2 4
y 0 ∞ 0 ∞ 0 ∞ 0 ∞ 0 y –1 0 –1 0 –1 0 –1 0 –1
y y

x
O π π 3π π 5π 3π 7π 2π
4 2 4 4 2 4
O x −1
  3  5 3 7 2
4 2 4 4 2 4

119

F5 Mod A+ ADDM 6(113-137).indd 119 5/1/2021 3:49:02 PM


Exercise 7 Solve each of the following.
Selesaikan setiap yang berikut.
PL 3 Apply the understanding of trigonometric functions to perform simple tasks.

Example 7
1 Sketch the graph y = sin x for 0 < x < p. Then, on
the same axes, draw a suitable straight line to find
Sketch the graph y = cos x for 0 < x < 2p. Then, on the number of solutions for the equation
the same axes, draw a suitable straight line to find x
the number of solutions for the equation 2 sin x − p = 0.
x Lakarkan graf y = sin x untuk 0 < x < p. Seterusnya, pada
2 cos x = 1 − p .
paksi yang sama, lukis garis lurus yang bersesuaian untuk
Lakarkan graf y = kos x untuk 0 < x < 2p. Seterusnya, pada
paksi yang sama, lukis garis lurus yang bersesuaian untuk mencari bilangan penyelesaian bagi persamaan
x x
mencari bilangan penyelesaian bagi persamaan 2 kos x = 1 – p . 2 sin x − p = 0.

Solution y = sin x
y = cos x p
x 0 p
2
p 3p
x 0 p 2p y 0 1 0
2 2
y 1 0 –1 0 1 x
2 sin x − p = 0
x x
2 cos x = 1 − p sin x – 2p = 0
x 0 p x 0 p
1 x x
cos x = 2 − 2p sin x = 2p
1 1
1 x y 0 x y 0
2 y = 2p 2
y = 2 − 2p
y
y

1
1 y = cos x y = sin x x
1 y=
2 2
1
O x
π π 3π 2π 2
2 2 1 x
y= − x
2 2π
−1 O  
2
Number of solutions/ Bilangan penyelesaian = 2 Number of solutions = 2

2 Sketch the graph y = −tan x for 0 < x < 2p. Then, on p tan x + 2x = 0
the same axes, draw a suitable straight line to find 2x
tan x + p = 0
the number of solutions for the equation
p tan x + 2x = 0. 2x
p = – tan x x 0 p
Lakarkan graf y = −tan x untuk 0 < x < 2p. Seterusnya, pada 2x
paksi yang sama, lukis garis lurus yang bersesuaian untuk y= p
mencari bilangan penyelesaian bagi persamaan y 0 2
p tan x + 2x = 0.
y
2x
y = – tan x y=


p 3p
x 0 p 2p 2
2 2
y 0 ∞ 0 ∞ 0 x
O   3 2
2 2

Number of solutions = 3

120

F5 Mod A+ ADDM 6(113-137).indd 120 5/1/2021 3:49:03 PM


Sketch the graph y = 3 cos 2x for 0 < x < 2p. Then,
3 4 Sketch the graph y =|tan 2x| for 0 < x < 2p. Then,
on the same axes, draw a suitable straight line to on the same axes, draw a suitable straight line to
find the number of solutions for the equation find the number of solutions for the equation
x 1 2x
cos 2x = 3p − 3  . 2|tan 2x|= p – 4.
Lakarkan graf y = 3 kos 2x untuk 0 < x < 2p. Seterusnya, Lakarkan graf y =|tan 2x| untuk 0 < x < 2p. Seterusnya,
pada paksi yang sama, lukis garis lurus yang bersesuaian pada paksi yang sama, lukis garis lurus yang bersesuaian
untuk mencari bilangan penyelesaian bagi persamaan untuk mencari bilangan penyelesaian bagi persamaan
x 1 2x
kos 2x = −  .
3p 3
2|tan 2x| =
p
– 4.

y = 3 cos 2x y = |tan 2x|


p 3p p 3p
x 0 p 2p
x 0
2 
p
2 
2p 2  2 
y 3 –3 3 –3 3 y 0 0 0 0 0


x 1
cos 2x = 3p –  
3 x 0 p 2p

2x
2|tan 2x| = p – 4 (× 12  )
x x
3 cos 2x = p – 1 |tan 2x| = p – 2
x 0 p 2p
x y –1 0 1 x
y = p – 1 y = p – 2
y –2 –1 0
y
y
3 y = 3 cos 2x
y =|tan 2x|
2
x
y= 1
1
2
x
O   3 2 y= x –2
1 p
2 2 x
O   3 2
2 2 2
2
3

Number of solutions = 4 Number of solutions = 1

3
5 Sketch the graph y = 2 cos 2  x for 0 < x < p. Then,
3 1 x
on the same axes, draw a suitable straight line to cos 2  x =   – 2p
find the number of solutions for the equation 2 x 0 p 2p
3 1 x 3 x
cos 2  x = 2 − 2p . 2 cos 2  x = 1 – p
y 1 0 –1
3 x
Lakarkan graf y = 2 kos  x untuk 0 < x < p. Seterusnya, y = 1 – p
2
pada paksi yang sama, lukis garis lurus yang bersesuaian y
untuk mencari bilangan penyelesaian bagi persamaan
3 1 x 2
kos  x = − .
2 2 2p 3
1 y = 2 cos x
2
3
y = 2 cos 2  x
O x
π 2π π 4π 5π 2π
p 2p 4p 5p −1 3 3 3
x 0 p 2p 3
3  3 3 3 y=1− x
−2 π
y 2 0 –2 0 2 0 –2
Number of solutions = 3

121

F5 Mod A+ ADDM 6(113-137).indd 121 5/1/2021 3:49:03 PM


6.4 Basic Identities/ Identiti Asas

Smart Tip
1 sin2 θ + cos2 θ = 1 2 1 + tan2 θ = sec2 θ 3 1 + cot2 θ = cosec2 θ
sin2 θ + kos2 θ = 1 1 + tan2 θ = sek2 θ 1 + kot2 θ = kosek2 θ

Exercise 8 Solve the following trigonometric equations for 0° < x < 360°.
Selesaikan persamaan trigonometri yang berikut untuk 0° < x < 360°.
PL 3 Apply the understanding of trigonometric functions to perform simple tasks.

Example 8
1 2 cos2 x = 1 – sin x
2 kos2 x = 1 – sin x
tan2 x – 3 sec x = 3
tan2 x – 3 sek x = 3
2 cos2 x = 1 – sin x
2(1 – sin2 x) = 1 – sin x
Solution
2 – 2 sin2 x = 1 – sin x
tan2 x – 3 sec x = 3
2 sin x – sin x – 1 = 0
2
(sec x – 1) – 3 sec x – 3 = 0
2
(2 sin x + 1)( sin x – 1) = 0
sec2 x – 3 sec x – 4 = 0
2 sin x + 1 = 0 , sin x – 1 = 0
(sec x + 1)( sec x – 4) = 0
2 sin x = −1 , sin x = 1
sec x + 1 = 0 , sec x – 4 = 0 1
sec x = −1 , sec x = 4 sin x = − 2 ,
1 1 1
cos x = −1 , cos x = 41 When sin x = − 2
cos x = −1 , cos x = 4 x = 210°, 330°
When/ Apabila cos x = −1 When sin x = 1
x = 180° x = 90°
1 [ x = 90°, 210°, 330°
When/ Apabila cos x = ,
4
x = 75°31’, 284°29’
[ x = 75°31’, 180°, 284°29’

2 cot2 x – 5 cosec x + 7 = 0 3 sec2 x = tan x (3 tan x + 1)


kot2 x – 5 kosek x + 7 = 0 sek2 x = tan x (3 tan x + 1)

cot2 x – 5 cosec x + 7 = 0 sec2 x = tan x(3 tan x + 1)


(cosec2 x – 1) – 5 cosec x + 7 = 0 (1 + tan2 x) = 3 tan2 x + tan x
cosec2 x – 5 cosec x + 6 = 0 0 = 3 tan2 x + tan x – 1 – tan2 x
(cosec x − 2)( cosec x – 3) = 0 2 tan2 x + tan x – 1 = 0
cosec x – 2 = 0 , cosec x – 3 = 0 (2 tan x − 1)(tan x + 1) = 0
cosec x = 2 , cosec x = 3 2 tan x – 1 = 0 , tan x + 1 = 0
1 1 1
sin x = 2 , sin x = 3 tan x = 2 , tan x = −1
1
When sin x = 2 1
When tan x = 2
x = 30°, 150°
1 x = 26°34’, 206°34’
When sin x = 3 When tan x = −1
x = 19°28’, 160°32’ x = 135°, 315°
[ x = 19°28’, 30°, 150°, 160°32’ [ x = 26°34’, 135°, 206°34’, 315°

122

F5 Mod A+ ADDM 6(113-137).indd 122 5/1/2021 3:49:04 PM


Exercise 9 Prove each of the following trigonometric identities.
Buktikan setiap identiti trigonometri yang berikut.
PL 4 Apply appropriate knowledge and skills of trigonometric functions in the context of simple routine problem solving.

Example 9
1 7 – 6 sin2 θ = 1 + 6 cos2 θ
7 – 6 sin2 θ = 1 + 6 kos2 θ
sec2 x(cos2 x − cot2 x) − 1 = − cosec2 x
sek2 x(kos2 x − kot2 x) − 1 = − kosek2 x LHS:
7 – 6 sin2 θ
Solution = 7 – 6(1 – cos2 θ)
LHS/ Sebelah kiri: = 7 – 6 + 6 cos2 θ
sec2 x(cos2 x − cot2 x) − 1 = 1 + 6 cos2 θ (RHS)
1
( cos2 x
= cos2 x cos2 x − sin2 x − 1 )
1
=1– −1
sin2 x
= − cosec x (RHS/ Sebelah kanan)
2

2 (cos θ + sin θ)2 − 1 = 2 sin θ cos θ cos θ


3 = sec θ
(kos θ + sin θ)2 − 1 = 2 sin θ kos θ 1 – sin2 θ
kos θ
= sek θ
LHS: 1 – sin2 θ
(cos θ + sin θ)2 − 1
= cos2 θ + 2 sin θ cos θ + sin2 θ − 1 LHS:
cos θ
= (cos2 θ + sin2 θ) + 2 sin θ cos θ − 1
1 – sin2 θ
= (1) + 2 sin θ cos θ − 1 cos θ
= 2 sin θ cos θ (RHS) = cos2 θ
1
= cos θ
= sec θ (RHS)

4 (sec θ + 1)(sec θ − 1) = tan2 θ 5 3 + 2 cosec2 θ = 5 + 2 cot2 θ


(sek θ + 1)(sek θ − 1) = tan2 θ 3 + 2 kosek2 θ = 5 + 2 kot2 θ

LHS: LHS:
(sec θ + 1)(sec θ − 1) 3 + 2 cosec2 θ
= sec2 θ − 1 = 3 + 2 (cot2 θ + 1)
= (1 + tan2 θ ) − 1 = 3 + 2 cot2 θ + 2
= tan2 θ (RHS) = 5 + 2 cot2 θ (RHS)

1 7 tan2 x(cot2 x + 1) = sec2 x


6 – cos2 x = cot2 x(cos2 x)
tan2 x tan2 x(kot2 x + 1) = sek2 x
1
– kos2 x = kot2 x(kos2 x) LHS:
tan2 x
tan2 x(cot2 x + 1)
LHS:
1 – cos2 x
= tan2 x (1
tan2 x + 1 )
tan2 x tan2 x
cos2 x sin2 x cos2 x = + tan2 x
tan2 x
= −
sin x
2
sin2 x = 1 + tan2 x
cos x(1 – sin2 x)
2
= sec2 x (RHS)
=
sin2 x
= cot x(cos2 x) (RHS)
2

123

F5 Mod A+ ADDM 6(113-137).indd 123 5/1/2021 3:49:04 PM


8 cos x − sec x = −tan x sin x 1
cosec x – cot x = cosec x + cot x
9
kos x − sek x = −tan x sin x
1
= kosek x + kot x
LHS: kosek x – kot x
cos x − sec x LHS:
1 1
= cos x –
cos x cosec x – cot x
cos2 x – 1 1 cosec x + cot x
= =
cos x cosec x – cot x × cosec x + cot x
– sin2 x cosec x + cot x
= =
cos x cosec2 x – cot2 x
– sin x sin x cosec x + cot x
= =
cos x 1
= −tan x sin x (RHS) = cosec x + cot x (RHS)

6.5 Addition Formulae and Double Angle Formulae / Rumus Sudut Majmuk dan Rumus Sudut Berganda

Smart Tip
Addition formulae/ Rumus sudut majmuk:
1 sin ((A ± B) = sin A cos B ± cos A sin B/ sin (A ± B) = sin A kos B ± kos A sin B
2 cos ((A ± B) = cos A cos B sin A sin B/ kos (A ± B) = kos A kos B sin A sin B
tan A ± tan B tan A ± tan B
( ± B) =
3 tan (A / tan (A ± B) =
1 tan A tan B 1 tan A tan B

Exercise 10 Prove each of the following trigonometric identities.


Buktikan setiap identiti trigonometri yang berikut.
PL 4 Apply appropriate knowledge and skills of trigonometric functions in the context of simple routine problem solving..

Example 10 cos (A + B) – cos A cos B


1 = –tan B
sin (A + B) – cos A sin B
sin (A + B) + sin (A – B) kos (A + B) – kos A kos B
= tan A = –tan B
cos (A + B) + cos (A – B) sin (A + B) – kos A sin B
sin (A + B) + sin (A – B)
= tan A LHS:
kos (A + B) + kos (A – B)

Solution cos (A + B) – cos A cos B


LHS/ Sebelah kiri: sin (A + B) – cos A sin B
cos A cos B – sin A sin B – cos A cos B
sin (A + B) + sin (A – B) =
sin A cos B + cos A sin B – cos A sin B
cos (A + B) + cos (A – B) – sin A sin B
=
sin A cos B + cos A sin B + sin A cos B sin A cos B
– cos A sin B
= sin B
cos A cos B – sin A sin B + cos A cos B =–
+ sin A sin B cos B
2 sin A cos B = − tan B (RHS)
=
2 cos A cos B
sin A
=
cos A
= tan A (RHS/ Sebelah kanan)

124

F5 Mod A+ ADDM 6(113-137).indd 124 5/1/2021 3:49:05 PM


sin (A + B) – sin (A – B) 3 sin (45° − θ) + sin (45° + θ) = 2 cos θ
2 = − cot A
cos (A + B) – cos (A – B) sin (45° − θ) + sin (45° + θ) = 2 kos θ
sin (A + B) – sin (A – B)
= − kot A
kos (A + B) – kos (A – B) LHS:
LHS: sin (45° − θ) + sin (45° + θ)
= sin 45° cos θ − cos 45° sin θ + sin 45° cos θ +
sin (A + B) – sin (A – B)
= cos 45° sin θ
cos (A + B) – cos (A – B)
= 2 sin 45° cos θ
sin A cos B + cos A sin B –
(sin A cos B – cos A sin B) =2 ( )
1
2
cos θ
=
cos A cos B – sin A sin B –
(cos A cos B + sin A sin B) =2 ( 1
2
×
2
2
)
cos θ
sin A cos B + cos A sin B –
sin A cos B + cos A sin B
= cos A cos B – sin A sin B –
=2 ( )
2
2
cos θ

cos A cos B – sin A sin B = 2 cos θ (RHS)


2 cos A sin B
=
–2 sin A sin B
cos A
=–
sin A
= − cot A (RHS)

cos b + sin b cot A cot B – 1


4 tan (b + 45°) = 5 cot (A + B) =
cos b – sin b cot A + cot B
kos b + sin b kot A kot B – 1
tan (b + 45°) = kot (A + B) =
kos b – sin b kot A + kot B
LHS: LHS:
tan (b + 45°) cot (A + B)
tan b + tan 45° cos (A + B)
= =
1 – tan b tan 45° sin (A + B)
tan b + 1 cos A cos B – sin A sin B
= =
1 – tan b(1) sin A cos B + cos A sin B
1 + tan b 1
= cos A cos B – sin A sin B sin A sin B
1 – tan b =
sin A cos B + cos A sin B × 1
sin b
1 + cos b sin A sin B
= sin A cos B + cos A sin B
sin b =
1– cos A cos B
cos b
cos b + sin b cos A cos B – sin A sin B
cos b sin A sin B
= =
cos b – sin b sin A cos B + cos A sin B
cos b sin A sin B
cos b + sin b cos b cos A cos B sin A sin B
= × cos b – sin b –
cos b sin A sin B sin A sin B
=
cos b + sin b sin A cos B cos A sin B
= (RHS) +
cos b – sin b sin A sin B sin A sin B
cot A cot B – 1
=
cot B + cot A
cot A cot B – 1
= (RHS)
cot A + cot B

125

F5 Mod A+ ADDM 6(113-137).indd 125 5/1/2021 3:49:05 PM


6 ( p
) p
(
sin 4θ − 3 + sin 4θ + 3 = sin 4θ ) 7
sin (A + B)
cos A cos B
= tan A + tan B

LHS: sin (A + B)
= tan A + tan B
kos A kos B
(
sin 4θ −
p
)
3 + sin 4θ + 3
p
( ) LHS:

( p
= sin 4θ cos − cos 4θ sin
3
p
3 )
+ sin (A + B)
cos A cos B

( p
sin 4θ cos 3 + cos 4θ sin
p
3 ) =
sin A cos B + cos A sin B
cos A cos B
p sin A cos B cos A sin B
= 2 sin 4θ cos =
3 cos A cos B + cos A cos B
= 2 sin 4θ
1
2 () =
sin A sin B
cos A + cos B
= sin 4θ (RHS) = tan A + tan B (RHS)

Exercise 11 Simplify each of the following as a single trigonometric ratio.


Permudahkan setiap yang berikut kepada nisbah trigonometri tunggal.
PL 3 Apply the understanding of trigonometric functions to perform simple tasks.

Example 11 1 cos 50° cos 36° + sin 50° sin 36°


kos 50° kos 36° + sin 50° sin 36°
sin 60°cos 20° + cos 60° sin 20°
sin 60° kos 20° + kos 60° sin 20°
cos 50° cos 36° + sin 50° sin 36°
= cos (50° − 36°)
Solution
= cos 14°
sin 60° cos 20° + cos 60° sin 20°
= sin (60° + 20°)
= sin 80°

2 sin 50° cos 40° − cos 50° sin 40° 3 cos 55° cos 15° − sin 55° sin 15°
sin 50° kos 40° − kos 50° sin 40° kos 55° kos 15° − sin 55° sin 15°

sin 50° cos 40° − cos 50° sin 40° cos 55° cos 15° − sin 55° sin 15°
= sin (50° − 40°) = cos (55° + 15°)
= sin 10° = cos 70°

tan 32° + tan 24° tan 72° – tan 23°


4 5
1 – tan 32° tan 24° 1 + tan 72° tan 23°
tan 32° + tan 24° tan 72° – tan 23°
1 – tan 32° tan 24° 1 + tan 72° tan 23°
= tan (32° + 24°) = tan (72° – 23°)
= tan 56° = tan 49°

126

F5 Mod A+ ADDM 6(113-137).indd 126 5/1/2021 3:49:05 PM


Exercise 12 Without using a calculator, solve each of the following.
Tanpa menggunakan kalkulator, selesaikan setiap yang berikut.
PL 3 Apply the understanding of trigonometric functions to perform simple tasks.

Example 12
3 12
Given sin A = and tan B = − 5 , where A is an acute angle and B is an obtuse angle. Find the value of
5
3 12
Diberi sin A = dan tan B = − , dengan keadaan A ialah sudut tirus dan B ialah sudut cakah. Cari nilai
5 5
(a) cos (A + B)/ kos (A + B),
(b) tan (A – B).
Solution (a) cos(A + B)
y = cos A cos B − sin A sin B

sin A =
3
5
= ( )( ) ( )( )
4
5

5
13

3 12
5 13
5 4 20 36
3 cos A = =− −
5 65 65
A 56
x 3 =−
O 4 tan A = 65
4
y
12
sin B = (b) tan(A – B)
13
13 5 tan A – tan B
12 cos B = − =
13 1 + tan A tan B
() ( )
B x 12 3 12
−5 O tan B = − – −
5 = 4 5
1+ ( )( )
3
4

12
5
63
=−
16

4 5
1 Given sin A = and tan B = 12 , where A and B are acute angles. Find the value of
5
4 5
Diberi sin A = dan tan B = , dengan keadaan A dan B ialah sudut tirus. Cari nilai
5 12
(a) tan (A + B),
(b) sin (B − A).

y (a) tan (A + B)
4 tan A + tan B
sin A = =
5 1 – tan A tan B
5 3 4 5
4 cos A = +
A
5 3 12
x 4 =
O 3 tan A =
3 1– ( )( )
4 5
3 12
7
= 4
y 4
5 9
sin B = 63
13 =
13 12 16
5 cos B =
B 13 (b) sin (B – A)
x
O 12 5 = sin B cos A − cos B sin A
tan B =
12
= ( )
5 3
13 5
− ( )
12 4
13 5
15 48
= −
65 65
33
=−
65

127

F5 Mod A+ ADDM 6(113-137).indd 127 5/1/2021 3:49:06 PM


3 12
2 Given cos R =– for 180° < R < 360° and cos Q = for 180° < Q < 360°, find the value of
5 13
3 12
Diberi kos R = − bagi 180° < R < 360° dan kos Q = bagi 180° < Q < 360°, cari nilai
5 13
(a) sin (R + Q),
(b) cos (R + Q)./ kos (R + Q).

y (a) sin (R + Q)
= sin R cos Q + cos R sin Q

3
sin R = –
4
5
= − ( )( ) ( )( )
4 12
5 13
+ −
3
5

5
13
x
R O 3 48 15
cos R = – =− +
4
5 5 65 65
33
=−
65
(b) cos (R + Q)
y
= cos R cos Q – sin R sin Q

12
= − ( )( ) ( )( )
3 12
5 13
– −
4
5

5
13
x 5 36 20
O Q sin Q = – =− –
5
13 65 65
13 12 56
cos Q =
13 =−
65

3 Given cos θ = m for 0° < θ < 90°, express in terms (a) cos (90° − θ)
of m, = cos 90°cos θ + sin 90° sin θ
Diberi kos θ = m bagi 0° < θ < 90°, ungkapkan dalam sebutan = sin θ
m, = 1 – m2
(a) cos(90°− θ)/ kos(90°− θ),
(b) cot θ/ kot θ. (b) cot θ
1
y =
tan θ
cos θ = m 1
sin θ = 1 – m2 =
1 1  m2
tan θ =
1 – m2 1 – m2
m

θ° m
x m
O m =
1 – m2

Smart Tip
Double-angle formulae: Half-angle formulae:
Rumus sudut berganda: Rumus sudut separuh:
1 sin 2A = 2 sin A cos A A A
1 sin A = 2 sin cos
sin 2A = 2 sin A kos A 2 2
A A
2 cos 2A = cos2 A – sin2 A sin A = 2 sin kos
2 2
kos 2A = kos A – sin A
2 2

cos 2A = 2 cos2 A – 1 A A A A
2 cos A = cos2 − sin2 / kos A = kos2 2 − sin2 2
kos 2A = 2 kos A – 12 2 2
A A
cos 2A = 1 – 2 sin2 A = 2 cos2 − 1 = 2 kos2 − 1
2 2
kos 2A = 1 – 2 sin2 A A 2A
= 1 − 2 sin2 = 1 − 2 sin
2 tan A 2 2
3 tan 2A =
1 – tan2 A A
2 tan
2
3 tan A =
A
1 – tan 2
2

128

F5 Mod A+ ADDM 6(113-137).indd 128 5/1/2021 3:49:07 PM


Exercise 13 Prove each of the following trigonometric identities.
Buktikan setiap identiti trigonometri yang berikut.
PL 4 Apply appropriate knowledge and skills of trigonometric functions in the context of simple routine problem solving.

Example 13 1 sin 2A = 2 sin A cos A


sin 2A = 2 sin A kos A
cos 2A = 1 – 2 sin2 A
kos 2A = 1 – 2 sin2 A LHS:
sin 2A
Solution = sin (A + A)
LHS/ Sebelah kiri: = sin A cos A + cos A sin A
cos 2A = 2 sin A cos A (RHS)
= cos (A + A)
= cos A cos A – sin A sin A
= cos2 A – sin2 A
= (1 – sin2 A) – sin2 A
= 1 – 2 sin2 A (RHS/ Sebelah kanan)

2 tan A cot2 A – 1 kot2 A – 1


2 tan 2A = 3 cot 2A = / kot 2A = 2 kot A
1 – tan2 A 2 cot A
LHS: LHS:
tan 2A 1
= tan (A + A) cot 2A =
tan 2A
tan A + tan A 1
= =
1 – tan A tan A 2 tan A
=
2 tan A
(RHS) 
1 – tan2 A 
1 – tan2 A
1 – tan2 A
=
2 tan A
1
1 – tan2 A tan2 A
= ×
2 tan A 1
tan2 A
1
–1
= tan 2
A
2
tan A
cot2 A – 1
= (RHS)
2 cot A

Exercise 14 Prove each of the following trigonometric identities.


Buktikan setiap identiti trigonometri yang berikut.
PL 4 Apply appropriate knowledge and skills of trigonometric functions in the context of simple routine problem solving.

Example 14 sin 2θ sin 2θ


1 = tan θ/ 1 + kos 2θ = tan θ
1 + cos 2θ
tan θ + cot θ = 2 cosec 2θ
tan θ + kot θ = 2 kosek 2θ LHS:
Solution sin 2θ 2 sin θ cos θ
=
LHS/ Sebelah kiri: 1 + cos 2θ 1 + (2 cos2 θ – 1)
sin θ cos θ 2 sin θ cos θ
tan θ + cot θ = + =
cos θ sin θ 2 cos2θ
sin2 θ + cos2 θ sin θ
= =
sin θ cos θ cos θ
1 = tan θ (RHS)
=
1
2 sin 2θ
= 2 cosec 2θ (RHS/ Sebelah kanan)

129

F5 Mod A+ ADDM 6(113-137).indd 129 5/1/2021 3:49:07 PM


2 sin θ 2 sin θ 3 cosec θ sec θ = 2 cosec 2θ/ kosek θ sek θ = 2 kosek 2θ
2 = tan 2θ/ = tan 2θ
cos θ(2 – sec2 θ) kos θ(2 – sek2 θ)
LHS:
LHS:
2 sin θ 2 sin θ
cos θ

1
cosec θ sec θ =
sin θ
×
1
cos θ
=
2 – sec2 θ
cos θ (2 – sec2 θ) 1
2 (tan θ) =
sin θ cos θ
=
2 – (1 + tan2 θ) 1
2 tan θ =
1
=
1 – tan2 θ 2  sin 2θ
= tan 2θ (RHS) = 2 cosec 2θ (RHS)

1 – cos 2θ 1 – kos 2θ cos θ – sin θ 1


4 sec θ = / sek θ = sin θ sin 2θ 5 =
sin θ sin 2θ cos 2θ cos θ + sin θ
kos θ – sin θ 1
RHS: =
kos 2θ kos θ + sin θ
1 – cos 2θ 1 – (1 – 2 sin2 θ)
= LHS:
sin θ sin 2θ sin θ (2 sin θ cos θ) cos θ – sin θ cos θ – sin θ
1 – 1 + 2 sin2 θ =
= cos 2θ cos2 θ – sin2 θ
2 sin2 θ cos θ
cos θ – sin θ
2 sin2 θ =
= (cos θ + sin θ) (cos θ – sin θ)
2 sin2 θ cos θ 1
1 = (RHS)
= cos θ + sin θ
cos θ
= sec θ (LHS)

6 cos 2θ = cos4 θ – sin4 θ 2 tan θ


7 = sin 2θ
kos 2θ = kos4 θ – sin4 θ 1 + tan2 θ

RHS: LHS:
sin θ
cos4 θ – sin4 θ = (cos2 θ + sin2 θ)(cos2 θ – sin2 θ) 2 tan θ 2 cos θ

= (1)(cos 2θ) =
1 + tan2 θ sec2 θ
= cos 2θ (LHS) sin θ
2 cos θ

=
1
 cos 2 
θ

sin θ
=2 cos θ
 × cos2 θ
= 2 sin θ cos θ
= sin 2θ (RHS)

130

F5 Mod A+ ADDM 6(113-137).indd 130 5/1/2021 3:49:08 PM


Exercise 15 Solve each of the following.
Selesaikan setiap yang berikut.
PL 3 Apply the understanding of trigonometric functions to perform simple tasks.

Example 15

3
Given cos θ = − , where θ is an obtuse angle. Find the value of
5
3
Diberi kos θ = − , dengan keadaan θ ialah sudut cakah. Cari
5
1 1
(a) sin 4θ, (b) cos θ./ kos θ.
2 2

Solution θ
(b) 2 cos2 − 1 = cos θ
y 2
θ cos θ + 1
4 cos2 =
sin θ = 2 2
5 θ cos θ + 1
5 3 cos =
4 cos θ = − 2 2
θ 5
(− 35 ) + 1
x
−3 O

=
2
(a) sin 4θ
= 2 sin 2θ cos 2θ ( 25 )
= 2 (2 sin θ cos θ)(2 cos2 θ − 1) =
2
=2 2
4
5[ ( )( )][ ( ) ]

3
5
2−
3 2
5
−1
=
1
5
=2 −
24
25( )( )

7
25
336
=
625

12 35
1 Given tan x = for 0° < x < 90°, find 2 Given cos A = where A is an acute angle. Find
5 37 ,
12 the value of
Diberi tan x = bagi 0° < x < 90°, cari 35
5 Diberi kos A = , dengan keadaan A ialah sudut tirus.
(a) sin 2x, (b) tan 2x. 37
Cari nilai
y A A A
12 (a) cos / kos , (b) sin .
sin x = 2 2 2
13
5 A
13 cos x =
12 13 (a) cos A = 2 cos2 −1
2 y
x 12
O
x tan x = A cos A + 1
5 5 cos =
2 2
37
( )
12
35
(a) sin 2x = 2 sin x cos x +1 A
= 37
( )( )
12 5 x
O 35
=2 2
13 13 6
= 12
120 37 sin A =
= 37
169
A A 35
2 tan x (b) sin A = 2 sin cos cos A =
2 2 37
(b) tan 2x =
1 – tan2 x
( )
12 A 6
12 = 2 sin
2
5
  37 2 37
= A 12 1 37
12 2
1–
5
  sin
2
=
37 2
× ×
6
24 37
5
  =
37
= 1
119 =

25 37
120
=–
119

131

F5 Mod A+ ADDM 6(113-137).indd 131 5/1/2021 3:49:09 PM


θ 3
3 Given sin = t, where θ is an obtuse angle. Express 4 Given sin x = , where x is an obtuse angle. Find
2 5
in terms of t 3
Diberi sin x =
, dengan keadaan x ialah sudut cakah. Cari
θ 5
Diberi sin = t, dengan keadaan θ ialah sudut cakah. (a) cos 2x,/ kos 2x,
2
Ungkapkan dalam sebutan t (b) cos 4x,/ kos 4x,
(a) cosec θ,/ kosek θ, (b) tan θ. 1
(c) cos x./ kos 1 x.
y 2 2
θ
sin =t y
2
θ 3
cos
t
1
= 1 – t2 sin x =
θ 2 5
5
3 4
2 x θ t cos x x =–
tan
1  t2
O = 5
2 1 – t2 x
4 O

(a) cosec θ
1 (a) cos 2x = 2 cos2 x – 1
=
sin θ
=
1 = 2 – ( )
4 2
5
– 1
θ θ
2 sin cos
2 2 = 32 – 1
25
1
=
2(t)( 1 – t2 ) = 7
25
1
= (b) cos 4x = 2 cos2 (2x) – 1
2t( 1 – t2 )
( )
= 2 7 – 1
2

25
(b) tan θ
θ = 98 – 1
2 tan 625
2
=
θ = – 527
1 – tan2 625
2

2( t
1 – t2 ) x
− 1 = cos x
(c) 2 cos2
2
=
( t
)
2
1– x cos x + 1
1–t 2 cos2 =
2 2
( 2t
1 – t2 ) x
cos =
2
cos x+1
2
=
t2
1–
1 – t2

=

4
5
+1

( )

( 2t
1 – t2 ) 2

=
1 – 2t2
=
( 15 )
1 – t2 2
2t 1 – t2
= × 1
1–t 2
1 – 2t2 =

10
2t(1 – t2)
=
(1 – 2t2) 1 – t2
2t(1 – t2) 1 – t2
= ×
(1 – 2t2) 1 – t2 1 – t2
2t(1 – t2) 1 – t2
=
(1 – 2t2)(1 – t2)
2t 1 – t2
=
1 – 2t2

132

F5 Mod A+ ADDM 6(113-137).indd 132 5/1/2021 3:49:10 PM


6.6 Application of Trigonometric Functions / Aplikasi Fungsi Trigonometri

Exercise 16 Solve each of the following equations for 0 < x < 360°.
Selesaikan setiap persamaan yang berikut bagi 0 < x < 360°.
PL 3 Apply the understanding of trigonometric functions to perform simple tasks.

Example 16
1 2 sin 2x = cos x
2 sin 2x = kos x
2 cos 2x – 4 sin x + 1 = 0/ 2 kos 2x – 4 sin x + 1 = 0
2 sin 2x = cos x
Solution
2 sin 2x – cos x = 0
2 cos 2x – 4 sin x + 1 = 0
2(2 sin x cos x) – cos x = 0
2(1 – 2 sin² x) – 4 sin x + 1 = 0
4 sin x cos x – cos x = 0
2 – 4 sin² x – 4 sin x + 1 = 0
cos x (4 sin x – 1) = 0
4 sin² x + 4 sin x − 3 = 0
cos x = 0 or 4 sin x – 1 = 0
(2 sin x − 1)(2 sin x + 3) = 0 1
2 sin x – 1 = 0 or/atau 2 sin x + 3 = 0 sin x = 4
1 3 When cos x = 0
sin x = 2 sin x = − 2
1 x = 90°, 270°
When/ Apabila sin x = 2 1
When sin x = 4
x = 30°, 150°
3 x = 14°29’, 165°31’
When sin x = − 2 , x has no value.
[ x = 14°29’, 90°, 165°31’, 270°
3
Apabila sin x = − , x tidak mempunyai nilai.
2
[ x = 30°, 150°

2 3 sin 2x = sin x 3 cos 2x + cos x = 0


kos 2x + kos x = 0
3 sin 2x = sin x
3 sin 2x – sin x = 0 cos 2x + cos x = 0
3(2 sin x cos x) – sin x = 0 (2 cos² x − 1) + cos x = 0
6 sin x cos x – sin x = 0 2 cos² x + cos x – 1 = 0
sin x (6 cos x – 1) = 0 (2 cos x − 1) (cos x + 1) = 0
sin x = 0 or 6 cos x – 1 = 0 2 cos x − 1 = 0 or cos x + 1 = 0
1 1
cos x = cos x = 2 cos = −1
6
When sin x = 0 1
When cos x = 2
x = 0°, 180°, 360°
1 x = 60°, 300°
When cos x = When co = −1
6
x = 80°24’, 279°36’ x = 180°
[ x = 0°, 80°24’, 180°, 279°36’, 360° x
[ = 60°, 180°, 300°

4 3 cos 2x – 4 sin x = 1 5 tan 2x + tan x = 0


3 kos 2x – 4 sin x = 1
tan 2x + tan x = 0
3 cos 2x – 4 sin x = 1 2 tan x
3(1 − 2 sin2 x ) – 4 sin x = 1 1 − tan2 x + tan x = 0
3 − 6 sin2 x – 4 sin x = 1 2 tan x + tan x(1 – tan2 x) = 0
6 sin2 x + 4 sin x – 2 = 0 ( 2) 2 tan x + tan x − tan3 x = 0
3 sin2 x + 2 sin x – 1 = 0 3 tan x − tan3 x = 0
(3 sin x − 1)(sin x + 1) = 0 tan3 x – 3 tan x = 0
3 sin x – 1 = 0 or sin x + 1 = 0 tan x(tan2 x − 3) = 0
1 sin x = −1 tan x = 0 or tan2 x − 3 = 0
sin x = 3 tan x = 3 or tan x = − 3
1
When sin x = 3 When tan x = 0
x = 19°28’, 160°32’ x = 0°, 180°, 360°
When sin x = −1 When tan x = 3
x = 270° x = 60°, 240°
[ x = 19°28’, 160°32’, 270° When tan x = − 3
x = 120°, 300°
[ x = 0°, 60°, 120°, 180°, 240°, 300°, 360°

133

F5 Mod A+ ADDM 6(113-137).indd 133 5/1/2021 3:49:10 PM


6 cos 2x + 3 cos x = 1 7 3 cos 2x + 8 sin x + 5 = 0
kos 2x + 3 kos x = 1 3 kos 2x + 8 sin x + 5 = 0

cos 2x + 3 cos x = 1 3 cos 2x + 8 sin x + 5 = 0


(2 cos2 x – 1) + 3 cos x – 1 = 0 3(1 – 2 sin2 x) + 8 sin x + 5 = 0
2 cos2 x + 3 cos x – 2 = 0 3 – 6 sin2 x + 8 sin x + 5 = 0
(2 cos x − 1)(cos x + 2) = 0 6 sin2 x − 8 sin x – 8 = 0 ( 2)
2 cos x – 1 = 0 or cos x + 2 = 0 3 sin2 x − 4 sin x – 4 = 0
1 (3 sin x + 2)(sin x – 2) = 0
cos x = 2 cos x = −2
3 sin x + 2 = 0 or sin x – 2 = 0
1 2
When cos x = 2 sin x = − 3 sin x = 2
x = 60°, 300° 2
When cos x = −2 , x has no value. When sin x = − 3
[ x = 60°, 300° x = 221°49’, 318°11’
PAK-21 When sin x = 2, x has no value.
[ x = 221°49’, 318°11’
VIDEO

Exercise 17 Solve each of the following problems.


Selesaikan setiap masalah yang berikut.
PL 4 Apply appropriate knowledge and skills of trigonometric functions in the context of simple routine problem solving.

Example 17
y
u
The diagram shows a particle is projected with a speed u m s at an acute angle −1

θ° from the horizontal ground. The range of the displacement of the particle, Sx,
is given by Sx = u2 sin θ cos θ.
Rajah menunjukkan satu zarah dilancarkan dengan kelajuan u m s−1 pada sudut tirus θ° dari
permukaan tanah mengufuk. Julat bagi sesaran zarah, Sx, diberi oleh Sx = u2 sin θ kos θ. θ
x
O
If Sx = 170 and u = 20, determine the values of θ.
Jika Sx = 170 dan u = 20, tentukan nilai-nilai θ.

Solution
Sx = u2 sin θ cos θ
1
(
= u2 sin 2θ
2 )
u2
= sin 2θ
2
When/Apabila Sx = 170, u = 20
202
170 = sin 2θ
2
170 × 2
sin 2θ = Video
202
= 0.85 Scan QR code or visit https://youtu.
2θ = 58.21°, 121.79° be/7Eo-fuy0f7g to watch an example
of using double angle identities to
θ = 29.11°, 60.90° solve equations.
For educational purposes only

134

F5 Mod A+ ADDM 6(113-137).indd 134 5/1/2021 3:49:11 PM


1 The diagram shows a particle is projected with a velocity of v m s−1 from a y
fixed point O at the horizontal ground. The angle of the projection from the v
x-axis is θ, where θ is an acute angle. After t seconds, the particle is at the point
P(20, 140). Given the motion of the particle satisfies the equation y = x tan θ +
gx2 P(20, 140)
sec² θ, where g m s−2 is the gravitational acceleration. If v = 10 and g = 10,
2v2 θ
determine the value of θ. x
O A(24, 0)
Rajah menunjukkan suatu zarah yang dilancarkan dengan halaju v m s dari suatu titik tetap
−1

O pada permukaan tanah mengufuk. Sudut pelancaran dari paksi-x ialah θ, dengan keadaan θ
ialah sudut tirus. Selepas t saat, zarah itu berada di titik P(20, 140). Diberi gerakan zarah itu
gx2
memenuhi persamaan y = x tan θ + sek² θ, dengan keadaan g m s−2 ialah pecutan graviti.
2v2
Jika v = 10 dan g = 10, tentukan nilai θ.

gx2
y = x tan θ + sec² θ
2v2
(10)(20)²
140 = 20 tan θ + (1 + tan2 θ)
2(10)²
140 = 20 tan θ + 20 (1 + tan2 θ)
140 = 20 tan θ + 20 + 20 tan2 θ
0 = 20 tan2 θ + 20 tan θ − 120
= tan2 θ + tan θ − 6 = 0
(tan θ − 2)(tan θ + 3) = 0
tan θ − 2 = 0 or tan θ + 3 = 0
tan θ = 2 tan θ = −3
Since 0° < θ < 90°
tan θ = 2
θ = 63°26’

2 The diagram shows a water wheel with centre P. P is h m above the water level.
The position of point Q on the water wheel is represented by the equation
p Q
y = − 6 sin t + 5, where t is the time taken for the wheel to complete one cycle,
8
in seconds and y is the distance from the water level, in metre. P Pusat
Centre
Rajah menunjukkan sebuah roda air berpusat P. P ialah h m di atas paras air. Kedudukan hm
p
titik Q pada roda air itu diwakili oleh persamaan y = − 6 sin 8 t + 5, dengan keadaan t ialah
masa yang diambil oleh roda air untuk membuat satu pusingan lengkap, dalam saat dan Paras air
Water level
y ialah jarak dari paras air, dalam meter.
Determine/ Tentukan
(a) the value of h,
nilai h,
(b) the diameter, in m, of the water wheel,
diameter, dalam m, bagi roda air,
(c) the maximum distance, in m, of point Q from the water level,
jarak maksimum, dalam m, bagi titik Q dari paras air,
(d) the time taken, in seconds, for the wheel to make a complete cycle.
masa yang diambil, dalam saat, bagi roda itu untuk membuat satu putaran lengkap.

(a) h = 5 m

(b) Amplitude = 6 m
[ Diameter = 2 × 6 m = 12 m

(c) 6 m + 5 m = 11 m
p
t = 2p
(d)
8
8
t = 2p × p
= 16 seconds

135

F5 Mod A+ ADDM 6(113-137).indd 135 5/1/2021 3:49:11 PM


Review 6
Paper 1

Section A (a) cos θ,


1 Solve each of the following trigonometric equations for kos θ,
0° < x < 360°. (b) cosec 2θ,
Selesaikan setiap persamaan trigonometri yang berikut bagi kosek 2θ,
0° < x < 360°. (c) tan 2θ.
(a) 2 sin 2x = 1, [5 marks/markah]
(b) 2 sin 2x = cos x./2 sin 2x = kos x.
[5 marks/markah] 1
5 Given sin θ = for 90° < θ < 180°. Find the value of
10
2 Given cos θ = m, express each of the following in terms 1
Diberi sin θ = untuk 90° < θ < 180°. Cari nilai
of m, 10
(a) tan θ,
Diberi kos θ = m, ungkapkan setiap yang berikut dalam sebutan
m, (b) tan (θ + 225°).
(a) cosec θ,/ kosek θ, [5 marks/markah]
(b) cot θ,/ kot θ,
(b) sin 2θ. Section B
[5 marks/markah] 6 (a) Solve the trigonometric equation 6 cot θ = 7 – tan θ
for 0° < θ < 360°.
Selesaikan persamaan trigonometri 6 kot θ = 7 – tan θ
3 (a) Given tan b = p, express tan (60° − b) in terms of p.
untuk 0° < θ < 360°.
Diberi tan b = p, ungkapkan tan (60° − b) dalam sebutan p.
[4 marks/markah]
[2 marks/markah]
5 (b) Solve 2 sin x + cos x = 0 for 0° < x < 360°.
(b) Diagram 1 shows a part of the graph y = 3 cos 3x. Selesaikan 2 sin x + kos x = 0 untuk 0° < x < 360°.
5 [4 marks/markah]
Rajah 1 menunjukkan sebahagian daripada graf y = kos 3x.
3 x
y 7 (a) Express sin 2 in terms of cos x, where x is an
2– 2
acute angle. Hence, show that sin 22.5° = .
2
x
Ungkapkan sin dalam sebutan kos x, dengan keadaan x
2– 2 2
2 0
x ialah sudut tirus. Seterusnya, tunjukkan bahawa sin 22.5°
k
2– 2
= .
2
h
Diagram 1 / Rajah 1 [4 marks/markah]
(b) Solve the equation 2 sec2 x = 3 + tan x for
State the value of h and of k. 0° < x < 360°.
Nyatakan nilai h dan nilai k. Selesaikan persamaan 2 sek2 x = 3 + tan x untuk
[2 marks/markah] 0° < x < 360°.
[4 marks/markah]
1
4 Given tan θ = h , where 0° < θ < 90°. Find in
terms of t,
1
Diberi tan θ = , dengan keadaan 0° < θ < 90°. Cari dalam
h
sebutan t,

Paper 2
Section A
3
1 (a) Sketch the graph of y = 4 sin x for 0 < x < 2p.
2
3
Lakarkan graf y = 4 sin x bagi 0 < x < 2p.
2
[4 marks/markah]
(b) Hence, on the same axes, draw a suitable straight line to find the number of solutions for the equation
3
12p sin x = 6x − 3p for 0 < x < 2p.
2
Seterusnya, menggunakan paksi yang sama, lukis satu garis lurus yang bersesuaian untuk mencari bilangan penyelesaian bagi persamaan
3
12p sin x = 6x − 3p bagi 0 < x < 2p.
2
[3 marks/markah]

136

F5 Mod A+ ADDM 6(113-137).indd 136 5/1/2021 3:49:11 PM


Section B
x sin x
2 (a) (i) Prove that cot =
2 1 – cos x
x sin x
Buktikan bahawa kot =
2 1 – kos x
(ii) Hence, find the value of cot 45° without using a calculator.
Seterusnya, cari nilai bagi kot 45° tanpa menggunakan kalkulator.
[4 marks/markah]
5
(b) (i) Sketch the graph of y = − sin x for 0 < x < 2p.
2
5
Lakarkan graf y = − sin x bagi 0 < x < 2p.
2
(ii) Hence, on the same axes, draw a suitable straight line to find the number of solutions for the equation
5 x
sin x + 2p + 1 = 0 for 0 < x < 2p.
2
Seterusnya, menggunakan paksi yang sama, lukis satu garis lurus yang bersesuaian untuk mencari bilangan penyelesaian bagi
5 x
persamaan sin x + 2p + 1 = 0 bagi 0 < x < 2p.
2
[6 marks/markah]

3 (a) Prove that 6 tan θ (1 – sin2 θ) = 3 sin 2θ.


Buktikan bahawa 6 tan θ (1 – sin2 θ) = 3 sin 2θ.
Hence, solve the equation 6 tan θ (1 – sin2 θ) = 1 for 0 < θ < 2p.
Seterusnya, selesaikan persamaan 6 tan θ (1 – sin2 θ) = 1 bagi 0 < θ < 2p.
[5 marks/markah]
(b) (i) Sketch the graph of y = 3 sin 2θ for 0 < θ < 2p.
Lakarkan graf y = 3 sin 2θ bagi 0 < θ < 2p.
(ii) Hence, on the same axes, draw a suitable straight line to find the number of solutions for the equation
6p tan θ (1 – sin2 θ) = 2p – x for 0 < θ < 2p.
Seterusnya, pada paksi yang sama, lukis garis lurus yang bersesuaian untuk mencari bilangan penyelesaian bagi persamaan
6p tan θ (1 – sin2 θ) = 2p – x bagi 0 < θ < 2p.
[5 marks/markah]

4 (a) Prove that 4 cos3 x − 3 cos x = cos 3x.


Buktikan bahawa 4 kos3 x − 3 kos x = kos 3x.
[4 marks/markah]
(b) (i) Sketch the graph of y = 8 cos3 x − 6 cos x for 0 < x < p.
Lakarkan graf y = 8 kos3 x − 6 kos x bagi 0 < x < p.
(ii) Hence, on the same axes, draw a suitable straight line to find the number of solutions for the equation
p(16 cos3 x − 12 cos x) = 2p − 3x for 0 < x < p.
Seterusnya, menggunakan paksi yang sama, lukis garis lurus yang bersesuaian untuk mencari bilangan penyelesaian bagi persamaan
p (16 kos3 x − 12 kos x) = 2p − 3x bagi 0 < x < p.
[6 marks/markah]

H O TS Zo n e
1 Express cos 3x in terms of cos x. Hence, solve the equation cos 3x + cos x = 0 for 0° < x < 360°. Applying

Ungkapkan kos 3x dalam sebutan kos x. Seterusnya, selesaikan persamaan kos 3x + kos x = 0 bagi 0° < x < 360°.

2 Prove that sin (p + q) sin (p − q) = sin2 p – sin2 q.


Hence, find the value of sin 75° sin 15° without using a calculator. Applying

Buktikan sin (p + q) sin (p − q) = sin p – sin q.


2 2

Seterusnya, cari nilai sin 75° sin 15° tanpa menggunakan kalkulator.

137

F5 Mod A+ ADDM 6(113-137).indd 137 5/1/2021 3:49:12 PM


Learning Area: Application of Social Science

7 Linear Programming
Pengaturcaraan Linear
7.1 Linear Programming Model / Model Pengaturcaraan Linear

Smart Tip
Constraint Inequality
Kekangan Ketaksamaan

y is greater than x y.x


y adalah lebih besar daripada x
y is less than x y,x
y adalah lebih kecil daripada x
y is not more than x y<x
y adalah tidak lebih daripada x
y is not less than x y>x
y adalah tidak kurang daripada x
The sum of x and y is not more than c x+y<c
Jumlah x dan y adalah tidak lebih daripada c
The sum of x and y is not less than c x+y>c
Jumlah x dan y adalah tidak kurang daripada c
The minimum value of y is c y>c
Nilai minimum y ialah c
The maximum value of y is c y<c
Nilai maksimum y ialah c
y is at least k times the value of x y > kx
y adalah sekurang−kurangnya k kali nilai x
y is at most k times the value of x y < kx
y adalah selebih-lebihnya k kali nilai x

Exercise 1 Write down a mathematical model for each of the following situations.
Tuliskan satu model matematik bagi setiap situasi yang berikut.
PL 2 Demonstrate the understanding of linear programming model.

Example 1 1 The perimeter of a piece of rectangular land is less


than 64 m.
A shop is selling two types of bags with the price Perimeter bagi sebuah tanah yang berbentuk segi empat tepat
RM37.90 and RM60 each. The total sales of the shop adalah kurang daripada 64 m.
in a day is at least RM1 000.
Sebuah kedai menjual dua jenis beg dengan harga RM37.90 dan
Let x = the length of the land
RM60 seunit. Jumlah jualan yang diperoleh kedai itu dalam
sehari adalah sekurang-kurangnya RM1 000.
y = the width of the land

Solution Perimeter = 2x + 2y
Let x = number of bags with the price RM37.90 sold [ 2x + 2y , 64
in a day
bilangan beg yang berharga RM37.90 yang dijual
dalam sehari
y = number of bags with the price RM60 sold in
a day
bilangan beg yang berharga RM60 yang dijual dalam
sehari
[ 37.90x + 60y > 1 000

138

F5 Mod A+ ADDM 7(138-153).indd 138 5/1/2021 3:40:25 PM


2 There are some girls and boys on a stage. 3 A stall sells nasi lemak and fried noodles. The price
Terdapat beberapa orang murid perempuan dan murid lelaki di of each packet of nasi lemak is RM2.40 and the price
atas pentas. of each packet of fried noodles is RM1.50. The total
(i) The maximum total number of girls and boys on sales of the stall is at least RM300 daily.
the stage is 25. Sebuah gerai menjual nasi lemak dan mi goreng. Harga
Jumlah bilangan maksimum murid perempuan dan murid sebungkus nasi lemak ialah RM2.40 dan harga sebungkus mi
lelaki di atas pentas ialah 25 orang. goreng ialah RM1.50. Jumlah jualan kedai itu ialah sekurang-
(ii) The number of boys must be more than half of kurangnya RM300 sehari.
the number of girls.
Bilangan murid lelaki mesti melebihi separuh daripada Let x = number of packets of nasi lemak sold
bilangan murid perempuan. y = number of packets of fried noodles sold
Let x = number of girls [ 2.40x + 1.50y > 300
y = number of boys

(i) x + y < 25
1
(ii) y . x
2

Smart Tip
1 If the inequalities are ax + by . c and ax + by > c, shade the region above the straight line ax + by = c.
Jika ketaksamaan ialah ax + by . c dan ax + by > cc, lorekkan rantau di bahagian atas garis lurus ax + by = c.
2 If the inequalities are ax + by , c and ax + by < c, shade the region below the straight line ax + by = c.
Jika ketaksamaan ialah ax + by , c dan ax + by < cc, lorekkan rantau di bahagian bawah garis lurus ax + by = c.
3 The solid line ( ----------------- ) is used for ‘>’ and ‘<’, while the dotted line ( ------- ) is used for ‘.’ and ‘,’.
Garis padu ( ----------------- ) digunakan untuk ‘>’ dan ‘<’,
’, manakala garis sempang ( ------- ) digunakan untuk ‘.’ dan ‘,’.

Exercise 2 Represent each of the following linear inequalities graphically.


Wakilkan setiap ketaksamaan linear yang berikut secara grafik.
PL 2 Demonstrate the understanding of linear programming model.

Example 2
1 x – 3y , 6
x + 2y , 6

Solution
y

4
2

2
x
4 2 0 2 4 6

x 2
0 2 4 6

4

139

F5 Mod A+ ADDM 7(138-153).indd 139 5/1/2021 3:40:26 PM


2 3x + 2y < 12 3 2x – 4y . 16

y y

4
6

x
O 2 4 6 8
2
2
2

x
4 2 0 2 4 4

Exercise 3 Write a mathematical model related to the following situations. Hence, draw a graph to
represent each mathematical model obtained.
Tuliskan model matematik yang berkaitan dengan situasi berikut. Seterusnya, lukiskan graf untuk mewakili setiap
model matematik yang diperoleh.
PL 3 Apply the understanding of linear programming model to perform simple tasks.

Example 3

A restaurant allocates RM6 000 to buy more than 60 tables and chairs. The price of a table and a chair are
RM240 and RM80 respectively.
Sebuah restoran memperuntukkan RM6 000 untuk membeli lebih daripada 60 buah meja dan kerusi. Harga bagi sebuah meja
dan sebuah kerusi masing-masing ialah RM240 dan RM80.

Solution
Let/ Katakan
x = number of tables/ bilangan meja
y = number of chairs/ bilangan kerusi
Constraint II/Kekangan II
Constraint I/ Kekangan I: x + y . 60
y
Constraint II/ Kekangan II: 240x + 80y < 6 000
80
Constraint I/ Kekangan I
y
70

60
60

50
50

40
x + y  60 40

30
30

20
20

10 10

240x + 80y  6 000


x x
0 10 20 30 40 50 60 0 10 20 30 40 50

140

F5 Mod A+ ADDM 7(138-153).indd 140 5/1/2021 3:40:27 PM


1 A new factory is required to install machine P and machine Q. The total number of machines to be installed is
less than 40. The number of machine P is not more than twice the number of machine Q.
Sebuah kilang baharu perlu memasang mesin P dan mesin Q. Jumlah bilangan mesin yang perlu dipasang adalah kurang daripada
40 buah. Bilangan mesin P tidak melebihi dua kali bilangan mesin Q.

Let x = number of machine P and y = number of machine Q


Constraint I: x + y , 40
Constraint II: x < 2y
Constraint I Constraint II
y y

40

40

30

30

20
x + y  40
20

10 x  2y

10

x
0
10 20 30 40
x
0 10 20 30 40

2 A shop sells two types of shoes, A and B with the price RM120 and RM80 respectively. The total sales of the
shop is at least RM24 000 in a week. The total sales of shoes B is more than twice the total sales of shoes A.
Sebuah kedai menjual dua jenis kasut, A dan B masing-masing dengan harga RM120 dan RM80. Jumlah jualan bagi kedai itu dalam
seminggu adalah sekurang-kurangnya RM24 000. Jumlah jualan kasut B adalah lebih daripada dua kali jumlah jualan kasut A.

Let x = number of shoes type A and y = number of shoes type B


Constraint I: 120x + 80y > 24 000
Constraint II: y . 2x
Constraint I Constraint II
y
y

400 300

300 200
y  2x

200 100
120x + 80y  24 000

100 x
0 100 200 300

x
0
100 200 300

141

F5 Mod A+ ADDM 7(138-153).indd 141 5/1/2021 3:40:27 PM


3 A developer plans to build x units of single storey house and y units of double storey house. The number of
single storey houses is at least 150 units more than the number of double storey houses. The total number of
houses to be built is at most 400.
Satu pemaju ingin membina x unit rumah satu tingkat dan y unit rumah dua tingkat. Bilangan rumah satu tingkat adalah sekurang-
kurangnya 150 unit lebih daripada bilangan rumah dua tingkat. Jumlah bilangan rumah yang akan dibina adalah selebih-lebihnya 400
unit.

Let x= number of single storey houses and y = number of double storey houses
Constraint I: x – y > 150
Constraint II: x + y < 400
Constraint I: Constraint II:

y y

400 400

300 300

200 200

100 100 x + y  400


x  y  150

x x
O 0
100 200 300 400 100 200 300 400

Exercise 4 Write the optimal function for each of the following situations.
Tuliskan fungsi optimum bagi setiap situasi yang berikut.
PL 3 Apply the understanding of linear programming model to perform simple tasks.

Example 4

The table shows the selling price and the cost price of two types of foods sold in a stall.
Jadual menunjukkan harga jual dan harga kos bagi dua jenis makanan yang dijual di sebuah gerai.

Food Selling price (RM) Cost price (RM)


Makanan Harga jual (RM) Harga kos (RM)
Nasi lemak 1.60 1.20
Fried noodles/ Mi goreng 2.00 1.45
Write the optimal function for the total profit of the sales.
Tuliskan fungsi optimum bagi jumlah keuntungan jualan.

Solution
Let/Katakan
x = number of packets of nasi lemak sold
bilangan bungkus nasi lemak yang dijual Smart Tip
y = number of packets of fried noodles sold
Optimal function/ Fungsi optimum, k = ax + by
bilangan bungkus mi goreng yang dijual
k = (1.60 – 1.20)x + (2.00 – 1.45)y
k = 0.4x + 0.55y

142

F5 Mod A+ ADDM 7(138-153).indd 142 5/1/2021 3:40:28 PM


1 A shop sells exercise books and workbooks. The 2 A factory produces blue pens and red pens. The
price of each exercise book and each workbook is cost of the production of a blue pen and a red pen is
RM0.80 and RM3.20 respectively. Write the optimal RM1.20 and RM0.85 respectively. Write the optimal
function for the total sales of the shop. function for the total cost of the production of the
Sebuah kedai menjual buku latihan dan buku kerja. Harga bagi pens.
sebuah buku latihan dan sebuah buku kerja masing-masing ialah Sebuah kilang menghasilkan pen biru dan pen merah. Kos
RM0.80 dan RM3.20. Tuliskan fungsi optimum bagi jumlah untuk menghasilkan sebatang pen biru dan sebatang pen merah
jualan di kedai itu. masing-masing ialah RM1.20 dan RM0.85. Tuliskan fungsi
optimum bagi jumlah kos penghasilan pen itu.
Let x = number of exercise books sold
y = number of workbooks sold Let x = number of blue pens produced
y = number of red pens produced
Optimal function:
k = 0.8x + 3.2y Optimal function:
k = 1.2x + 0.85y

3 The table shows the selling price and the cost price of two types of calculators sold in a stationery shop.
Jadual menunjukkan harga jual dan harga kos bagi dua jenis kalkulator yang dijual di sebuah kedai alat tulis.

Type Selling price (RM) Cost price (RM)


Jenis Harga jual (RM) Harga kos (RM)
A 16.00 13.60
PAK-21
B 64.50 51.60

ACTIVITY
Write the optimal function for the total profit obtained by the shop.
Tuliskan fungsi optimum bagi jumlah keuntungan yang diperoleh kedai itu.

Let x = number of calculators A sold


y = number of calculators B sold

Optimal function: k = (16.00 – 13.60)x + (64.50 – 51.60)y


k = 2.4x + 12.9y

PAK-21 Gallery Walk

Steps/Langkah-langkah:
1 Teacher provides a set of questions involving linear programming model on the coloured cards.
Guru menyediakan beberapa set soalan melibatkan model pengaturcaraan linear pada kad berwarna.
Example/Contoh:
(a) Identify the constraints in the problem.
Kenal pasti kekangan yang terdapat dalam masalah.
(b) Form a mathematical model related to the problem.
Tuliskan model matematik yang berkaitan dengan masalah.
(c) Represent each mathematical model obtained in (b) graphically.
Wakilkan setiap model matematik yang diperoleh di (b) secara grafik.
2 Students perform this activity in groups of three students. A coloured card is randomly selected for each group.
Murid melakukan aktiviti ini secara berkumpulan yang terdiri daripada tiga orang murid. Satu kad berwarna dipilih secara rawak
bagi setiap kumpulan.
3 Each group is required to answer all questions on the selected card. Write each answer on a mahjung paper.
Setiap kumpulan dikehendaki menjawab semua soalan yang terdapat pada kad yang dipilih. Tulis setiap jawapan pada kertas
mahjung.
4 The group work of each group is posted on the class’s notice board. Students are required to stand next to their
group work.
Hasil kerja setiap kumpulan ditampal pada papan kenyataan kelas. Murid-murid dikehendaki berdiri di sebelah hasil kerja masing-
masing.
5 A group is required to move to each group to evaluate the work of other groups. Once completed, other groups
need to do the same.
Satu kumpulan dikehendaki bergerak ke setiap kumpulan bagi menilai hasil kerja kumpulan yang lain. Setelah selesai, kumpulan
lain perlu melakukan langkah yang sama.
6 Teacher holds a discussion with students to enhance their understanding.
Guru mengadakan perbincangan dengan murid untuk menambahkan kefahaman mereka.

143

F5 Mod A+ ADDM 7(138-153).indd 143 5/1/2021 3:40:28 PM


Exercise 5 Solve each of the following.
Selesaikan setiap yang berikut.
PL 3 Apply the understanding of linear programming model to perform simple tasks.

Example 5

The diagram shows the shaded region


which satisfies a few constraints of a y

situation.
Rajah menunjukkan rantau berlorek yang 60
memenuhi beberapa kekangan daripada suatu
(20, 54)
situasi.
(a) By using a suitable value of k, draw a 50
straight line k = 3x + 2y on the graph.
On the same graph, draw a straight
line parallel to the line k = 3x + 2y 40
that passes through each point of the
vertices of the region.
Menggunakan satu nilai k yang sesuai, lukis 30
garis lurus k = 3x + 2y pada graf tersebut.
Pada graf yang sama, lukis garis lurus yang
selari dengan garis lurus k = 3x + 2y dan 20
melalui setiap bucu rantau tersebut. (60, 15)
(b) Hence, find the maximum value and (10, 15)
the minimum value of k. 10
Seterusnya, cari nilai maksimum dan nilai
minimum bagi k.
x
O
10 20 30 40 50 60 70

Solution
y
Given/ Diberi k = 3x + 2y
(a) Let/ Katakan k = 12,
[ 3x + 2y = 12 60

(20, 54)
x 0 4
y 6 0 50

(b) Maximum point of the shaded


region = (60, 15) 40
Titik maksimum bagi rantau berlorek
= (60, 15)
k = 3(60) + 2(15) = 210 30
[ The maximum value of k
is 210.
Nilai maksimum bagi k ialah 210. 20
Minimum point of the shaded (60, 15)
region = (10, 15) (10, 15)
Titik minimum bagi rantau berlorek 10
= (10, 15)
k = 3(10) + 2(15) = 60
[ The minimum value of k is 60. x
Nilai minimum bagi k ialah 60. O
10 20 30 40 50 60 70
3x + 2y = 12

144

F5 Mod A+ ADDM 7(138-153).indd 144 5/1/2021 3:40:29 PM


1 The diagram shows the shaded region which satisfies a few constraints of a situation.
Rajah menunjukkan rantau berlorek yang memenuhi beberapa kekangan daripada suatu situasi.

600

(300, 500)
500

400 (500, 400)

300

200

(100, 150)
100

2x +y = 200
x
0
100 200 300 400 500 600

(a) By using a suitable value of k, draw a straight line k = 2x + y on the graph. On the same graph, draw a
straight line parallel to the line k = 2x + y that passes through each point of the vertices of the region.
Menggunakan satu nilai k yang sesuai, lukis garis lurus k = 2x + y pada graf tersebut. Pada graf yang sama, lukis garis lurus
yang selari dengan garis lurus k = 2x + y dan melalui setiap bucu rantau tersebut.
(b) Hence, find the maximum value and the minimum value of k.
Seterusnya, cari nilai maksimum dan nilai minimum bagi k.

Given k = 2x + y
(a) Let k = 200, [ 2x + y = 200

x 0 100
y 200 0

(b) Maximum point of the shaded region = (500, 400)


k = 2(500) + 400 = 1 400
[ The maximum value of k is 1 400.
Minimum point of the shaded region = (100, 150)
k = 2(100) + 150 = 350
[ The minimum value of k is 350.

145

F5 Mod A+ ADDM 7(138-153).indd 145 5/1/2021 3:40:29 PM


2 The diagram shows the shaded region which satisfies a few constraints of a situation.
Rajah menunjukkan rantau berlorek yang memenuhi beberapa kekangan daripada suatu situasi.

700
(600, 680)

600

500 (100, 450)

400

300

200
(600, 150)

100
5x + 2y = 1 000

x
0
100 200 300 400 500 600

(a) By using a suitable value of k, draw a straight line k = 5x + 2y on the graph. On the same graph, draw a
straight line parallel to the line k = 5x + 2y that passes through each point of the vertices of the region.
Menggunakan satu nilai k yang sesuai, lukis garis lurus k = 5x + 2y pada graf tersebut. Pada graf yang sama, lukis garis lurus
yang selari dengan garis lurus k = 5x + 2y dan melalui setiap bucu rantau tersebut.
(b) Hence, find the maximum value and the minimum value of k.
Seterusnya, cari nilai maksimum dan nilai minimum bagi k.

Given k = 5x + 2y
(a) Let k = 1 000, [ 5x + 2y = 1 000

x 0 200
y 500 0

(b) Maximum point of the shaded region = (600, 680)
k = 5(600) + 2(680) = 14 360
[ The maximum value of k is 4 360.
Minimum point of the shaded region = (100, 450)
k = 5(100) + 2(450) = 1 400
[ The minimum value of k is 1 400.

146

F5 Mod A+ ADDM 7(138-153).indd 146 5/1/2021 3:40:30 PM


3 The diagram shows the shaded region which satisfies a few constraints of a situation.
Rajah menunjukkan rantau berlorek yang memenuhi beberapa kekangan daripada suatu situasi.

1 400

1 200

(400, 1 080)

1 000 (900, 960)

800

(200, 700)

600

400

200 (400, 200)

x
0
200 400 600 800 1 000 1 200
x + 2y = 400

(a) By using a suitable value of k, draw a straight line k = x + 2y on the graph. On the same graph, draw a
straight line parallel to the line k = x + 2y that passes through each point of the vertices of the region.
Menggunakan satu nilai k yang sesuai, lukis garis lurus k = x + 2y pada graf tersebut. Pada graf yang sama, lukis garis lurus
yang selari dengan garis lurus k = x + 2y dan melalui setiap bucu rantau tersebut.
(b) Hence, find the maximum value and the minimum value of k.
Seterusnya, cari nilai maksimum dan nilai minimum bagi k.

Given k = x + 2y
(a) Let k = 400, [ x + 2y = 400

x 0 400
y 200 0

(b) Maximum point of the shaded region = (900, 960)
k = 900 + 2(960) = 2 82
0
[ The maximum value of k is 2 820.
Minimum point of the shaded region = (400, 200)
k = 400 + 2(200) = 800
[ The minimum value of k is 800.

147

F5 Mod A+ ADDM 7(138-153).indd 147 5/1/2021 3:40:30 PM


7.2 Application of Linear Programming / Aplikasi Pengaturcaraan Linear
Exercise 6 Solve each of the following problems.
Selesaikan setiap masalah yang berikut.
PL 4 Apply appropriate knowledge and skills of linear programming in the context of simple routine problem solving.

Example 6
A tuition centre offers two subjects, namely Biology and Chemistry for Form Five students. x is the number of
students taking Biology and y is the number of students taking Chemistry. The admission of the students to the
tuition centre must be based on the following constraints.
Sebuah pusat tusyen menawarkan dua mata pelajaran, iaitu Biologi dan Kimia untuk murid-murid Tingkatan Lima. x ialah bilangan
murid yang mengambil Biologi dan y ialah bilangan murid yang mengambil Kimia. Kemasukan murid ke pusat tusyen itu mestilah
berdasarkan kekangan yang berikut.
I The ratio of the number of students taking Biology to the number of students taking Chemistry is not more
than 70 : 30.
Nisbah bilangan murid yang mengambil Biologi kepada bilangan murid yang mengambil Kimia adalah tidak lebih daripada 70 : 30.
II The total number of students for both subjects is less than or equal to 70.
Jumlah bilangan murid untuk kedua-dua subjek adalah kurang atau sama dengan 70.
III The number of students taking Chemistry is at least 8.
Bilangan murid yang mengambil Kimia adalah sekurang-kurangnya 8 orang.
IV The number of students taking Biology is at least 15.
Bilangan murid yang mengambil Biologi adalah sekurang- kurangnya 15 orang.
(a) Write four inequalities, other than x > 0 and y > 0 that satisfy all the above constraints.
Tulis empat ketaksamaan, selain daripada x > 0 dan y > 0 yang memuaskan semua kekangan di atas.
(b) Using a scale of 2 cm to 10 students on both axes, construct and shade the region R which satisfies all the
above constraints.
Dengan menggunakan skala 2 cm kepada 10 orang murid pada kedua-dua paksi, bina dan lorekkan rantau R yang memenuhi semua
kekangan di atas.
(c) Based on the graph constructed in (b), find
Berdasarkan graf yang dibina di (b), cari
(i) the range of the number of students taking Biology if the number of students taking Chemistry is 15,
julat bilangan murid yang mengambil Biologi jika bilangan murid yang mengambil Kimia ialah 15 orang,
(ii) the maximum total fees obtained by the tuition centre if the fees for Biology and Chemistry are RM100
and RM300 respectively.
jumlah yuran maksimum yang diperoleh pusat tusyen itu jika yuran Biologi dan Kimia masing-masing ialah RM100 dan
RM300.

Solution
x 70 (b) y
(a) I: y < 30
3 70
y> 7 x
x = 15
II: x + y < 70
III: y > 8 60
IV: x > 15
50
(c) (i) 15 < x < 35
(ii) Maximum total fees
Jumlah yuran maksimum 40
= 100x + 300y
= 100(15) + 300(55)
3
= RM18 000 30 y =7 x
R

20

x + y = 70
10
y=8

x
O 10 20 30 40 50 60 70

148

F5 Mod A+ ADDM 7(138-153).indd 148 5/1/2021 3:40:30 PM


1 A bakery bakes two types of cakes, chocolate cakes and banana cakes. The bakery bakes x chocolate cakes
and y banana cakes. The production of cakes is based on the following constraints.
Sebuah kedai roti membuat dua jenis kek, iaitu kek coklat dan kek pisang. Kedai roti itu membuat x biji kek coklat dan y biji kek pisang.
Penghasilan kek itu adalah berdasarkan kekangan yang berikut.
I The number of banana cakes produced must exceed the number of chocolate cakes by at most 150.
Bilangan kek pisang yang dihasilkan mesti melebihi bilangan kek coklat selebih-lebihnya 150 biji.
II The number of banana cakes produced is at least half of the number of chocolate cakes.
Bilangan kek pisang yang dihasilkan sekurang-kurangnya separuh daripada bilangan kek coklat.
III The total number of cakes produced is not more than 300.
Jumlah bilangan kek yang dihasilkan tidak lebih daripada 300 biji.
(a) Write three inequalities, other than x > 0 and y > 0 that satisfy all the above constraints.
Tulis tiga ketaksamaan, selain daripada x > 0 dan y > 0 yang memuaskan semua kekangan di atas.
(b) Using a scale of 2 cm to 50 cakes on both axes, construct and shade the region R which satisfies all the
above constraints.
Dengan menggunakan skala 2 cm kepada 50 biji kek pada kedua-dua paksi, bina dan lorekkan rantau R yang memenuhi semua
kekangan di atas.
(c) By using the graph constructed in (b), answer the following questions.
Dengan menggunakan graf yang dibina di (b), jawab soalan yang berikut.
Given the costs of baking a chocolate cake and a banana cake are RM40 and RM20 respectively, find
Diberi kos untuk membuat sebiji kek coklat dan sebiji kek pisang masing-masing ialah RM40 dan RM20, cari
(i) the maximum cost to produce the cakes,
kos maksimum untuk menghasilkan kek itu,
(i) the minimum cost to produce the cakes when the number of banana cakes produced is 200.
kos minimum untuk menghasilkan kek itu apabila bilangan kek pisang yang dihasilkan ialah 200 biji.

(a) I: y − x < 150 (b) y


1
II: y >  x
2
III: x + y < 300
300

(c) (i) Maximum cost


= 40x + 20y
y  x = 150
= 40(200) + 20(100) 250
= RM10 000
(ii) Minimum cost
= 40(50) + 20(200) 200
= RM6 000
1
y = 2x

150

R
100

50
x + y = 300

x
0 50 100 150 200 250 300 350

149

F5 Mod A+ ADDM 7(138-153).indd 149 5/1/2021 3:40:31 PM


2 A shoes shop sells two types of school shoes, black shoes and white shoes. In a particular day, the shop sells x
pairs of black shoes and y pairs of white shoes. The profits from the sales of a pair of black shoes is RM20 and a
pair of white shoes is RM5. The sales of the shoes per day are based on the following constraints.
Sebuah kedai kasut menjual dua jenis kasut, iaitu kasut hitam dan kasut putih. Pada suatu hari tertentu, kedai itu menjual x pasang
kasut hitam dan y pasang kasut putih. Keuntungan daripada jualan sepasang kasut hitam ialah RM20 dan sepasang kasut putih ialah
RM5. Jualan kasut pada setiap hari adalah berdasarkan kekangan yang berikut.
I The total number of pairs of shoes sold are at most 400.
Jumlah bilangan kasut yang dijual selebih-lebihnya 400 pasang.
II The number of black shoes sold are not more than three times the number of white shoes.
Bilangan kasut hitam yang dijual tidak melebihi tiga kali bilangan kasut putih.
III The minimum total profit for the sales of both shoes is RM2 000.
Jumlah keuntungan minimum bagi jualan kedua-dua kasut ialah RM2 000.
(a) Write three inequalities, other than x > 0 and y > 0 that satisfy all the above constraints.
Tulis tiga ketaksamaan, selain daripada x > 0 dan y > 0 yang memuaskan semua kekangan di atas.
(b) Using a scale of 2 cm to 50 pairs of shoes on both axes, construct and shade the region R which satisfies all
the above constraints.
Dengan menggunakan skala 2 cm kepada 50 pasang kasut pada kedua-dua paksi, bina dan lorekkan rantau R yang memenuhi
semua kekangan di atas.
(c) By using the graph constructed in (b), find
Dengan menggunakan graf yang dibina di (b), cari
(i) the minimum number of black shoes sold if the number of white shoes sold is 100,
bilangan minimum kasut hitam yang dijual jika bilangan kasut putih yang dijual ialah 100 pasang,
(i) the maximum total profit of the shop in day.
keuntungan maksimum yang diperoleh kedai itu dalam sehari.

(a) I: x + y < 400 (b) y


II: x < 3y
III: 20x + 5y > 2 000
400
(c) (i) Number of
black shoes
= 75 350
(ii) Maximum profit
= 20(300) + 5(100)
= RM6 500 300

250

200

R
150

x = 3y
100

x + y = 400
50

20x + 5y = 2 000
x
0 50 100 150 200 250 300 350 400

150

F5 Mod A+ ADDM 7(138-153).indd 150 5/1/2021 3:40:31 PM


Review 7
Paper 2
Section C
1 Use a graph paper to answer this question.
Gunakan kertas graf untuk menjawab soalan ini.
A school plans to buy x chocolate cakes and y cheesecakes from a bakery in conjunction with school canteen day. The
prices for a chocolate cake and a cheesecake are RM80 and RM120 respectively. The purchase of the cakes will be based
on the following constraints.
Sebuah sekolah bercadang untuk membeli x biji kek coklat dan y biji kek keju dari sebuah kedai roti sempena hari kantin sekolah. Harga bagi
sebiji kek coklat dan sebiji kek keju masing-masing ialah RM80 dan RM120. Pembelian kek itu adalah berdasarkan kekangan yang berikut.
I The total number of cakes must not be more than 65.
Jumlah bilangan kek mestilah tidak lebih daripada 65 biji.
II The amount allocated is at least RM4 000.
Jumlah wang yang diperuntukkan sekurang-kurangnya RM4 000.
2
III The ratio of the number of chocolate cakes to the number of cheesecakes is more than 3 .
2
Nisbah bilangan kek coklat kepada bilangan kek keju adalah lebih daripada .
3
(a) Write three inequalities, other than x > 0 and y > 0 that satisfy all the above constraints.
Tulis tiga ketaksamaan, selain daripada x > 0 dan y > 0 yang memuaskan semua kekangan di atas.
[3 marks/markah]
(b) Using a scale of 2 cm to 10 cakes on both axes, construct and shade the region R which satisfies all the above
constraints.
Dengan menggunakan skala 2 cm kepada 10 biji kek pada kedua-dua paksi, bina dan lorekkan rantau R yang memenuhi semua
kekangan di atas.
[3 marks/markah]
(c) By using the graph constructed in (b), find
Dengan menggunakan graf yang dibina di (b), cari
(i) the range of the number of cheesecakes if the number of chocolate cakes purchased is 35,
julat bilangan kek keju jika bilangan kek coklat yang dibeli ialah 35 biji,
(ii) the maximum amount of profit obtained by the bakery if the profits of selling a chocolate cake and a
cheesecake are RM16 and RM24 respectively.
jumlah keuntungan maksimum yang diperoleh kedai roti itu jika keuntungan bagi jualan sebiji kek coklat dan sebiji kek keju
masing-masing ialah RM16 dan RM24.
[4 marks/markah]

2 Use a graph paper to answer this question.


Gunakan kertas graf untuk menjawab soalan ini.
A mall wants to install x microphones and y loudspeakers. The mall has an allocation of RM9 000. The installation of
the gadgets are based on the following constraints.
Sebuah pasar raya ingin memasang x buah mikrofon dan y buah pembesar suara. Pasar raya itu mempunyai peruntukan sebanyak
RM9 000. Pemasangan gajet itu adalah berdasarkan kekangan yang berikut.
I The costs of installing a microphone and a loudspeaker are RM150 and RM100 respectively.
Kos pemasangan sebuah mikrofon dan sebuah pembesar suara masing-masing ialah RM150 dan RM100.
II The total number of microphones and loudspeakers is not less than 30.
Jumlah bilangan mikrofon dan pembesar suara tidak kurang daripada 30 buah.
III The number of loudspeakers is at most 45.
Bilangan pembesar suara selebih-lebihnya 45 buah.
IV The number of loudspeakers is at least 75% of the number of microphones.
Bilangan pembesar suara sekurang-kurangnya 75% daripada bilangan mikrofon.
(a) Write four inequalities, other than x > 0 and y > 0 that satisfy all the above constraints.
Tulis empat ketaksamaan, selain daripada x > 0 dan y > 0 yang memuaskan semua kekangan di atas.
[3 marks/markah]
(b) Using a scale of 2 cm to 10 gadgets on both axes, construct and shade the region R which satisfies all the above
constraints.
Dengan menggunakan skala 2 cm kepada 10 gajet pada kedua-dua paksi, bina dan lorekkan rantau R yang memenuhi semua kekangan
di atas.
[3 marks/markah]

151

F5 Mod A+ ADDM 7(138-153).indd 151 5/1/2021 3:40:31 PM


(c) By using the graph constructed in (b), find
Dengan menggunakan graf yang dibina di (b), cari
(i) the maximum number of microphones can be installed if 39 loudspeakers have been installed,
bilangan maksimum mikrofon yang boleh dipasang jika 39 buah pembesar suara dipasang,
(ii) the maximum amount of profit obtained by the seller if the profits of selling a microphone and a
loudspeaker are RM20 and RM40 respectively.
jumlah keuntungan maksimum yang diperoleh penjual jika keuntungan bagi jualan sebuah mikrofon dan sebuah pembesar suara
masing-masing ialah RM20 dan RM40.
[4 marks/markah]

3 Use a graph paper to answer this question.


Gunakan kertas graf untuk menjawab soalan ini.
A bus company provides trips to two towns, P and Q. To meet customers’ demand, the company provides x trips
to town P and y trips to town Q every day. The fares for a trip to town P and a trip to town Q are RM40 and RM80
respectively. The service provided by the company is based on the following constraints.
Sebuah syarikat bas menyediakan trip perjalanan ke dua buah bandar, iaitu bandar P dan bandar Q. Bagi memenuhi permintaan
pelanggan, syarikat itu menyediakan x trip perjalanan ke bandar P dan y trip perjalanan ke bandar Q pada setiap hari. Tambang bagi
satu perjalanan ke bandar P dan satu perjalanan ke bandar Q masing-masing ialah RM40 dan RM80. Perkhidmatan yang disediakan oleh
syarikat itu adalah berdasarkan kepada kekangan yang berikut.
I The total number of trips is at most 15.
Jumlah bilangan perjalanan selebih-lebihnya 15.
II The number of trips to town Q does not exceed two times the number of trips to town P.
Bilangan perjalanan ke bandar Q tidak melebihi dua kali bilangan perjalanan ke bandar P.
III The total fares collected in a day must be more than RM240.
Jumlah tambang yang diperoleh dalam sehari mesti melebihi RM240.
(a) Write three inequalities, other than x > 0 and y > 0 that satisfy all the above constraints.
Tulis tiga ketaksamaan, selain daripada x > 0 dan y > 0 yang memuaskan semua kekangan di atas.
[3 marks/markah]
(b) Using a scale of 2 cm to 2 trips on both axes, construct and shade the region R which satisfies all the above
constraints.
Dengan menggunakan skala 2 cm kepada 2 perjalanan pada kedua-dua paksi, bina dan lorekkan rantau R yang memenuhi semua
kekangan di atas.
[3 marks/markah]
(c) By using the graph constructed in (b), find
Dengan menggunakan graf yang dibina di (b), cari
(i) the range of the number of trips to town P if 6 trips to town Q are provided in a day,
julat bilangan perjalanan ke bandar P jika 6 perjalanan ke bandar Q disediakan dalam sehari,
(ii) the maximum amount of profit per day if the profits for each trip to town P and each trip to town Q are
RM20 and RM10 respectively.
jumlah keuntungan maksimum sehari jika keuntungan daripada setiap perjalanan ke bandar P dan setiap perjalanan ke bandar Q
masing-masing ialah RM20 dan RM10.
[4 marks/markah]

4 Use a graph paper to answer this question.


Gunakan kertas graf untuk menjawab soalan ini.
A shop sells two types of calculators, A and B. The selling prices for a calculator A and a calculator B are RM60 and
RM150 respectively. The shop sells x calculator A and y calculator B based on the following constraints.
Sebuah kedai menjual dua jenis kalkulator, A dan B. Harga jualan bagi sebuah kalkulator A dan kalkulator B masing-masing ialah RM60
dan RM150. Kedai itu menjual x buah kalkulator A dan y buah kalkulator B berdasarkan kekangan yang berikut.
I The total number of calculators in the shop is not more than 70.
Jumlah bilangan kalkulator di dalam kedai itu tidak lebih daripada 70 buah.
II Two times the number of calculator A exceeds the number of calculator B by at least 20.
Dua kali bilangan kalkulator A melebihi bilangan kalkulator B sekurang-kurangnya 20 buah.
III The total sales must be more than RM2 400.
Jumlah jualan mestilah melebihi RM2 400.
(a) Write three inequalities, other than x > 0 and y > 0 that satisfy all the above constraints.
Tulis tiga ketaksamaan, selain daripada x > 0 dan y > 0 yang memuaskan semua kekangan di atas.
[3 marks/markah]
(b) Using a scale of 2 cm to 10 calculators on both axes, construct and shade the region R which satisfies all the
above constraints.
Dengan menggunakan skala 2 cm kepada 10 buah kalkulator pada kedua-dua paksi, bina dan lorekkan rantau R yang memenuhi
semua kekangan di atas.
[3 marks/markah]

152

F5 Mod A+ ADDM 7(138-153).indd 152 5/1/2021 3:40:31 PM


(c) By using the graph constructed in (b), find
Dengan menggunakan graf yang dibina di (b), cari
(i) the maximum number of calculator B sold if 24 calculator A are sold,
bilangan maksimum kalkulator B yang dijual jika 24 buah kalkulator A dijual,
(ii) the maximum amount of profit obtained if the profit for a calculator A is RM8 and the profit for a
calculator B is RM16.
jumlah keuntungan maksimum yang diperoleh jika keuntungan sebuah kalkulator A ialah RM8 dan keuntungan sebuah
kalkulator B ialah RM16.
[4 marks/markah]

H O TS Zo n e
1 An oil company has two oil refineries, A and B. The operating costs per day of oil refinery A and B are RM25 000 and
RM24 000 respectively. The table below shows the amount of oil produced in both oil refineries in a day.
Sebuah syarikat minyak mempunyai dua kilang penapis minyak, A dan B. Kos operasi sehari bagi kilang penapis minyak A dan B masing-
masing ialah RM25 000 dan RM24 000. Jadual di bawah menunjukkan jumlah minyak yang dihasilkan oleh dua buah kilang penapis
minyak itu dalam sehari.

Amount of oil produced (barrels)


Oil refinery Jumlah minyak yang dihasilkan (tong)
Kilang penapis minyak High grade Medium grade Low grade
Gred tinggi Gred sederhana Gred rendah

A 50 100 150
B 100 50 100

The company receives a demand to supply 800 barrels of high grade oil, 1 000 barrels of medium grade oil and 1 800
barrels of low grade oil. How many days are needed for each oil refinery to meet the demand with the minimum
operating cost? Evaluating

Syarikat itu menerima permintaan untuk membekalkan 800 tong minyak gred tinggi, 1 000 tong minyak gred sederhana dan 1 800 tong
minyak gred rendah. Berapa harikah yang diperlukan oleh setiap kilang penapis minyak untuk memenuhi permintaan itu dengan kos
operasi yang minimum?

153

F5 Mod A+ ADDM 7(138-153).indd 153 5/1/2021 3:40:31 PM


Learning Area: Application of Science and Technology

8 Kinematics of Linear Motion


Kinematik Gerakan Linear
Displacement, Velocity and Acceleration as a Function of Time
8.1 Sesaran, Halaju dan Pecutan sebagai Fungsi Masa

Smart Tip
1 A distance of a particle from a fixed point, O, measured in a specific direction is known as instantaneous displacement, s.
Jarak suatu zarah dari satu titik tetap, O, yang diukur pada arah tertentu dikenali sebagai sesaran seketika, s.
s = negative s = positive
negatif s=0 positi
positif

to the left of O particle at O to the right of O


ke sebelah kiri O zarah di titik O ke sebelah kanan O

2 The rate of change of displacement with respect of time is known as instantaneous velocity, v.
Kadar perubahan sesaran terhadap masa dikenali sebagai halaju seketika, v.
v zero/ sifar v positive/ positif v negative/ negatif
negati

at rest O move to the left


keadaan rehat move to the right bergerak ke kiri
bergerak ke kanan

3 When a particle/Apabila suatu zarah


(a) stops instantaneously, v = 0 m s–1,
berhenti seketika, v = 0 m s–1,
(b) moves to the right, v . 0 m s–1,
bergerak ke kanan, v . 0 m s–1,
(c) moves to the left, v  0 m s–1.
bergerak ke kiri, v  0 m s–1.
4 The rate of change of the instantaneous velocity with respect of time is known as instantaneous acceleration, a.
Kadar perubahan halaju seketika terhadap masa dikenali sebagai pecutan seketika, a.

Exercise 1 Solve each of the following.


Selesaikan setiap yang berikut.
PL 3 Apply the understanding of displacement, velocity and acceleration to perform simple tasks.

Example 1

A particle moves along a straight line and passes through a fixed point O. Its displacement, s m, t seconds after
passing through O is given by s = 5t 2 − t. Find
Suatu zarah bergerak di sepanjang satu garis lurus dan melalui satu titik tetap O. Sesaran, s m, zarah itu pada masa t saat selepas melalui
O diberi oleh s = 5t2 − t. Cari
(a) the instantaneous displacement, in m, of the particle when t = 4,
sesaran seketika, dalam m, zarah itu apabila t = 4,
(a) the distance, in m, travelled by the particle in the fourth second.
jarak, dalam m, yang dilalui oleh zarah itu dalam saat keempat.

Solution The distance travelled by the particle in the


(a) s = 5(4)2 − 4 = 76 m fourth second
(b) When/Apabila t = 4, s4 = 5(4)2 – 4 = 76 m Jarak yang dilalui oleh zarah dalam saat keempat
When/Apabila t = 3, s3 = 5(3)2 – 3 = 42 m = |76 – 42|
t=4
= 34 m
t=3

O 42 76 Smart Tip
in the fourth second The distance travelled in the
dalam saat keempat nth second
Jarak yang dilalui pada saat ke-n
= |Sn − Sn −1|

154

F5 Mod A+ ADDM 8(154-170).indd 154 5/1/2021 3:38:25 PM


1 A particle moves along a straight line from a fixed 2 A particle moves along a straight line and passes
point O. Its velocity, v m s–1, t seconds after passing through a fixed point O. Its displacement, s m, is
through O is given by v = 3t2 − 21t + 30. Find given by s = t2 − 4t, where t is the time in seconds
Suatu zarah bergerak di sepanjang satu garis lurus dari satu after passing through O. Find
titik tetap O. Halaju, v m s–1, zarah itu pada masa t saat selepas Suatu zarah bergerak di sepanjang satu garis lurus dan melalui
melalui O diberi oleh v = 3t2 − 21t + 30. Cari satu titik tetap O. Sesarannya, s m, diberi oleh s = t2 − 4t,
(a) the time, in seconds, when the particle is dengan keadaan t ialah masa dalam saat selepas melalui O. Cari
instantaneously at rest, (a) the instantaneous displacement, in m, when
masa, dalam saat, apabila zarah itu berehat seketika, t = 1,
(b) the range of time, in seconds, when the particle sesaran seketika, dalam m, apabila t = 1,
moves to the left. (b) the distance, in m, travelled at the fourth
julat masa, dalam saat, apabila zarah itu bergerak ke kiri. second,
jarak, dalam m, yang dilalui pada saat keempat,
(a) v = 0 (c) the value of t, in seconds, when the particle
3t2 − 21t + 30 = 0 (4 3) passes through point O again.
t2 − 7t + 10 = 0 nilai t, dalam saat, apabila zarah itu melalui titik O
(t – 2)(t – 5) = 0 semula.
t – 2 = 0 or t – 5 = 0
t = 2 s t = 5 s (a) s = (1)2 − 4(1) = −3 m
The particle is located 3 m to the left from the
The particle stops at t = 2 s or t = 5 s fixed point O.
(b) v < 0
3t2 − 21t + 30  0 (b) s4 = 42 − 4(4) = 0
t2 − 7t + 10  0 s3 = 32 − 4(3) = −3
(t – 2)(t – 3)  0 Distance = |s4 − s3| =|0 − (−3)| = 3 m
2 s  t  3 s
(c) t 2 − 4t = 0
t (t − 4) = 0

2 3
t t = 0 or t − 4 = 0
t =4
2st3s
The particle passes through O again when t = 4.
3 A particle moves along in a straight line from a 4 A particle moves along a straight line from a fixed
fixed point O. Its displacement, s m, t seconds after point O. Its velocity, v m s–1, t seconds after passing
passing through O is given by s = 2t2 − 44t. Find through O is given by v = 2t2 − 18t + 36. Find
Suatu zarah bergerak di sepanjang satu garis lurus dari satu Suatu zarah bergerak di sepanjang satu garis lurus dari satu
titik tetap O. Sesarannya, s m, pada masa t saat selepas melalui titik tetap O. Halajunya, v m s–1, pada masa t saat selepas
O diberi oleh s = 2t2 − 44t. Cari melalui O diberi oleh v = 2t2 − 18t + 36. Cari
(a) the time, in seconds, when the particle passes (a) the time, in seconds, when the particle is
through the point O again, instantaneously at rest,
masa, dalam saat, apabila zarah itu melalui titik O sekali masa, dalam saat, apabila zarah itu berhenti seketika,
lagi, (b) the range of time, in seconds, when the particle
(b) the distance, in m, travelled by the particle moves to the right.
during the third second. julat masa, dalam saat, apabila zarah itu bergerak ke
jarak, dalam m, yang dilalui oleh zarah itu dalam saat kanan.
ketiga.
(a) v = 0
(a) s = 0 2t2 − 18t + 36 = 0 (4 2)
2t2 − 44t = 0 t2 − 9t + 18 = 0
2t(t − 22) = 0 (t – 3)(t – 6) = 0
t = 0 or t − 22 = 0 t – 3 = 0 or t – 6 = 0
t = 22 t = 3 t = 6
[ t = 22 s The particle stops at t = 3 s or t = 6 s

(b) s3 = 2(3)2 − 44(3) = − 114 (b) v . 0


s2 = 2(2)2 − 44(2) = −80 2t2 − 18t + 36 . 0 (4 2)
Distance = |−114 − (−80)| t2 − 9t + 18 . 0
= |−34| (t – 3)(t – 6) . 0
= 34 m 0 s < t , 3 s or t . 6 s

t
3 6

155

F5 Mod A+ ADDM 8(154-170).indd 155 5/1/2021 3:38:26 PM


Exercise 2 Solve each of the following.
Selesaikan setiap yang berikut.
PL 3 Apply the understanding of displacement, velocity and acceleration to perform simple tasks.

Example 2 1 A particle moves along a straight line from a fixed


point O. Its displacement, s m, t seconds after passing
A particle moves along a straight line from a fixed through O is given by s = 6t – t 2 – 1. Find the total
point O. Its displacement, s m, t seconds after distance, in m, travelled by the particle in the first
passing through O is given by s = 4t – t2 + 2. Find 4 seconds.
the total distance, in m, travelled by the particle in Suatu zarah bergerak di sepanjang satu garis lurus dan melalui
the first 6 seconds. satu titik tetap O. Sesaran, s m, zarah itu pada masa t saat
Suatu zarah bergerak di sepanjang satu garis lurus dari satu selepas melalui O diberi oleh s = 6t – t2 − 1. Cari jumlah jarak,
titik tetap O. Sesarannya, s m, pada masa t saat selepas melalui
dalam m, yang dilalui oleh zarah itu dalam 4 saat yang pertama.
O diberi oleh s = 4t – t2 + 2. Cari jumlah jarak, dalam m, yang
dilalui oleh zarah itu dalam 6 saat yang pertama.
t (s) 0 1 2 3 4
Solution
Given/ Diberi s = 4t – t2 + 2 s (m) –1 4 7 8 7
t (s) 0 1 2 3 4 5 6
9m
s (m) 2 5 6 5 2 –3 –10
1 4 7 8

4m 1m

10 3 2 5 6 The total distance travelled in the first 4 seconds


=9+1
16 m = 10 m
The total distance travelled in the first 6 seconds
Jumlah jarak yang dilalui dalam 6 saat yang pertama
= 4 + 16
= 20 m

2 A particle moves along a straight line and passes 3 A particle moves along a straight line and passes
through a fixed point O. Its displacement, s m, through a fixed point O. Its displacement, s m, is
t seconds after passing through O is given by given by s = 8t – t 2 – 5, where t is the time in seconds
s = t 2 – 4t + 2. Calculate the total distance, in m, after passing through O. Calculate the total distance,
travelled by the particle in the first 5 seconds. in m, travelled by the particle in the first 7 seconds.
Suatu zarah bergerak di sepanjang satu garis lurus dan melalui Suatu zarah bergerak di sepanjang satu garis lurus dan melalui
satu titik tetap O. Sesaran, s m, zarah itu pada masa t saat satu titik tetap O. Sesarannya, s m, diberi oleh s = 8t – t2 – 5,
selepas melalui O diberi oleh s = t2 – 4t + 2. Hitung jumlah dengan keadaan t ialah masa dalam saat selepas melalui O.
jarak, dalam m, yang dilalui oleh zarah itu dalam 5 saat yang Hitung jumlah jarak, dalam m, yang dilalui oleh zarah itu dalam
pertama. 7 saat yang pertama.

t (s) 0 1 2 3 4 5 t (s) 0 1 2 3 4 5 6 7

s (m) 2 –1 –2 –1 2 7 s (m) –5 2 7 10 11 10 7 2

4m
16 m

2 1 2 7 5 2 7 10 11
9m
9m

The total distance travelled in the first 5 seconds The total distance travelled in the first 7 seconds
=4+9 = 16 + 9
= 13 m = 25 m

156

F5 Mod A+ ADDM 8(154-170).indd 156 5/1/2021 3:38:26 PM


8.2 Differentiation in Kinematics of Linear Motion/ Pembezaan dalam Kinematik Gerakan Linear

Smart Tip
ds dv d2s
v = dt a= =
dt dt2

Displacement, s Velocity, v Acceleration, a


Sesaran, s Halaju, v Pecutan, a

Exercise 3 Solve each of the following.


Selesaikan setiap yang berikut.
PL 3 Apply the understanding of displacement, velocity and acceleration to perform simple tasks.

Example 3
1 A particle moves along a straight line so that its
displacement, s m, from a fixed point O is given
A particle moves along a straight line and passes
by s = t3 − 12t2, where t is the time in seconds after
through a fixed point O. Its displacement, s m,
movement starts. Find
is given by s = t2 − 4t + 1, where t is the time in
Satu zarah bergerak di sepanjang satu garis lurus supaya
seconds after passing through O. Find
sesarannya, s m, dari titik tetap O diberi oleh s = t3 − 12t2,
Suatu zarah bergerak di sepanjang satu garis lurus dan melalui
dengan keadaan t ialah masa dalam saat selepas gerakan
satu titik tetap O. Sesarannya, s m, diberi oleh s = t2 − 4t + 1,
bermula. Cari
dengan keadaan t ialah masa dalam saat selepas melalui O. Cari
(a) the displacement, in m, when the particle is
(a) the time, in seconds, when the particle stops
instantaneously at rest,
instantaneously,
sesaran, dalam m, apabila zarah itu berehat seketika.
masa, dalam saat, apabila zarah itu berhenti seketika,
(b) the range of values of t, in seconds, when the
(b) the instantaneous velocity, in m s–1, of the
particle moves to the left.
particle when t = 5,
julat nilai t, dalam saat, apabila zarah itu bergerak ke kiri.
halaju seketika, dalam m s–1, zarah itu apabila t = 5,
(c) the range of values of t, in seconds, when the
particle moves to the right, s = t3 − 12t2
ds
julat nilai t, dalam saat, apabila zarah itu bergerak ke v= = 3t2 − 24t
kanan, dt
(d) the initial velocity, in m s–1, of the particle. (a) When v = 0
halaju awal, dalam m s–1, zarah itu. 3t2 − 24t = 0
3t(t − 8) = 0
Solution t = 0 or t = 8
s = t2 − 4t + 1 t = 0 is not accepted. Hence, t = 8 s.
ds
v= = 2t – 4 When t = 8, s = 83 − 12(8)2
dt
= −256 m
(a) When the particle stops instantaneously, v = 0 The particle is 256 m to the left of O.
Apabila zarah berhenti seketika, v = 0
2t − 4 = 0 (b) v0
2t = 4 3t2 − 24t  0
t=2s 3t(t − 8)  0

(b) When t = 5/ Apabila t = 5,


v = 2(5) − 4
= 6 m s–1 t
0 8
0st8s
(c) v .0
2t − 4 .0
2t .4
t .2s

(d) When/Apabila t = 0,
v = 2(0) – 4
= −4 m s–1

157

F5 Mod A+ ADDM 8(154-170).indd 157 5/1/2021 3:38:27 PM


2 The velocity of a particle that moves along a straight 3 A particle moves along a straight line. Its
line and passes through a fixed point O is given by displacements, s m, is s = 3t3 + t2 – 5t at t seconds
v = 3 − 2t + t2, where t is the time in seconds after after passing through a fixed point O. Find
passing through O. Find Satu zarah bergerak di sepanjang satu garis lurus. Sesarannya,
Halaju bagi satu zarah yang bergerak di sepanjang satu garis s m, adalah s = 3t3 + t2 – 5t pada masa t saat ketika melalui satu
lurus dan melalui satu titik tetap O diberi oleh v = 3 − 2t + t2, titik tetap O. Cari
dengan keadaan t ialah masa dalam saat selepas melalui O. Cari (a) the initial velocity, in m s–1, of the particle,
(a) the initial acceleration, in m s , of the particle,
–2
halaju awal, dalam m s–1, zarah itu,
pecutan awal, dalam m s , zarah itu,
–2
(b) the initial acceleration, in m s–2, of the particle,
(b) the instantaneous acceleration, in m s , of the –2
pecutan awal, dalam m s–2, zarah itu,
particle when t = 4. (c) the instantaneous acceleration, in m s–2, of the
pecutan seketika, dalam m s , zarah itu apabila t = 4.
–2
particle when t = 3.
pecutan seketika, dalam m s–2, zarah itu apabila t = 3.
dv
(a) a = = –2 + 2t
dt ds
v= = 9t2 + 2t – 5
When t = 0, a = −2 + 2(0) dt
= −2 m s–2
(a) When t = 0, v = 9(0)2 + 2(0) − 5
= –5 m s–1
(b) When t = 4, a = −2 + 2(4)
= −2 + 8 dv
(b) a = = 18t + 2
= 6 m s–2 dt
When t = 0, a = 18(0) + 2
= 2 m s–2

(c) When t = 3, a = 18(3) + 2


= 56 m s–2

Exercise 4 Solve each of the following.


Selesaikan setiap yang berikut.
PL 3 Apply the understanding of displacement, velocity and acceleration to perform simple tasks.

Example 4

A particle moves along a straight line and passes through a fixed point O. Its displacement, s m, t seconds after
passing through O is given by s = −2t3 + 5t2 + 4t. Find
Suatu zarah bergerak di sepanjang satu garis lurus dan melalui satu titik tetap O. Sesaran, s m, zarah itu pada masa t saat selepas melalui
O diberi oleh s = −2t3 + 5t2 + 4t. Cari
(a) the value of t, in seconds, when the particle is instantaneously at rest,
nilai t, dalam saat, apabila zarah itu berehat seketika, Smart Tip
(b) the range of values of t, in seconds, when the particle moves to the left, The velocity is maximum or
julat nilai t, dalam saat, apabila zarah itu bergerak ke kiri, dv
minimum when = a = 0.
(c) the maximum velocity, in m s–1, of the particle. dt
Halaju maksimum atau minimum
halaju maksimum, dalam m s–1, zarah itu. dv
apabila = a = 0.
dt

Solution
dv
(a) s = −2t3 + 5t2 + 4t (b) v0 (c) =0
ds dt
v= = −6t2 + 10t + 4 −6t2 + 10t + 4  0 4 (−2) −12t + 10 = 0
dt 3t2 – 5t – 2 . 0 5
When/Apabila v = 0, (3t + 1)(t – 2) . 0 t=
6
−6t2 + 10t + 4 = 0 4 (−2)
3t2 – 5t – 2 = 0 Vmaximum/maksimum
(3t + 1)(t – 2) = 0
( 56 ) + 10( 56 ) + 4
2
= −6
1
t = − 3 or t = 2 1
t . 0, thus, t = 2 s =8 m s−1
1 2
t 6
3
t.2s

158

F5 Mod A+ ADDM 8(154-170).indd 158 5/1/2021 3:38:27 PM


1 A particle moves along a straight line. Its 2 A particle moves along a straight line and passes
1 through a fixed point O. Its displacement, s m, is
displacement, in m, is s = − 3 t3 + t2 + 24t at t seconds
given by s = t3 − 15t2 + 72t + 40, where t is the time
after passing through a fixed point O. Find in seconds, after passing through O. Find
Satu zarah bergerak di sepanjang satu garis lurus. Sesarannya, Suatu zarah bergerak di sepanjang satu garis lurus dan
1
dalam m, adalah s = − t3 + t2 + 24t pada masa t saat ketika melalui satu titik tetap O. Sesarannya, s m, diberi oleh
3
melalui satu titik tetap O. Cari s = t 3 − 15t2 + 72t + 40, dengan keadan t ialah masa, dalam
(a) the time, in seconds, when the particle stops saat, selepas melalui O. Cari
instantaneously, (a) the time, in seconds, when the particle is
masa, dalam saat, apabila zarah itu berhenti seketika, instantaneously at rest,
(b) the range of time, in seconds, when the particle masa, dalam saat, apabila zarah itu berehat seketika,
moves to the right, (b) the range of values of t, in seconds, when the
julat masa, dalam saat, apabila zarah itu bergerak ke particle moves to the left,
kanan, julat nilai t, dalam saat, apabila zarah itu bergerak ke kiri,
(c) the value of t, in seconds, when the velocity of (c) the minimum velocity, in m s–1, of the particle.
the particle is uniform. halaju minimum, dalam m s–1, zarah itu.
nilai t, dalam saat, apabila halaju zarah seragam.
(a) s = t3 − 15t2 + 72t + 40
1 ds
(a) s = − t3 + t2 + 24t v= = 3t2 − 30t + 72
3 dt
ds
v= = −t2 + 2t + 24 When v = 0,
dt
3t2 − 30t + 72 = 0 (÷ 3)
When v = 0, t2 – 10t + 24 = 0
−t2 + 2t + 24 = 0 (t − 4)(t – 6) = 0
t2 – 2t – 24 = 0 t − 4 = 0 or t – 6 = 0
(t − 6)(t + 4) = 0 t = 4 t = 6
t − 6 = 0 or t + 4 = 0 t = 4 s or t = 6 s
t = 6 t = −4
t > 0, thus t = 6 s (b) v  0
3t2 − 30t + 72  0 (4 3)
(b) v > 0 t2 – 10t + 24  0
−t2 + 2t + 24 . 0 × (−1) (t − 4)(t – 6)  0
t2 – 2t – 24 , 0
(t − 6)(t + 4)  0

t
4 6
t
4 6 4st6s

t . 0, thus 0 s < t  6 s dv
(c) = 6t − 30
dv dt
(c) Uniform velocity, =0 dv
dt When =0
−2t + 2 = 0 dt
−2t = 2 6t − 30 = 0
t = 1 s 6t = 30
t = 5
vminimum = 3(5)2 − 30(5) + 72
= –3 m s–1

159

F5 Mod A+ ADDM 8(154-170).indd 159 5/1/2021 3:38:28 PM


Exercise 5 Solve each of the following.
Selesaikan setiap yang berikut.
PL 3 Apply the understanding of displacement, velocity and acceleration to perform simple tasks.

Example 5 1 A particle moves along a straight line and passes


through a fixed point O. Its displacement, s m,
A particle moves along a straight line and passes
t seconds after passing through O is given by
through a fixed point O. Its displacement, s m, is 1
given by s = −t3 + 2t2 + 7t, where t is the time in s = − 3 t3 + 3t2 + 27t. Find
seconds after passing through O. Find Suatu zarah bergerak di sepanjang satu garis lurus dan melalui
Suatu zarah bergerak di sepanjang satu garis lurus dan melalui satu titik tetap O. Sesaran, s m, zarah itu pada masa t saat
satu titik tetap O. Sesarannya, s m, diberi oleh s = −t3 + 2t2 + 7t, 1
dengan keadan t ialah masa dalam saat selepas melalui O. Cari selepas melalui O diberi oleh s = − t3 + 3t2 + 27t. Cari
3
(a) the initial acceleration, in m s–2, of the particle, (a) the initial acceleration, in m s , of the particle,
–2

pecutan awal, dalam m s , zarah itu,


–2 pecutan awal, dalam m s–2, zarah itu,
(b) the instantaneous acceleration, in m s–2, of the (b) the instantaneous acceleration, in m s–2, of the
particle when t = 3. particle when t = 4.
pecutan seketika, dalam m s–2, zarah itu apabila t = 3. pecutan seketika, dalam m s–2, zarah itu apabila t = 4.

Solution 1
s = − t3 + 3t2 + 27t
s = −t3 + 2t2 + 7t 3
ds
ds v= = −t2 + 6t + 27
v= = −3t2 + 4t + 7 dt
dt
d2s
d2s a = 2 = −2t + 6
a = 2 = −6t + 4 dt
dt
(a) When/Apabila t = 0, a = −6(0) + 4 (a) When t = 0, a = −2(0) + 6 = 6 m s−2
= 4 m s−2
(b) When t = 4, a = −2(4) + 6 = −2 m s−2
(b) When/Apabila t = 3, a = −6(3) + 4
= −14 m s−2

2 A particle moves along a straight line from a fixed 3 A particle moves along a straight line from a fixed
point O. Its velocity, v m s–1, t seconds after passing point O. Its velocity, v m s–1, t seconds after passing
through O is given by v = 12 – (t – 4)2. Find through O is given by v = −10t + 2t2 − 6. Find
Suatu zarah bergerak di sepanjang satu garis lurus dari satu Suatu zarah bergerak di sepanjang satu garis lurus dari satu
titik tetap O. Halajunya, v m s–1, pada masa t saat selepas titik tetap O. Halajunya, v m s–1, pada masa t saat selepas
melalui O diberi oleh v = 12 – (t – 4)2. Cari melalui O diberi oleh v = −10t + 2t2 − 6. Cari
(a) the initial acceleration, in m s–2, of the particle, (a) the initial acceleration, in m s–2, of the particle,
pecutan awal, dalam m s , zarah itu,
–2
pecutan awal, dalam m s–2, zarah itu,
(b) the instantaneous acceleration, in m s , of the –2
(b) the instantaneous acceleration, in m s–2, of the
particle when t = 5. particle when t = 4.
pecutan seketika, dalam m s–2, zarah itu apabila t = 5. pecutan seketika, dalam m s–2, zarah itu apabila t = 4.

v = 12 – (t – 4)2 v = −10t + 2t2 − 6


= 12 – (t2 – 8t + 16) dv
a= = −10 + 4t
= 12 – t2 + 8t − 16 dt
= −t2 + 8t − 4
dv (a) When t = 0, a = −10 + 4(0) = −10 m s−2
a= = −2t + 8
dt
(b) When t = 4, a = −10 + 4(4) = 6 m s−2
(a) When t = 0, a = −2(0) + 8 = 8 m s −2

(b) When t = 5, a = −2(5) + 8 = −2 m s−2

160

F5 Mod A+ ADDM 8(154-170).indd 160 5/1/2021 3:38:28 PM


8.3 Integration in Kinematics of Linear Motion/ Pengamiran dalam Kinematik Gerakan Linear

Smart Tip
Displacement, s Velocity, v Acceleration, a
Sesaran, s Halaju, v Pecutan, a

s = v dt v = a dt

Exercise 6 Solve each of the following.


Selesaikan setiap yang berikut.
PL 3 Apply the understanding of displacement, velocity and acceleration to perform simple tasks.

Example 6

A particle moves along a straight line and passes through a fixed point O. Its velocity, v m s–1, is given by
v = t2 − 9t + 18, where t is the time in seconds after leaving point O.
Suatu zarah bergerak di sepanjang satu garis lurus dan melalui satu titik tetap O. Halajunya, v m s–1, diberi oleh v = t2 − 9t + 18,
dengan keadaan t ialah masa dalam saat selepas meninggalkan titik O.
(a) Find the initial velocity, in m s–1, of the particle.
Cari halaju awal, dalam m s–1, zarah itu.
(b) Find the maximum displacement, in m, of the particle.
Cari sesaran maksimum, dalam m, zarah itu.
(c) Sketch a velocity-time graph for a time period 0 < t < 6.
Lakarkan graf halaju-masa untuk tempoh masa 0 < t < 6.
(d) Find the total distance, in m, travelled by the particle in the first 5 seconds.
Cari jumlah jarak, dalam m, yang dilalui oleh zarah itu dalam 5 saat yang pertama.

Solution
(a) When/Apabila t = 0, (c) v
v = (0)2 − 9(0) + 18 = 18 m s–1

(b) v=0 18
t − 9t + 18 = 0
2

(t − 6)(t − 3) = 0 t
t = 6 or/ atau t = 3 0 3 6
s = ∫ v dt
(d) Area above the x-axis/ Luas di bahagian atas paksi-x
= ∫ t2 − 9t + 18 dt 3
t3 9t2 = (t2 − 9t + 18) dt
0
= 3 − 2 + 18t + c
[
t3 9t2
]
3
= – + 18t
When/Apabila t = 0, s = 0, 3 2 0

(0)3 9(3)3
0 = 3 − 2 + 18(0) + c
= [
(3)3 9(3)2
3

2
+ 18(3) –
3 ] [
(0)3 9(0)2

2
+ 18(0) ]
c=0 = 22.5 − 0
t3 9t2 = 22.5 m
s = 3 − 2 + 18t
Smart Tip Area below the x-axis/ Luas di bahagian bawah paksi-x
When/Apabila t = 3, The displacement is 5
= (t2 − 9t + 18) dt
(3)3 9(3)2 3

[ ]
s = 3 − 2 + 18(3) maximum or minimum t3 9t2 5
ds = – + 18t
= 22.5 m when = v = 0. 3 2 3
dt

When/Apabila t = 6,
Sesaran maksimum atau
minimum apabila
ds
= v = 0.
= [
(5)3 9(5)2
3

2
+ 18(5) –
3 ] [
(3)3 9(3)2

2
+ 18(3) ]
dt 1
(6)3 9(6)2 = 19 6 − 22.5
s = 3 − 2 + 18(6)
= 18 m 1
= −3 3 m
Maximum displacement = 22.5 m
Sesaran maksimum = 22.5 m |
1
Total distance/Jumlah jarak = 22.5 + −3 3 |
5
= 25 6 m

161

F5 Mod A+ ADDM 8(154-170).indd 161 5/1/2021 3:38:29 PM


1 A particle moves along a straight line from a fixed point O. Its velocity, v m s–1, t seconds after passing
through O is given by v = 4 − 11t – 3t2. Find
Suatu zarah bergerak di sepanjang satu garis lurus dari satu titik tetap O. Halajunya, v m s−1, pada masa t saat selepas melalui O
diberi oleh v = 4 − 11t – 3t2. Cari
(a) the displacement, s, of the particle from O as a function of t,
sesaran, s, zarah itu dari O sebagai fungsi t,
(b) the displacement, in m, of the particle when the particle stops instantaneously,
sesaran, dalam m, zarah itu apabila zarah itu berhenti seketika,
(c) the instantaneous displacement, in m, of the particle when t = 2 s.
sesaran seketika, dalam m, zarah itu apabila t = 2 s.

(a) s = ∫ v dt (b) v = 0
= ∫ (4 − 11t − 3t2) dt 4 − 11t − 3t2 = 0
11t 2 3t 3 3t2 + 11t − 4 = 0
= 4t − 2 − 3 + c
(3t − 1)(t + 4) = 0
11t 2 3t – 1 = 0 or t + 4 = 0
= 4t − 2 − t3 + c
3t = 1 t = −4
1
When t = 0, s = 0, t =3
11(0)2 1
0 = 4(0) − 2 − (0)3 + c t . 0, thus t = 3
c=0
11 ()
1 2


11t 2
Thus, s = 4t − 2 − t3 s = 4 ()
1
3

2
3
−()1 3
3
37
= m
54

11(2)2
(c) s = 4(2) − − (2)3
2
= −22 m
The particle is located 22 m to the left from
the fixed point O when t = 2.

2 The acceleration of a particle that moves along a straight line, t seconds after passing through a fixed point
O is given by a = t − 6. Given the initial velocity of the particle is 5 m s−1. Find
Pecutan suatu zarah yang bergerak di sepanjang satu garis lurus, pada masa t saat selepas melalui titik tetap O diberi oleh
a = t − 6. Diberi halaju awal zarah itu ialah 5 m s−1. Cari
(a) the minimum velocity, in m s–1, of the particle,
halaju minimum, dalam m s–1, zarah itu,
(b) the range of values of t, in seconds, when the particle is decelerating.
julat nilai t, dalam saat, apabila zarah itu mengalami nyahpecutan.

(a) a = 0 (b) When the particle is decelerating, a  0


t − 6 = 0 t – 6 < 0
t =6 t 6
v = ∫ a dt
v = ∫ (t − 6) dt
t2
= − 6t + c
2
When t = 0, v = 5 m s–1
(0)2
5= – 6(0) + c
2
c = 5
t2
[ v = – 6t + 5
2
(6) 2
vminimum = – 6(6) + 5
2
= –13 m s–1

162

F5 Mod A+ ADDM 8(154-170).indd 162 5/1/2021 3:38:29 PM


3 A particle moves along a straight line and passes through a fixed point O with an initial velocity of 18 m s−1.
Its acceleration, a m s−2, t seconds after passing through O is given by a = 7 − 2t. The particle stops
instantaneously after t seconds.
Suatu zarah bergerak di sepanjang satu garis lurus dan melalui satu titik tetap O dengan halaju awal 18 m s−1. Pecutannya, a m s−2,
pada masa t saat selepas melalui O diberi oleh a = 7 − 2t. Zarah itu berhenti seketika selepas t saat.
(a) Find/Cari
(i) the maximum velocity, in m s−1, of the particle,
halaju maksimum, dalam m s−1, zarah itu,
(ii) the value of t/ nilai t.
(b) Sketch a velocity-time graph for a time period 0 < t < 9,
Lakarkan graf halaju-masa untuk tempoh masa 0 < t < 9,
(c) Find the total distance, in m, travelled by the particle in the first t seconds.
Cari jumlah jarak, dalam m, yang dilalui oleh zarah itu dalam t saat yang pertama.

(a) (i) a = 7 − 2t (b)


v
When maximum velocity, a = 0
7 − 2t = 0 30 1
2t = 7 4
7
t= 2
18
v = ∫ (7 − 2t) dt
t
2t 2 O
= − 2 + 7t + c 31 9 PAK-21
2

ACTIVITY
= −t2 + 7t + c
(c) s = ∫ v dt
When t = 0, v = 18 9
18 = −(0)2 + 7(0) + c = (−t2 + 7t + 18) dt
0
c = 18
[
t3 7t2
]
9
= – + + 18t
3 2 0
Thus, v =−t2 + 7t + 18
vmax = −
7 2
() ()
+7
7
+ 18
= – [
(9)3 7(9)2
3
+
2 ] [
+ 18(9) – –
(0)3 7(0)2
3
+
2
+ 18(0) ]
2 2
= 202.5 – 0
1
= 30 m s−1 = 202.5 m
4
(ii) When the particles stops at t, v = 0
−t2 + 7t + 18 = 0
t2 − 7t − 18 = 0
(t − 9)(t + 2) = 0
t = 9 or t = −2
t > 0, thus t = 9 s

PAK-21 Think-Pair-Share

Steps/Langkah-langkah:
1 Students choose their partner.
Murid memilih pasangan masing-masing.
2 Teacher gives a set of question related to the kinematics of linear motion to each pair.
Guru memberikan satu set soalan berkaitan kinematik gerakan linear kepada setiap pasangan murid.
Example/ Contoh:

A particle moves along a straight line. Its displacement, in m, is s = t3 – 3t2 – 4t at t seconds after passing
through a fixed point O. Find the initial velocity, in m s –1, of the particle.
Suatu zarah bergerak di sepanjang satu garis lurus. Sesarannya, dalam m, adalah s = t3 – 3t2 – 4t pada masa t saat
selepas melalui satu titik tetap O. Cari halaju awal, dalam m s –1, zarah itu.

3 In pairs, students are required to discuss and solve the question given.
Secara berpasangan, murid dikehendaki berbincang dan menyelesaikan soalan yang diberi.
4 Students share their solution and explain the answers obtained to all the students in the class.
Murid berkongsi penyelesaian dan menjelaskan jawapan yang diperoleh kepada semua murid di dalam kelas.
5 Teacher evaluates the answer and the other students compare their answers.
Guru menilai jawapan dan murid lain membandingkan jawapan mereka.

163

F5 Mod A+ ADDM 8(154-170).indd 163 5/1/2021 3:38:29 PM


Exercise 7 Solve each of the following.
Selesaikan setiap yang berikut.
PL 3 Apply the understanding of displacement, velocity and acceleration to perform simple tasks.

Example 7

A particle moves along a straight line and passes through a fixed point O. Its acceleration, a m s−2, t seconds
after passing through O is given by a = −6t + 12. Given the initial velocity of the particle is 36 m s−1. Find
Suatu zarah bergerak di sepanjang satu garis lurus dan melalui satu titik tetap O. Pecutannya, a m s−2, pada masa t saat selepas melalui
O diberi oleh a = −6t + 12. Diberi halaju awal zarah ialah 36 m s−1. Cari
(a) the range of values of t, in seconds, when the particle moves to the left,
julat nilai t, dalam saat, apabila zarah itu bergerak ke kiri,
(b) the instantaneous displacement of the particle, in m, when t = 2.
sesaran seketika zarah itu, dalam m, apabila t = 2.

Solution
(a) v = ∫ a dt (b) s = ∫ v dt
= ∫ (−6t + 12) dt s = ∫ (−3t2 + 12t + 36) dt
= −3t2 + 12t + c = −t3 + 6t2 + 36t + c
When/Apabila t = 0, v = 36 When/Apabila t = 0, s = 0
36 = −3(0)2 + 12(0) + c 0 = −(0)3 + 6(0)2 + 36(0) + c
c = 36 c=0
Thus/ Maka, v = −3t2 + 12t + 36
Thus/maka, s = −t3 + 6t2 + 36t
When the particle moves to the left, v  0
When/Apabila t = 2,
Apabila zarah bergerak ke kiri, v  0
s = −(2)3 + 6(2)2 + 36(2)
−3t2 + 12t + 36  0
= 88 m
t2 − 4t – 12 . 0
(t + 2)(t – 6) . 0
[ The instantaneous displacement is 88 m
when t = 2.
Sesaran seketika ialah 88 m apabila t = 2.

t
2 6

t . 0, thus/maka t . 6 s

1 A particle moves along a straight line so that its acceleration, a m s−2, t seconds after passing through a fixed
point O is given by a = − 2t + 3. Given the initial velocity of the particle is 4 m s−1. Find the maximum velocity,
in m s–1, of the particle.
Suatu zarah bergerak di sepanjang satu garis lurus supaya pecutannya, a m s−2, t saat selepas melalui titik tetap O diberi oleh
a = − 2t + 3. Diberi halaju awal zarah ialah 4 m s−1. Cari halaju maksimum, dalam m s–1, zarah itu.

v = ∫ a dt d2v
= ∫ (−2t + 3) dt dt2 =–2, ( 0), v is maximum
3
= −t2 + 3t + c When t = 2
When t = 0, v = 4
4 = −(0)2 + 3(0) + c () ()
3 2 3
vmax = – 2 + 3 2 + 4
c=4 = 6.25 m s–1
Thus, v = −t2 + 3t + 4
Maximum velocity
–2t + 3 = 0
2t = 3
3
t=2

164

F5 Mod A+ ADDM 8(154-170).indd 164 5/1/2021 3:38:30 PM


2 A particle moves along a straight line from a fixed point O with an initial velocity of 12 m s−1. Given the
acceleration is −6 m s−2. Find
Suatu zarah bergerak di sepanjag satu garis lurus dari satu titik tetap O dengan halaju awal ialah 12 m s−1. Diberi pecutan ialah
–6 m s−2. Cari
(a) the range of values of t, in seconds, when the particle moves to the right,
julat nilai t, dalam saat, apabila zarah itu bergerak ke kanan,
(b) the value of t, in seconds, when the particle passes through point O for the second time.
nilai t, dalam saat, apabila zarah itu melalui titik O buat kali kedua.

(a) v = ∫ a dt (b) s = ∫ v dt
= ∫ (−6) dt s = ∫ (−6t + 12) dt
= −6t + c = −3t² + 12t + c
When t = 0, v = 12 When t = 0, s = 0
v = −6t + c 0 = − 3(0)² + 12(0) + c
12 = − 6(0) + c c = 0
c = 12
Thus, s = −3t² + 12t
Thus, v = −6t + 12
When s = 0,
When the particle moves to −3t² + 12t = 0
the right, v . 0. 3t2 – 12t = 0
− 6t + 12 . 0 3t(t – 4) = 0
−6t . −12 t = 0 or t = 4
t , 2
The particle passes through O for the
second time when t = 4.

3 A particle moves along a straight line from a fixed point O with an initial velocity of 9 m s−1. Its acceleration,
a m s−2, t seconds after passing through O is given by a = −4t + 3. Find
Suatu zarah bergerak di sepanjang satu garis lurus dari satu titik tetap O dengan halaju awal 9 m s−1. Pecutannya, a m s−2, pada
masa t saat selepas melalui O diberi oleh a = −4t + 3. Cari
(a) the range of values of t, in seconds, when the particle moves to the left,
julat nilai t, dalam saat, apabila zarah itu bergerak ke kiri,
(b) the instantaneous displacement, in m, of the particle when t = 4.
sesaran seketika, dalam m, zarah itu apabila t = 4.

(a) v = ∫ a dt (b) s = ∫ v dt
= ∫ (−4t + 3) dt s = ∫ (−2t2 + 3t + 9) dt
= −2t2 + 3t + c 2 3
x = − 3  t³ + 2  t2 + 9t + c
When t = 0, v = 9 3 3
2
9 = −2(0)2 + 3(0) + c When t = 0, s = 0
c = 9 2 3
t . 0, thus, t . 3 s 0 = − 3  (0)³ + 2  (0)2 + 9(0) + c
Thus, v = −2t2 + 3t + 9 c=0
When the particle moves to 2 3
the left, v  0 s = − 3  t³ + 2  t2 + 9t
−2t2 + 3t + 9  0
When t = 4,
2t2 − 3t – 9 . 0
2 3
(2t + 3)(t – 3) . 0 s = − 3  (4)³ + 2  (4)2 + 9(4)
1
= 17 3   m

165

F5 Mod A+ ADDM 8(154-170).indd 165 5/1/2021 3:38:30 PM


8.4 Application of Kinematics of Linear Motion/ Aplikasi Kinematik Gerakan Linear
Exercise 8 Solve each of the following.
Selesaikan setiap yang berikut.
PL 4 Apply appropriate knowledge and skills of kinematics of linear motion in the context of simple routine problem solving.

Example 8
1 An object moves along a straight line and passes
An object moves along a straight line from a fixed through a fixed point O with an initial velocity of
point O. Its velocity, v m s–1, at t seconds after 14 m s−1. Its acceleration, a m s−2, at t seconds after
passing through O is given by v = 13t – 3t2 + 10. passing through O is given by a = 12 – 4t.
Find Suatu objek bergerak di sepanjang satu garis lurus dari satu
titik tetap O dengan halaju awal 14 m s−1. Pecutannya, a m s−2,
Suatu objek bergerak di sepanjang satu garis lurus dari satu
pada masa t saat selepas melalui O diberi oleh a = 12 – 4t.
titik tetap O. Halaju, v m s−1, objek itu pada masa t saat
(a) Find the instantaneous displacement, in m, of
selepas melalui O diberi oleh v = 13t – 3t2 + 10. Cari
the object when its velocity is uniform.
(a) the acceleration, in m s–2, of the object when Cari sesaran seketika, dalam m, objek itu apabila halajunya
t = 1, adalah seragam.
pecutan, dalam m s−2, bagi objek itu apabila t = 1,
(b) Sketch a velocity-time graph for 0 < t < 7.
(b) the total distance, in m, travelled by the object Lakarkan graf halaju-masa untuk 0 < t < 7.
in the first 6 seconds.
jumlah jarak, dalam m, yang dilalui oleh objek itu dalam (a) Given a = 12 – 4t
6 saat yang pertama. v = ∫ a dt
= ∫ (12 − 4t) dt
Solution
= 12t − 2t2 + c
(a) v = 13t – 3t2 + 10
dv When t = 0, v = 14
a= = 13 – 6t
dt 14 = 12(0) – 2(0)2 + c
When/ Apabila t = 1, c = 14
a = 13 – 6(1) = 7 m s−2
Thus, v = 12t − 2t2 + 14
(b) When the object stops, v = 0 s = ∫ v dt
Apabila objek berhenti, v = 0 = ∫ (12t − 2t2 + 14) dt
2
13t – 3t2 + 10 = 0 = 6t2 – t3 + 14t + c
3
3t2 – 13t – 10 = 0 When t = 0, s = 0,
(3t + 2)(t – 5) = 0 2
3t + 2 = 0 or t – 5 = 0 0 = 6(0)2 – (0)3 + 14(0) + c
3
2 c=0
t = −3 t=5 2
Thus, s = 6t2 – t3 + 14t
t . 0, thus/ maka t = 5 v 3
v = 13t  3t2 + 10 For uniform velocity, a = 0
12 – 4t = 0
t=3
10 A 2
s = 6(3)2 – (3)3 + 14(3)
3
= 78 m
t
Area A/ Luas A 0 5 B6
5 (b) v = – 2t2 + 12t +14
= (13t – 3t2 + 10) dt
0 = −2(t2 – 6t – 7)
[
13 2 3
]
5
= t – t + 10t = −2(t + 1)(t – 7)
2 0

= [
13 2
2
(5) – (5) + 10(5) – 0
3
] v

= 87.5 m v = 2t2 + 12t + 14

Area B/ Luas B
6
= (13t – 3t2 + 10) dt
5
14

[
13 2 3
]
6
= t – t + 10t 0
t
2 5 7

= [
13 2
2
(6) – (6)3 + 10(6) –
13 2
2 ] [
(5) – (5)3 + 10(5) ]
= − 9.5 m
Total distance = Area A + Area B
Jumlah jarak = Luas A + Luas B
= 87.5 +|–9.5|
= 97 m

166

F5 Mod A+ ADDM 8(154-170).indd 166 5/1/2021 3:38:31 PM


2 An object moves along a straight line and passes through a fixed point O. Its velocity, v m s−1, at t seconds after
passing through O is given by v = 9t – 4t2 + 9. Find
Suatu objek bergerak di sepanjang satu garis lurus dan melalui satu titik tetap O. Halaju, v m s−1, objek itu selepas melalui O diberi
oleh v = 9t – 4t2 + 9. Cari
1
(a) the acceleration, in m s–2, of the object when t = 2  ,
1
pecutan, dalam m s–2, objek itu apabila t =  ,
2
(b) the distance, in m, travelled by the object in the first 5 seconds.
jarak, dalam m, yang dilalui objek itu dalam 5 saat yang pertama.

(a) v = 9t – 4t2 + 9
dv
a= = 9 – 8t
dt
1
When t = 2 ,

a=9–8 2
1
()
= 5 m s–2
(b) When the object stops,
v = 0
9t – 4t2 + 9 = 0
4t2 – 9t – 9 = 0
(4t + 3)(t – 3) = 0
4t + 3 = 0 or t – 3 = 0
3 t = 3
t = − 4
t . 0, thus, t = 3
v
v = 9t  4t2 + 9

9 A
5
t
0 3 B

B
46
Area A
3
= 0 (9t – 4t2 + 9) dt
9
[ 4
]
3
= t2 – t3 + 9t
2 3 0

[
9 2 4 3 9 4
= (3) – (3) + 9(3) – (0)2 – (0)3 + 9(0)
2 3 2 3 ] [ ]
= 31.5

Area B
5
= 3 (9t – 4t2 + 9) dt

[9 4
]
5
= t2 – t3 + 9t
2 3 3

[9 2 4 3 9 4
] [
= (5) – (5) + 9(5) – (3)2 – (3)3 + 9(3)
2 3 2 3 ]
1
= –9 – 31.5
6
2
= –40
3
Total distance
= Area A + Area B
= 31.5 + –40
2
3| |
1
= 72 m
6

167

F5 Mod A+ ADDM 8(154-170).indd 167 5/1/2021 3:38:31 PM


3 An object moves along a straight line and passes through a fixed point O. Its displacement, s m, at t seconds
after passing through O is given by s = mt3 + pt2 + 10t, where m and p are constants. If the maximum velocity of
the object is 18 m s−1 when t = 2, calculate
Suatu objek bergerak di sepanjang satu garis lurus dan melalui satu titik tetap O. Sesarannya, s m, pada masa t saat selepas melalui O
diberi oleh s = mt3 + pt2 + 10t, dengan keadaan m dan p ialah pemalar. Jika halaju maksimum objek itu ialah 18 m s−1 apabila t = 2, hitung
(a) the value of m and of p,
nilai m dan nilai p,
(b) the distance, in m, travelled by the object after 8 seconds.
jarak, dalam m, yang dilalui oleh objek itu selepas 8 saat.

ds 2
(a) v = = 3mt2 + 2pt + 10 (b) s = − 3 t3 + 4t2 + 10t
dt
dv ds
a = = 6mt + 2p v = = −2t2 + 8t + 10
dt dt
At maximum velocity, a = 0, t = 2 When v = 0,
0 = 6m(2) + 2p −2t2 + 8t + 10 = 0
12m + 2p = 0 t2 – 4t – 5 = 0
6m + p = 0 (t + 1)(t – 5) = 0
p = − 6m 1 t = –1 or t = 5
t . 0, thus t = 5
Maximum velocity = 18, t = 2
v
vmaximum = 3m(2)2 + 2p(2) + 10 v = 2t2 + 8t + 10
18 = 12m + 4p + 10
12m + 4p = 8
3m + p = 2 2 10 A

Substitute 1 into 2 , t
0 5 B8
3m + (−6m) = 2
−3m = 2
2 54
B
m = − 3  

2 Area A
Substitute m = − 3   into 1 , 5
= 0 (–2t2 + 8t + 10) dt
p = −6 − 3  
2
( ) 2
[
= – t3 + 4t2 + 10t
5
]
= 4 3 0
2
[ m = – and p = 4
3
2 3
[ 2
] [
= – (5) + 4(5) + 10(5) – – (0)3 + 4(0)2 + 10(0)
3
2
3 ]
2
= 66
3

Area B
8
= 5 (–2t2 + 8t + 10) dt
2
[ ]
8
= – t3 + 4t2 + 10t
3 5
2 3
3 [ 2 2
] [
= – (8) + 4(8) + 10(8) – – (5)3 + 4(5)2 + 10(5)
3 ]
1 2
= –5 – 66
3 3
= −72
2
Total distance = 66 + |–72|
3
2
= 138 m
3

168

F5 Mod A+ ADDM 8(154-170).indd 168 5/1/2021 3:38:31 PM


Review 8
Paper 2

Section C
1 A particle moves along a straight line from a fixed point O with an initial velocity of –6 m s−1. Its acceleration, a m s−2,
at t seconds after passing through O is given by a = 5 – 2t.
[Assume the movement to the right is positive]
Suatu zarah bergerak di sepanjang satu garis lurus dari satu titik tetap O dengan halaju awal –6 m s−1. Pecutan, a m s−2, zarah itu pada
masa t saat selepas melalui O diberi oleh a = 5 – 2t.
[Anggapkan gerakan ke arah kanan adalah positif]
Find/Cari
(a) the range of time, in seconds, when the particle is decelerating,
julat masa, dalam saat, apabila zarah itu mengalami nyahpecutan,
[2 marks/markah]
(b) the time, in seconds, when the particle is instantaneously at rest,
masa, dalam saat, apabila zarah itu berehat seketika,
[4 marks/markah]
(c) the total distance, in m, travelled by the particle in the first 4 seconds.
jumlah jarak, dalam m, yang dilalui oleh zarah itu dalam 4 saat yang pertama.
[4 marks/markah]

2 A particle moves along a straight line from fixed point O. Its velocity, v m s−1, t seconds after passing through O is given
by v = kt2 – 8t, where k is a constant. The acceleration of the particle is 16 m s−2 when t = 2 s.
[Assume the movement to the right is positive]
Suatu zarah bergerak di sepanjang satu garis lurus dari satu titik tetap O. Halajunya, v m s−1, pada masa t saat selepas melalui O diberi
oleh v = kt2 – 8t, dengan keadaan k ialah pemalar. Pecutan zarah itu ialah 16 m s−2 apabila t = 2 s.
[Anggapkan gerakan ke arah kanan adalah positif]
Find/Cari
(a) the value of k,
nilai k,
[3 marks/markah]
(b) the range of values of t, in seconds, when the particle moves to the left,
julat nilai t, dalam saat, apabila zarah itu bergerak ke kiri,
[2 marks/markah]
(c) the value of t, in seconds, when the particle is instantaneously at rest,
nilai t, dalam saat, apabila zarah itu berehat seketika,
[2 marks/markah]
(d) the total distance, in m, travelled by the particle in the first 3 seconds.
jumlah jarak, dalam m, yang dilalui oleh zarah itu dalam 3 saat yang pertama.
[3 marks/markah]

3 A particle starts from a fixed point O and moves along a straight line. After t seconds, its velocity, v m s−1, is given
by v = 16 + 6t − t 2. Calculate
[Assume the movement to the right is positive]
Suatu zarah bermula dari satu titik tetap O dan bergerak di sepanjang satu garis lurus. Selepas t saat, halajunya, v m s−1, diberi oleh
v = 16 + 6t − t2. Hitung
[Anggapkan gerakan ke arah kanan adalah positif]
(a) the instantaneous velocity, in m s−1, of the particle when t = 5,
halaju seketika, dalam m s−1, zarah itu apabila t = 5,
[2 marks/markah]
(b) the range of values of t, in seconds, when the particle moves to the right,
julat nilai t, dalam saat, apabila zarah itu bergerak ke kanan,
[2 marks/markah]
(c) the maximum velocity, in m s−1, of the particle,
halaju maksimum, dalam m s−1, bagi zarah itu,
[3 marks/markah]
(d) the total distance, in m, travelled by the particle in the first 9 seconds.
jumlah jarak, dalam m, yang dilalui oleh zarah itu dalam 9 saat pertama.
[3 marks/markah]

169

F5 Mod A+ ADDM 8(154-170).indd 169 5/1/2021 3:38:31 PM


4 A particle moves along a straight line and passes through a fixed point O. Its velocity, v m s−1, at t seconds after passing
through O is given by v = mt2 + nt, where m and n are constants. The particle stops instantaneously when t = 2 and its
acceleration is −6 m s−2 when t = 3. Calculate
[Assume the movement to the right is positive]
Suatu zarah bergerak di sepanjang satu garis lurus dan melalui satu titik tetap O. Halajunya, v m s−1, pada masa t saat selepas melalui O
diberi oleh v = mt2 + nt, dengan keadaan m dan n ialah pemalar. Zarah itu berhenti seketika apabila t = 2 dan pecutannya ialah –6 m s−2
apabila t = 3. Hitung
[Anggapkan gerakan ke arah kanan adalah positif]
(a) the value of m and of n,
nilai m dan nilai n,
[3 marks/markah]
(b) the instantaneous velocity, in m s–1, of the particle when t = 3,
halaju seketika, dalam m s–1, zarah itu apabila t = 3,
[2 marks/markah]
(c) the range of time, in seconds, when the particle moves to the left,
julat masa, dalam saat, apabila zarah itu bergerak ke kiri,
[2 marks/markah]
(d) the distance, in m, travelled by the particle at the fifth second.
jarak, dalam m, yang dilalui oleh zarah itu pada saat kelima.
[3 marks/markah]

H O TS Zo n e
1 An object moves along a straight line from a fixed point O. Its acceleration, a m s–2, at t seconds after passing
through O is given by a = 18 – ht, where h is a constant. When t = 2, the particle has a deceleration of 6 m s−2 and a
velocity of 72 m s−1. When t = 3, the particle is at P and when t = 4.5, the particle is at Q.
Suatu zarah bergerak di sepanjang satu garis lurus dari satu titik tetap O. Pecutannya, a m s–2, pada masa t saat selepas melalui O diberi
oleh a = 18 – ht, dengan keadaan h ialah pemalar. Apabila t = 2, zarah itu mengalami nyahpecutan 6 m s–2 dan mempunyai halaju
72 m s–1. Apabila t = 3, zarah itu berada di P dan apabila t = 4.5, zarah itu berada di Q.
(a) Find/ Cari
(i) the value of h,
nilai h,
(ii) the velocity at P.
halaju di P.
(b) Sketch a velocity-time graph for the particle. Analysing

Lakar graf halaju-masa bagi zarah itu.


(c) Hence, find the distance, in m, between P and Q. Evaluating

Seterusnya, cari jarak, dalam m, di antara P dan Q.

170

F5 Mod A+ ADDM 8(154-170).indd 170 5/1/2021 3:38:32 PM

You might also like